Электронная библиотека
Форум - Здоровый образ жизни
Акупунктура, Аюрведа Ароматерапия и эфирные масла,
Консультации специалистов:
Рэйки; Гомеопатия; Народная медицина; Йога; Лекарственные травы; Нетрадиционная медицина; Дыхательные практики; Гороскоп; Правильное питание Эзотерика


Политех

Политехнический музей – национальный музей науки и техники, один из крупнейших научно-технических музеев мира.

Миссия музея – просвещение и популяризация научных и технических знаний:

• Мы верим, что миром движут любопытство и созидание.

• Мы открываем людям прошлое, настоящее и будущее науки.

• Мы создаем территорию просвещения, свободной мысли и смелого эксперимента.


Среди просветительских проектов музея – многочисленные выставки, знаменитый Лекторий, Научные лаборатории для детей, Фестиваль актуального научного кино, а также запущенная в 2014 году издательская программа, цель которой – поддержка самых качественных научно-популярных книг, отобранных экспертами музея и выпущенных в сотрудничестве с лучшими издательствами страны.

Вы держите в руках одну из этих книг.


Подробнее о музее и его проектах – на сайте www.polymus.ru

Отказ от ответственности

Не пытайтесь повторить это дома. Автор этой книги всего лишь рисует комиксы в Интернете – он не эксперт по безопасности и не специалист в области здравоохранения. Кроме того, автор обожает смотреть, как что-нибудь горит или взрывается, и это его увлечение вряд ли всегда пойдет вам на пользу.


Издательство и автор не несут ответственности за разрушительные последствия, которые могут прямо или косвенно возникнуть в результате чтения этой книги.

Введение

Книга, которую вы держите в руках, – сборник ответов на гипотетические вопросы в жанре «А что, если?»

Эти вопросы приходят на мой сайт, где я не только веду нечто вроде рубрики советов для безумных ученых, но и выкладываю свой комикс xkcd.

Я не всегда рисовал комиксы. В свое время я изучал физику, а после окончания университета занимался робототехникой в НАСА, но в конце концов я ушел из этой конторы, чтобы полностью посвятить себя комиксам. Однако мой интерес к науке и математике никуда не делся и нашел себе новое применение – теперь я отвечаю на странные (а иногда вызывающие тревогу) вопросы в Интернете. В настоящей книге представлена подборка моих любимых ответов с сайта плюс немало новых вопросов, на которые я здесь отвечаю впервые.

Сколько себя помню, я пытался с помощью математики найти ответы на необычные вопросы. Когда мне было пять лет, моя мама записала один наш разговор и сохранила его. Когда она узнала, что я пишу эту книгу, она нашла записку и прислала ее мне. Вот этот диалог, дословно воспроизведенный с листочка бумаги, которому уже 25 лет.


Рэндалл: Каких предметов в нашем доме больше – мягких или твердых?

Джули: Не знаю.

Рэндалл: А во всем мире?

Джули: Понятия не имею.

Рэндалл: Ну, в каждом доме ведь есть три или четыре подушки, да?

Джули: Ну да.

Рэндалл: И в каждом доме есть где-то пятнадцать магнитиков, да?

Джули: Скорее всего.

Рэндалл: Значит, 15 плюс 3 или плюс 4… пусть будет 4… Получится 19, верно?

Джули: Верно.

Рэндалл: Значит, всего будет где-то три миллиарда мягких предметов и… пять миллиардов твердых. Ну и кто победил?

Джули: Похоже, твердые!


До сего дня я не имею ни малейшего представления, откуда я взял три и пять миллиардов. Очевидно, тогда я не вполне понимал, как работают цифры.

В математике я с годами набрался опыта, но причина, по которой я ею занимаюсь, та же, что была в пять лет: я хочу отвечать на вопросы.

Говорят, что глупых вопросов не бывает. Это неправда: думаю, например, что мой вопрос про твердые и мягкие предметы довольно глупый. Но оказывается, что если вы попытаетесь серьезно ответить на глупый вопрос, это может завести вас в довольно любопытные места.

Я все еще не знаю, каких предметов в мире больше – твердых или мягких, но, странствуя по этим любопытным местам, я узнал многое другое. Перед вами мои любимые эпизоды этих путешествий.

Рэндалл Манро

Всемирный ураган

ВОПРОС: А что, если Земля и все, что на ней находится, внезапно перестанет вращаться, но при этом атмосфера сохранит свою скорость движения?

– Эндрю Браун

ОТВЕТ: Практически все мы умрем. Но вот потом станет интересно.

В районе экватора поверхность Земли движется со скоростью 470 м/с относительно земной оси. Если бы Земля остановилась, а воздух продолжил бы движение, то в результате возник бы ураган со скоростью свыше 1000 км/ч.

Сильнее всего этот ураган бушевал бы на экваторе, но все, кто живет между 42° с. ш. и 42° ю. ш., а это примерно 85 % населения земного шара, внезапно оказались бы в зоне действия ветра, дующего со сверхзвуковой скоростью.



Вблизи земли такой ветер продержится всего несколько минут – трение о земную поверхность скоро его замедлит. Однако этих нескольких минут будет достаточно, чтобы обратить практически все построенное человеком в руины.

Вблизи полюсов ветра были бы слабее, но там нет городов, которые смогли бы избежать разрушения благодаря удаленности от экватора. Лонгйир, расположенный на норвежском острове Шпицберген, один из самых северных в мире поселков, и его тоже разрушили бы ветра, сравнимые по мощи с сильнейшими тропическими циклонами.

Если бы вы искали возможность переждать эту катастрофу, то некоторые шансы у вас появились бы в Хельсинки. Хотя столица Финляндии находится за 60° с. ш., ветер все равно сровняет ее с землей. Однако пласт скалистой породы, на котором стоит Хельсинки, пронизан сложной системой тоннелей, в которых прячутся подземный супермаркет, хоккейная площадка, бассейн и многое другое.



Не уцелело бы ни одного строения: даже достаточно прочные, чтобы устоять при таком ветре, оказались бы под угрозой. Как сказал однажды комик Рон Уайт, проблема не в том, что ветер дует, а в том, на что именно он дует.

Представьте себе, что вы сидите в огромном бункере, построенном из материала, вполне способного выдержать ветер, дующий со скоростью 1000 км/ч.



Бункер – это хорошо, и с вами все было бы в порядке… если бы у вас у одного был бункер. К сожалению, у вас, вероятно, есть соседи, и у соседей тоже есть бункеры, и если бункер соседа окажется менее устойчивым, чем ваш, вашим бункерам придется пережить столкновение на скорости 1000 км/ч.



Все человечество не погибнет[1]. Лишь немногие из людей на поверхности выживут: летящие обломки уничтожат все, что не было защищено настолько, чтобы пережить как минимум ядерный взрыв. Тем не менее многие люди под землей остались бы в живых. Если бы вы находились в глубоком подвале, а еще лучше – в тоннеле метро, когда Земля остановилась, у вас был бы неплохой шанс выжить.

Будут и другие счастливчики. Несколько десятков ученых и сотрудников научной станции Амундсен – Скотт на Южном полюсе были бы в безопасности. Для них первым признаком беды стало бы то, что весь внешний мир в радиоэфире внезапно замолчал.

Эта таинственная тишина, вероятно, заинтриговала бы полярников на какое-то время, но в конце концов кто-нибудь из них обратил бы внимание на кое-что еще более необычное.


Воздух

Когда ветер у поверхности уляжется, начнутся еще более странные события.

Порыв ветра превратится в тепловую волну. Обычно кинетическая энергия порыва ветра не слишком велика, и ею можно пренебречь в расчетах, но мы же имеем дело не с каким-то заурядным ветерком! В общем, когда ветер стихнет, воздух станет нагреваться.

На поверхности Земли это приведет к аномальному повышению температуры, а в зонах влажного климата к бурям и грозам.

В то же время бушующие над океаном ураганы превратят верхний слой воды в водяную пыль. На какое-то время у океана вообще не станет поверхности, и будет невозможно понять, где заканчивается водяная пыль и начинается толща воды.

Океаны холодные. Средняя температура воды под тонким поверхностным слоем составляет всего 5°C. Буря подняла бы холодную воду из глубин. Обилие очень холодной водяной пыли в перегретом воздухе породило бы погоду, которой раньше на Земле никогда не бывало: одновременно ветер, туман, мелкий дождь и резкие перепады температуры.

Приток воды из глубины дал бы сильный толчок к развитию жизни, ведь свежие питательные вещества поднялись бы к верхним слоям. Но одновременно это привело бы к вымиранию множества рыб, крабов, морских черепах и других животных, неспособных дышать в поднявшейся из глубин воде с низким содержанием кислорода. А морским животным, которые дышат воздухом, например китам или дельфинам, было бы трудно выжить на постоянно меняющейся границе моря и воздуха.

Гигантские волны прокатились бы по всему земному шару, с востока на запад, и каждый берег, обращенный на восток, пережил бы самый грандиозный шторм в своей истории. Сначала сушу накрыло бы ослепляющее облако водяной пыли, а за ним пришла бы грохочущая стена воды наподобие цунами. В некоторых местах эти волны проникли бы на много километров вглубь побережья.

Ураганы выбросили бы в атмосферу огромное количество пыли и обломков. Одновременно с этим плотное облако тумана сформировалось бы над холодными поверхностями океана. В обычной ситуации это привело бы резкому падению температур. Так бы и произошло. Во всяком случае, на одной стороне Земли.

Если бы Земля перестала вращаться, нормальный цикл дня и ночи прекратил бы свое существование. Солнце продолжало бы свое видимое движение по небу, но восходы и заходы случались бы теперь раз в год. День и ночь длились бы по шесть месяцев, даже на экваторе. На дневной половине Земли поверхность постоянно жарилась бы на солнце, тогда как на ночной стороне температура резко бы снизилась. Конвекция (перемешивание теплых и холодных слоев воздуха) на дневной стороне привела бы к возникновению мощных штормов и ураганов.



Земля стала бы отчасти напоминать Венеру в начале существования Солнечной системы. Специфика вращения Венеры такова, что, как и наша остановившаяся Земля, эта планета месяцами обращена к Солнцу одной и той же стороной. Однако в ее плотной атмосфере весьма быстрая циркуляция, благодаря чему температура на дневной и ночной стороне приблизительно одинакова.

Хотя длина дня изменилась бы, продолжительность месяца осталась бы прежней. Луна не прекратила бы вращаться вокруг Земли, однако перестала бы удаляться от Земли (как она это делает сейчас из-за приливных сил) и медленно начала приближаться к нам снова.

Более того, Луна сможет в какой-то мере исправить разрушения. Сейчас Земля вращается вокруг своей оси быстрее, чем Луна вокруг Земли, и притяжение нашего спутника замедляет вращение Земли, одновременно отталкивая от нас Луну[2]. Если бы Земля перестала вращаться, Луна перестала бы удаляться от нас и вместо того, чтобы замедлять Землю, начала бы ее ускорять. Потихоньку, помаленьку гравитация Луны тянула бы нашу планету за собой…



И Земля снова начала бы вращаться.


Релятивистский бейсбольный мяч

ВОПРОС: А что, если попытаться отбить бейсбольный мяч, брошенный со скоростью в 90 % от скорости света?

– Эллен Макмэнис

Оставим в стороне вопрос о том, как мы заставили мяч лететь так быстро. Предположим, что это был обычный бросок, просто в тот момент, когда подающий игрок (пичтер) бросил мяч, тот загадочным образом ускорился до 0,9 с. Дальше все происходит по обычным законам физики.


ОТВЕТ: Похоже, ответ будет таким: случится очень многое, случится очень быстро, и ничего хорошего ни для отбивающего игрока (баттера), ни для питчера из этого не выйдет.

Мяч будет лететь так быстро, что мир вокруг него станет практически неподвижным. Даже молекулы воздуха фактически замрут. Они будут вибрировать со скоростью несколько сотен км/ч, но мяч будет лететь сквозь них со скоростью чуть меньше 1 млрд км/ч, так что по сравнению с мячом молекулы можно считать неподвижно подвешенными.

Принципы аэродинамики здесь неприменимы. В обычных условиях воздух обтекает предметы, летящие через него, но у молекул воздуха, которые окажутся перед нашим мячом, просто не будет времени отлететь в сторону. Мяч врежется в них с такой силой, что между атомами молекул воздуха и атомами поверхности мяча начнется настоящая реакция ядерного синтеза. Каждое столкновение молекул будет приводить к выбросу гамма-излучения и рассеянных при столкновении частиц[3].



Эти гамма-лучи и частицы будут разлетаться, образуя раздувающийся пузырь, центр которого будет находиться в точке, где стоял питчер. Они начнут разрушать молекулы воздуха, выбивая из ядер электроны и превращая воздух на стадионе в расширяющуюся сферу раскаленной плазмы. Стенка этого раздувающегося пузыря будет со скоростью света приближаться к баттеру, лишь немного обгоняя сам мяч.

Постоянный ядерный синтез, происходящий перед мячом, будет оказывать на него давление, замедляя его, как если бы мяч был ракетой, которая летит хвостом вперед, включив двигатели. К сожалению, мяч будет двигаться так быстро, что даже невероятная сила термоядерного взрыва замедлит его совсем чуть-чуть. Однако эта сила начнет постепенно уничтожать поверхность мяча, и мельчайшие ее фрагменты будут разлетаться во все стороны. Они полетят так быстро, что при столкновении с молекулами воздуха запустят еще два-три раунда ядерного синтеза.

Спустя примерно 70 наносекунд мяч прилетит к базе. Баттер не успеет даже увидеть, как питчер бросил его, так как свет, несущий эту информацию, достигнет баттера примерно в тот же момент, что и мяч. Столкновения молекул в воздухе к этому моменту практически полностью уничтожат мяч, и он будет представлять собой несущееся как пуля облако расширяющейся плазмы (в основном состоящей из углерода, кислорода, водорода и азота), которое будет все так же врезаться в молекулы воздуха и запускать все больше реакций синтеза. Сначала до баттера доберется оболочка пузыря, состоящая из рентгеновских лучей, а спустя несколько наносекунд на него обрушится облако из осколков мяча.



Центр этого облака к тому моменту, когда оно достигнет базы, все еще будет перемещаться со скоростью, составляющей значительную часть скорости света. Когда центр облака столкнется с битой баттера, то и он, и база будут отброшены волной и пробьют ограждение поля, одновременно расщепляясь на молекулы. Оболочка из гамма-лучей и раскаленной плазмы будет расширяться в стороны и вверх, поглотит бейсбольное поле, обе команды, зрителей, окружающие кварталы, и все это в первую же микросекунду.

Представьте, что вы наблюдаете за этим с холма, расположенного вне города. Первое, что вы увидите, – ослепляющий свет, гораздо более яркий, чем солнце. Он постепенно тускнеет в течение нескольких секунд, и растущий огненный шар превращается в грибовидное облако. Затем раздается нарастающий грохот и приходит взрывная волна, ломающая деревья и сравнивающая с землей дома.

Все в радиусе примерно полутора километров от парка будет снесено до основания, и огненная буря поглотит окружающий город. Бейсбольная площадка превратится в довольно приличного размера кратер, центр которого будет в сотне метров за тем местом, где еще недавно стоял баттер.


Правило Бейсбольной лиги 6.08 (b) гласит, что в данной ситуации баттер явно «получил удар при подаче», а значит, может продвинуться на первую базу.

Купание в ядерном бассейне

ВОПРОС: А что, если искупаться в бассейне для отработавшего ядерного топлива?

– Джонатан Бастьен-Фильятро

ОТВЕТ: Если вы хорошо плаваете, то вы, вероятно, сможете продержаться на поверхности 10–40 часов. После этого вы потеряете сознание от изнеможения и утонете. То же самое, кстати, верно и для обычного бассейна, на дне которого не хранится ядерное топливо.

Отработавшее ядерное топливо из реактора крайне радиоактивно. Но вода очень хорошо изолирует и охлаждает топливо, и поэтому его можно хранить на дне в течение нескольких десятилетий, пока его активность не снизится до такой степени, что топливо можно будет переместить в сухие контейнеры. Человечество пока не придумало, куда девать эти контейнеры потом, и в течение ближайших десятилетий нам предстоит решить эту проблему.


Вот разрез типичного бассейна для отработавшего топлива:


Температура не будет серьезной проблемой. Теоретически вода в подобном бассейне может разогреться до 50°С, но на практике температура обычно составляет 25–35°С. Это теплее, чем в обычном бассейне, но прохладнее, чем горячая ванна.

Наиболее радиоактивны стержни, лишь недавно извлеченные из реактора. В случае с видами радиации, излучаемыми отработавшим ядерным топливом, каждые 7 см воды уменьшают степень излучения вдвое. Согласно данным компаний, управляющих атомными электростанциями, зоны поражения для «свежих» стержней будут такими:



Если нырнуть на самое дно, коснуться «свежего» стержня локтем и немедленно вынырнуть на поверхность, то этого, вероятно, будет достаточно для того, чтобы умереть от излучения.

Однако за пределами опасной зоны можно плавать сколько угодно – доза излучения, которую вы получите, будет меньше, чем обычное фоновое излучение в повседневной жизни. Вода защищает вас от большей части этой фоновой дозы, так что, плавая в бассейне для отработавшего ядерного топлива, вы получите меньшую дозу радиации, чем просто прогуливаясь по улице.

Но все это верно лишь при условии, что с самим бассейном все в порядке. Если же стенка контейнера, содержащего стержень, будет повреждена коррозией, то в воде могут оказаться продукты деления ядра. Воду в подобных бассейнах довольно эффективно очищают, и плавать в ней не опасно, но при этом она достаточно радиоактивна, чтобы ее нельзя было разливать в бутылки и продавать[4].


Напоминаю: я просто рисую комиксы! Если вы, стоя на краю радиоактивного бассейна, принимаете мои слова за советы по безопасности, то вы, вероятно, заслуживаете всего того, что с вами вот-вот случится.


Мы точно знаем, что в бассейнах для отработавшего топлива можно плавать, потому что их регулярно обслуживают ныряльщики. Однако этим ныряльщикам нужно соблюдать осторожность.

21 августа 2010 года ныряльщик работал в бассейне Лейбштадского ядерного реактора в Швейцарии. Он увидел на дне бассейна кусок какого-то шланга и связался с диспетчером, чтобы узнать, что делать. Ему велели поднять шланг и положить его в сумку с инструментами, что ныряльщик и сделал. Из-за бульканья воды в бассейне он не услышал, как сработал его датчик радиации.

Когда сумку с инструментами вытащили из воды, заработали все датчики радиации в здании. Сумку тут же бросили обратно в воду, а ныряльщик быстро вылез из бассейна. Дозиметры показали, что он получил высокую дозу радиации, причем особенно высоким было облучение его правой руки.

«Шланг» оказался фрагментом защитной оболочки радиационного монитора в ядре реактора, крайне радиоактивным за счет нейтронного потока. Этот кусок случайно отрезали от обшивки в 2006 году, когда закрывали контейнер, и с тех пор он валялся на дне в дальнем углу бассейна, где его никто не замечал в течение четырех лет.

Этот предмет был настолько радиоактивен, что если бы ныряльщик засунул его за пояс или в положил в рюкзак, где «шланг» оказался бы близко к телу, доза оказалась бы смертельной. К счастью, в данной ситуации рабочего защитила вода, и только рука – часть тела, куда более успешно противостоящая радиации, чем уязвимые внутренние органы, – получила серьезную дозу облучения.



Короче говоря, с вами, скорее всего, все будет в порядке, если вы не будете нырять на дно и подбирать там что попало.

Но чтобы быть окончательно в этом уверенным, я связался с моим приятелем, который работает на исследовательском реакторе, и спросил его: что, по его мнению, произойдет с человеком, который захочет поплавать в их бассейне для хранения топлива?

«В нашем бассейне? – он на секунду задумался. – Умрет еще до того, как доберется до воды: его пристрелит охрана».

Странные (и тревожные) вопросы из папки «Входящие» сайта «А что, если?»

ВОПРОС: Можно ли охладить зубы настолько, чтобы они треснули при попытке выпить горячий кофе?

– Шелби Хеберт

ВОПРОС: Сколько домов сгорает в США каждый год? И как проще всего значительно увеличить эту цифру (хотя бы на 15 %)?

– Аноним

Машина времени по-нью-йоркски

ВОПРОС: Когда путешествуешь во времени, всегда оказываешься в одной и той же точке пространства. Во всяком случае, в фильме «Назад в будущее» дело обстоит именно так. А что, если, стоя на Таймс-сквер, отправиться на 1000 лет назад? А на 100 000? А на миллиард лет? А что, если отправиться на миллион лет вперед?

– Марк Деттлинг
Тысячу лет назад

Манхэттен постоянно населен последние 2000 лет, а первые люди появились здесь приблизительно 9000 лет назад.

В XVII веке, когда прибыли европейцы, в этих краях жили индейцы ленапе[5].

Тысячу лет назад это место, вероятно, тоже населяла некая группа племен, но они жили за пять сотен лет до контакта с европейцами и были так же далеки от ленапе XVII века, как индеец XVII века от современного американца.

Чтобы увидеть, как выглядела Таймс-сквер до того, как появился город, стоит обратиться к замечательному проекту под названием Welikia, выросшему из маленького проекта Mannahatta. Разработчики создали подробную экологическую карту нью-йоркского ландшафта времен прибытия первых европейцев.

Интерактивная карта, доступная на сайте welikia.org, представляет собой воображаемый спутниковый снимок «другого» Нью-Йорка. В 1609 году остров Манхэттен был покрыт холмами, болотами и рощами, озерами и реками.

Район Таймс-сквер тысячелетней давности выглядел, вероятно, примерно так, как это показывает Welikia. Если не вдаваться в подробности, местный пейзаж, скорее всего, напоминал девственные леса, которые все еще можно увидеть на северо-востоке США. Однако были бы и серьезные отличия.

Тысячу лет назад на Манхэттене водились довольно крупные животные. В тех разрозненных остатках девственных лесов, что мы видим сегодня, практически нет крупных хищников, разве что несколько медведей, да немного волков и койотов, а вот пумы практически отсутствуют (зато популяции оленей, напротив, разрослись, благодаря в том числе и исчезновению крупных хищников).

В лесах Манхэттена тысячелетней давности в изобилии росли каштаны. До эпидемии паразитического гриба в начале XX века леса на востоке США на 25 % состояли из каштанов, от которых сегодня остались только пни.



Эти пни все еще можно увидеть в лесах Новой Англии. Периодически они дают новые побеги, которые, однако, быстро увядают под действием гриба. Однажды (и это будет довольно скоро) исчезнут последние из этих пней.

А в лесах тысячелетней давности вам встретятся волки, особенно когда вы будете продвигаться вглубь материка. Еще вы увидите пум[6] и странствующих голубей[7].

А вот чего вы точно не увидите, так это дождевых червей. Их не было в Новой Англии до появления европейских колонистов. Чтобы узнать, почему, давайте отправимся еще глубже в прошлое.

10 000 лет назад

Земля 10 000 лет назад только-только выбралась из очень холодного периода. Ледники, покрывавшие Новую Англию, уже исчезли. 22 000 лет назад южная граница льда проходила у Статен-Айленда[8], но уже через четыре тысячи лет назад отступила на север за Йонкерс[9]. К моменту нашего прибытия (еще спустя восемь тысяч лет) ледник по большей части отодвинулся к северу за пределы современной канадской границы.

Ледники сгладили ландшафт, обнажив скальные породы. В течение 10 000 лет жизнь медленно возвращалась вслед за ледником обратно на север. Некоторые виды перемещаются быстрее, чем другие, и когда европейцы пришли в Новую Англию, дождевые черви еще не успели вернуться сюда.

Ледники отступал, но гигантские ледяные горы откалывались от него и начинали медленно таять.



На месте этих ледяных гор остались заполненные водой впадины, которые называют ледниковыми озерами. Озеро Окленд-лейк в нью-йоркском округе Квинс одно из таких озер. А еще ледники тащили с собой валуны, и некоторые из этих ледниковых (эрратических) валунов можно и сегодня увидеть в Центральном парке.



Подо льдом, под большим давлением, текли реки талой воды, принося с собой песок и гравий. Эти отложения в виде земляных валов (так называемые эскеры) исчерчивают леса вокруг моего дома в Бостоне. На их счету немало странных образований в ландшафте, включая русла рек, имеющие в сечении форму буквы U.


100 000 лет назад

Пейзаж 100 000 лет назад выглядел, вероятно, во многом так же, как сегодня[10]. Мы живем в эпоху коротких, быстро сменяющих друг друга похолоданий и потеплений, но в течение последних 10 000 лет наш климат был стабильным[11] и теплым.

Сто тысяч лет назад Земля приближалась к концу аналогичного периода климатической стабильности. Уже закончилось так называемое Эемское межледниковье, и, скорее всего, в этот период флора и фауна региона показались бы нам весьма знакомыми.

А вот топография прибрежной зоны была бы совершенно иной: острова Статен-Айленд, Лонг-Айленд, Нантакет и Мартас-Винъярд в то время были полуостровами, которые окончательно отделит от материка лишь более поздний ледник, который пройдет по этому краю, словно бульдозер. Сотню тысяч лет назад прибрежные воды были испещрены совсем другими островами.

В лесах того времени водились многие современные животные, например птицы, белки, олени, волки, черные медведи, но встречались и удивительные исключения. Чтобы узнать о них, обратимся к тайне вилорога.

Современный вилорог – это загадка природы. Он быстро бегает, гораздо быстрее, чем необходимо. Он разгоняется до 88 км/час и способен несколько минут бежать с этой скоростью, покрыв за это время 5–6 километров. Однако его основные враги, волки и койоты, едва достигают скорости в 56 км/ч. Для чего же вилорог такой быстрый?

Ответ заключается в том, что мир, в котором вилорог эволюционировал, был куда опаснее современного. Сотню тысяч лет назад североамериканские леса были ареалом обитания так называемого волка ужасного (Canis dirus), малого короткомордого медведя (Arctodus pristinus) и смилодона (Smilodon fatalis). Все эти звери были, по всей видимости, быстрее и опаснее современных хищников. Все они исчезли во время плейстоцен-голоценового вымирания, которое произошло вскоре после того, как на континенте появились первые люди[12].

Если же мы углубимся еще дальше в прошлое, то встретим еще одного устрашающего хищника.

1 000 000 лет назад

Миллион лет назад, до наступления последнего крупного ледника, в мире было довольно тепло. Прошла половина четвертичного периода, великие современные ледниковые эпохи начались за несколько миллионов лет до того, но наступление и отступление ледников в этот момент замедлилось, а климат был довольно стабильным.

Хищникам, которых мы встретили ранее, быстроногим созданиям, способным охотиться на вилорога, составлял компанию еще один безжалостный убийца – длинноногая гиена, напоминавшая современного волка. Гиены в основном водились в Азии и Африке, но во время снижения уровня моря один вид гиен пересек Берингов пролив и попал в Северную Америку. За это достижение этот вид гиены назвали Chasmaporthetes, что значит «тот, кто видел каньон».

А затем вопрос Марка отправляет нас в еще более далекое прошлое.

1 000 000 000 лет наза д

Миллиард лет назад континентальные плиты были собраны в один огромный суперконтинент. Это была не хорошо известная Пангея, но ее предшественник Родиния. Геологические данные не отличаются особой точностью, но мы предполагаем, что выглядела она примерно вот так:



Во времена Родинии скальные породы, подстилающие Манхэттен, еще не сформировались, но часть континента, из которой впоследствии образовалась Северная Америка, уже была древней. Сегодняшний Манхэттен, вероятно, входил в состав большого куска суши, из других частей которого образовались территории Анголы и Южно-Африканской Республики.

В этом первобытном мире не было ни растений, ни животных. Жизнь наполняла океаны, но это была простая, одноклеточная жизнь: на поверхности воды плавали ковры из сине-зеленых водорослей. Эти скромные существа – самые смертоносные убийцы за всю историю жизни на Земле.

Сине-зеленые водоросли, или цианобактерии, были первыми фотосинтезирующими организмами. Они вдыхали углекислый газ и выдыхали кислород. Кислород весьма активное вещество, он заставляет железо ржаветь (окисление), дерево гореть (активное окисление). Когда цианобактерии только появились, кислород, который они выдыхали, был токсичен почти для всех остальных форм жизни. Вымирание видов, произошедшее в результате появления цианобактерий, называют кислородной катастрофой.

Но после того как цианобактерии наполнили атмосферу Земли и ее воду токсичным кислородом, развились создания, которые воспользовались активной природой этого газа, чтобы запустить новые биологические процессы. Мы наследники тех первых существ, которые научились дышать кислородом.

Многие подробности этой истории остаются неясными – непросто реконструировать мир, каким он был миллиард лет назад. Но сейчас вопрос Марка отправит нас в еще более неясное – время в будущее.

Миллион лет спустя

Когда-нибудь человечество вымрет. Никто не знает, когда именно[13], но ничто не живет вечно. Быть может, мы улетим к звездам и проживем миллиарды лет. А может быть, наша цивилизация рухнет, а мы все погибнем от болезней и голода, и последних из нас съедят бездомные кошки. Или нас всех убьют нанороботы, захватившие планету спустя всего несколько десятков лет после того, как вы прочли это предложение. Точно сказать нельзя.

Миллион лет – это много. Это в несколько раз дольше, чем существует Homo sapiens, и более чем в сто раз дольше, чем существует письменность. Разумно предположить, что, как бы ни разворачивалась история человечества, через миллион лет ее нынешняя фаза закончится.

Но геологические силы Земли продолжат свою работу и без нас. Ветра, дожди и песчаные бури уничтожат и похоронят артефакты нашей цивилизации. Изменения климата, вызванные человеческой деятельностью, возможно, отсрочат начало нового ледникового периода, но сам цикл этих периодов мы не прервали, и однажды ледники придут снова. Мало что останется от человечества спустя миллион лет.

Скорее всего, самой живучей реликвией нашего времени будет слой пластика, которым мы покрыли всю планету. Добывая нефть, превращая ее в прочные, надолго сохраняющиеся полимеры и разбрасывая их по поверхности Земли, мы оставили след, который может пережить все прочие наши достижения.

Пластик раскрошится, будет захоронен в почве, и, возможно, какие-нибудь микробы научатся его перерабатывать, но весьма вероятно, что и спустя миллион лет толстый слой обработанных углеводородов – фрагменты наших бутылок из-под шампуня и пластиковых пакетов – будет служить химическим памятником цивилизации.

Еще более отдаленное будущее

Солнце становится все ярче. В течение трех миллиардов лет сложная система обратных связей поддерживала температуру Земли относительно стабильной, пока Солнце понемногу теплело.

Но через миллиард лет эта система даст сбой. Наши океаны, которые вскармливали и охлаждали жизнь, превратятся в ее злейшего врага. Они закипят на горячем солнце, окружат планету толстым слоем водяного пара и усилят парниковый эффект. Через миллиард лет Земля станет второй Венерой.

Нагреваясь еще больше, планета может полностью лишиться воды, и ее атмосфера наполнится каменным паром, поскольку начнет кипеть и испаряться и поверхность суши. Спустя еще несколько миллиардов лет растущее Солнце в конце концов поглотит Землю.

Земля исчезнет, и множество молекул, из которых состояла площадь Таймс-сквер, разлетятся прочь от умирающего Солнца. Эти пылевые облака будут плыть через космос, возможно, рождая по пути новые звезды или планеты.

Если люди к тому времени выберутся за пределы Солнечной системы и переживут Солнце, не исключено, что наши потомки будут жить на одной из этих планет. Атомы Таймс-сквер, пройдя сквозь горнило Солнца, сформируют наши новые тела.



И в один прекрасный день мы все либо вымрем, либо станем ньюйоркцами до мозга костей.

Где моя вторая половинка?

ВОПРОС: А что, если бы у каждого человека на самом деле была лишь одна-единственная «вторая половинка» – некий человек, живущий неизвестно где?

– Бенджамин Стаффин

ОТВЕТ: Это был бы сущий кошмар.

С концепцией единственной второй половинки вообще много проблем. Как поет австралийский комик Тим Минчин,

Твоя любовь одна на миллион,
Ее не купишь ни за какую цену,
Но статистика говорит,
Что из оставшихся 999 000
Некоторые точно будут не хуже.

Так что, если бы у нас действительно имелась одна-единственная вторая половинка и больше мы ни с кем в мире не были бы счастливы? Удалось бы нам встретить свою половинку?

Предположим, что наша половинка предопределена при рождении. Вы ничего не знаете о том, кто этот человек и где он живет, но любовные романы учат нас, что вы узнаете друг друга, как только встретитесь взглядом.

Сразу возникают некоторые вопросы. Во-первых, жива ли еще ваша вторая половинка? За всю историю человечества на Земле жило около сотни миллиардов людей, но сегодня нас только семь миллиардов (таким образом, человеческая жизнь до сих пор приводила к смерти в 93 % случаев). Иными словами 93 % вторых половинок уже нет в живых.



Это ужасно! Но погодите, дальше будет только хуже. Простая логика подсказывает, что нельзя ограничиваться только людьми, жившими в прошлом: надо учитывать и неизвестное число людей будущего. Если ваша вторая половинка может жить в прошлом, значит, некоторые половинки могут находиться и в будущем, ведь и вы сами такая «будущая» половинка для кого-то в прошлом.

Теперь давайте предположим, что ваша половинка живет в то же время, что и вы. Предположим также, что вы сверстники в пределах нескольких лет (это более строгое ограничение, чем известное правило «половина вашего возраста плюс семь лет»[14], однако это правило работает, только если половинки встречаются уже взрослыми – скажем, одному 30, а другому 40 лет, и не работает, если бы они встретились лет на 15 лет раньше). С учетом этих ограничений у каждого из нас окажется примерно полмиллиарда потенциальных половинок.

Но как насчет пола и ориентации? Культуры? Языка? Мы можем и дальше использовать демографические категории, чтобы еще более сузить круг потенциальных партнеров, однако при этом мы уходим от идеи случайной половинки. В нашем сценарии вы с вашей половинкой ничего не знаете друг о друге, пока не посмотрите друг другу в глаза. Можно сказать, что ваша ориентация будет определяться исключительно вашей половинкой.

Как видим, шансы столкнуться со своей половинкой весьма малы. Количество незнакомцев, которым мы смотрим в глаза каждый день, варьируется от нуля (если вы отшельник или житель маленького городка) до нескольких тысяч (если вы полицейский на Таймс-сквер), но давайте предположим, что вы встречаетесь взглядом с несколькими десятками незнакомцев в день (я скорее интроверт, поэтому для меня это довольно щедрое предположение). Если 10 % из них примерно ваши сверстники, то всего за всю жизнь вы поймаете взгляд примерно 50 000 людей. С учетом того, что у каждого из нас есть 500 000 000 потенциальных половинок, истинную любовь в течение своей жизни обретет лишь один из 10 000…




Если опасность умереть в одиночестве столь велика, то общество просто обязано придумать способ увеличить вероятность визуального контакта. Например, построить огромные транспортные ленты, чтобы граждане могли перемещаться, глядя в глаза визави… А если того же эффекта можно достичь с помощью веб-камеры, то надо разработать модифицированную версию сайтов для случайных знакомств – таких, например, как Chat Roulette.com.



Если бы каждый из нас использовал эту систему по восемь часов в день семь дней в неделю и если бы нам требовалась бы всего пара секунд, чтобы определить, является ли человек нашей половинкой, то система могла бы теоретически свести всех людей со своими половинками за несколько десятилетий. (Я произвел несколько несложных вычислений, чтобы в целом оценить шансы участников. Если вы хотите рассчитать ваш конкретный случай с помощью математических инструментов, можете начать с задач на перестановку.)

Но в реальной жизни многим людям и без того непросто найти хоть какое-то время для личной жизни, и мало кто смог бы посвятить ее устройству два десятилетия. Так что не исключено, что только детки богатых родителей смогли бы часами сидеть на нашем сайте (назовем его «рулетка-для-половинок-точка-ком»). Но, увы, к несчастью для легендарного «золотого процента» (считается, что богачи составляют 1 % мирового населения), большая часть их половинок входила бы в оставшиеся 99 %… И если лишь один процент от «золотого процента» воспользуется нашей рулеткой, то лишь один процент от процента от процента найдет свою пару – то есть один из 10 000 человек.

Зато у оставшихся 9 999 богачей появился бы стимул втянуть в эту систему больше людей. В результате могли бы возникнуть благотворительные проекты, направленные, например, на то, чтобы у каждого жителя Земли оказался компьютер – нечто среднее между кампанией «Ноутбук каждому ребенку» и усовершенствованным сайтом знакомств. Профессии кассира в супермаркете или патрульного на Таймс-сквер стали бы невероятно престижными, поскольку дают возможность часто встречаться глазами с другими. Люди рвались бы в большие города и на разнообразные тусовки, чтобы найти свою любовь, точно так же, как они это делают и сейчас.

Но даже если бы кто-то из нас провел годы на сайте «рулетка-для-половинок-точка-ком», кто-то нашел работу, где постоянно заглядывал в глаза незнакомцам, а кто-то просто надеялся на удачу, – лишь малая часть всех нас нашла бы свою любовь. Остальным бы не повезло.

В условиях постоянного стресса и давления новой общественной нормы кто-то начал бы притворяться. Всем хочется вступить в клуб счастливых, и двое одиночек могли бы объединиться, чтобы разыграть мнимую встречу двух половинок. Поженившись, они продолжали бы скрывать свою выдумку и старались изобразить счастливейшую пару на глазах у своих друзей и своей семьи.

Мир случайных половинок был бы очень одиноким местом. Давайте надеяться, что наш мир не таков.

Лазерная указка

ВОПРОС: А что, если все люди на Земле одновременно направят на Луну лазерные указки? Изменит ли она свой цвет?

– Питер Липовиц

ОТВЕТ: Нет, если речь идет об обычной указке.

Во-первых, следует учесть, что не все мы видим Луну одновременно. Поскольку примерно 75 % населения Земли живет между нулевым и 120-м меридианом, идеальное положение Луны для нашего эксперимента где-то над Аравийским морем.

Какую луну выбрать – новую или полную? С одной стороны, луна в новолуние гораздо темнее, и свет наших лазеров было бы проще заметить. Но с другой стороны, такая луна – сложная мишень, поскольку видно ее в основном днем, а значит, результат наших усилий практически не будет виден.

Давайте лучше возьмем фазу четверти луны, чтобы можно было сравнивать эффект наших лазеров на темной и светлой стороне.


Это наша мишень



Обычная красная лазерная указка имеет мощность примерно в 5 милливатт, так что ее свет вполне сможет достичь Луны, хотя и рассеется по достаточно большому участку лунной поверхности. Земная атмосфера немного исказит луч и частично поглотит его, но большая часть света все же достигнет цели.

Давайте предположим, что все мы достаточно меткие, чтобы вообще попасть в Луну, но не все попадем в одно и то же место, так свет распределится по поверхности равномерно.

В 00:30 по Гринвичу все прицеливаются и нажимают кнопку!

Вот что произойдет.



Что ж, довольно обидно… Но такого результата и следовало ожидать. Солнце освещает Луну с мощностью, превышающей киловатт на квадратный метр. Поскольку площадь экваториального сечения Луны составляет около 10?? м?, ее омывает примерно 1016 ватт солнечного света то есть 10 петаватт, или 2 мегаватта на каждого жителя Земли, что намного превышает мощность наших 5-милливаттных лазерных указок. В каждой части этих расчетов есть некоторые неточности, но в целом соотношение именно таково.



Лазер мощностью в 1 Вт – очень опасная вещь. Он не просто способен ослепить вас, он может обжечь кожу и даже поджечь окружающие предметы. Совершенно логично, что его нет в свободной продаже в США… Шучу-шучу! Есть, конечно, и стоит он всего 300 долларов.

Итак, предположим, мы потратили два миллиарда долларов, чтобы купить каждому жителю Земли зеленый лазер мощностью 1 Вт. (Примечание для кандидатов в президенты – подобный пункт в Вашей программе помог бы Вам получить мой голос.) Этот лазер не просто более мощный, чем лазерная указка, – зеленый цвет находится ближе к середине видимого спектра, поэтому глаза воспринимают его лучше, и он кажется более ярким.

Вот какой будет эффект.



Упс!.. Наши лазерные указки посылают 150 люменов света (больше, чем фонарики), и ширина их луча составляет 5 угловых минут. Такой луч осветит поверхность Луны лишь примерно на  1/2  люкса (лк) по сравнению с освещенностью в 130 000 лк, которую обеспечивает нашему спутнику Солнце. Даже если все мы прицелимся с идеальной точностью, это даст нам дополнительно всего лишь около 5 лк на примерно 10 % поверхности Луны.



Для сравнения – полная Луна освещает поверхность Земли примерно на 1 лк. Это значит, что действия наших лазеров не видно не только с Земли: даже стоящему на Луне астронавту отсвет земных указок на лунной поверхности показался бы более слабым, чем лунный свет на Земле.

В последнее десятилетие с развитием литиевых батарей и светодиодов появилось очень много самых разнообразных фонарей, но очевидно, что карманный фонарик нам в любом случае не поможет. Так что пропустим все это и выдадим каждому участнику проекта устройство Nightsun.

Название может показаться вам незнакомым, но есть шанс, что вы видели это устройство в действии: так называются прожекторы, которые установлены на вертолетах полиции и береговой охраны. Испуская 50 000 люменов света, они вполне способны превратить ночь в день.

Луч такого устройства имеет угловую ширину несколько градусов, так что нам понадобятся фокусирующие линзы, чтобы сузить его до половины градуса, необходимой, чтобы попасть в Луну.

Вот полученный эффект.



Его почти не видно, но все же прогресс есть! Луч лазера дает освещенность в 20 лк, то есть он ярче, чем темная половина лунного диска. Однако разглядеть это очень сложно, и мы точно не увидим никаких следов луча на светлой половине Луны.



Давайте заменим каждый Nightsun на проектор IMAX 30 000-ваттную пару ламп с системой водяного охлаждения, которые вместе выдают более миллиона люменов.



И все равно едва заметно… На крыше отеля «Люксор» в Лас-Вегасе стоит самый мощный проектор на Земле. Давайте выдадим такой же каждому участнику нашего эксперимента.

Ах да, и добавим линзы, чтобы весь луч фокусировался именно на Луне.



Наш свет теперь точно виден, так что мы достигли своей цели. Поздравляю команду.



Кхм… Вообще-то министерство обороны США в свое время проектировало мегаваттные лазеры для уничтожения боеголовок в полете. Таким был Boeing YAL-1–химический кислородно-йодный мегаваттный лазер, который устанавливали на самолете Boeing-747. Это инфракрасный лазер, так что мы не увидим его луча, но давайте вообразим, что существует лазер аналогичной мощности, излучающий свет в рамках видимого спектра.



Наконец-то нам удалось сравняться с яркостью солнечного света! Правда, мы потратили на это пять петаватт энергии, что вдвое больше, чем в среднем тратит человечество за то же самое время.



Ладно, давайте установим по мегаваттному лазеру на каждом квадратном метре территории Азии. Правда, ради поддержания работы 50 миллиардов лазеров придется израсходовать все запасы нефти на Земле примерно за две минуты, зато в течение этих двух минут Луна будет выглядеть так:



Луна будет казаться такой же яркой, как полуденное солнце, и к концу этих двух минут лунная поверхность разогреется настолько, что начнет светиться сама.



Что ж, сделаем еще один уверенный шаг за пределы правдоподобия. Самый мощный лазер на земле находится в National Ignition Facility – лаборатории по исследованию управляемой термоядерной реакции. Это ультрафиолетовый лазер с мощностью 500 тераватт. Он испускает отдельные импульсы продолжительностью по несколько наносекунд, так что энергия одного импульса будет равна энергии, которую можно получить при сжигании примерно четверти стакана бензина.

Представим, что мы каким-то образом нашли способ заставить этот лазер постоянно работать, выдали каждому жителю Земли по экземпляру устройства и одновременно направили бы все эти устройства на Луну.

К сожалению, такой поток энергии обратит атмосферу на пути луча в плазму, которая немедленно подожжет поверхность Земли и убьет нас всех. Но давайте предположим, что этот лазер как-то умудряется проходить через атмосферу Земли, не взаимодействуя с ней.

Но даже при этих условиях, как выясняется, Земля все равно загорится. Свет, отраженный от Луны, будет в 4000 раз ярче света полуденного солнца. В таком сиянии океаны Земли выкипят меньше чем за год.

Но забудем на минуту о Земле. А что же будет с Луной?

Давление света ускорит вращение Луны примерно на одну десятимиллионную часть ее гравитационной силы. Это ускорение будет не слишком заметно в краткосрочной перспективе, но со временем его хватит, чтобы Луна переместилась на более высокую орбиту вокруг Земли…

…Но если бы давление света было бы единственным фактором, который повлияет на Луну!

Сорока мегаджоулей энергии достаточно, чтобы испарить килограмм скальной породы. Если предположить, что лунная порода имеет среднюю плотность 3 кг/л, то лазеры дадут достаточно энергии, чтобы поверхность нашего спутника начала испаряться и на ней образовался бы кратер, глубина которого будет увеличиваться со скоростью четыре метра в секунду:



Однако в реальности лунная порода будет испаряться не так быстро по очень важной причине: испарившийся камень не исчезает в небытие. Поверхность Луны превратится в плазму, и эта плазма перекроет путь лучу. Наш лазер будет вливать все больше энергии в плазму, нагревая ее все больше и больше. Частицы плазмы будут сталкиваться, отскакивать друг от друга, врезаться в поверхность Луны и в конце концов с потрясающей скоростью вылетят в космос.

Этот поток вещества по сути превратит всю поверхность Луны в ракетный двигатель, причем на удивление эффективный. Использование лазеров с целью удаления материала с поверхности называется лазерной абляцией, и этот метод весьма перспективен с точки зрения перемещения космических кораблей.

Луна огромна, но каменная плазма начнет медленно и неуклонно отталкивать ее от Земли (этот выброс также отдраит дочиста поверхность Земли и уничтожит, в том числе, и все лазеры, но мы пока притворяемся, что нам ничего не грозит). Одновременно плазма будет уничтожать и лунную поверхность, и это сложное взаимодействие нелегко смоделировать.



Но если предположить, что частицы плазмы отлетают от Луны со скоростью 500 км/с, потребуется несколько месяцев, чтобы Луна вышла за пределы действия наших лазеров. Наш спутник сохранит большую часть своей массы, но выйдет за пределы гравитации Земли и перейдет на асимметричную орбиту вокруг Солнца.

Технически Луна не станет новой планетой, если брать определение Международного астрономического союза. Поскольку ее новая орбита будет пересекаться с земной, она будет считаться карликовой планетой, как Плутон. Пересечение орбит Луны и Земли вызовет периодические непредсказуемые колебания. В конце концов Луна может упасть на Солнце, вылететь за пределы Солнечной системы или врезаться в одну из планет – весьма вероятно, нашу.


Вот что я называю настоящей мощностью!

Периодическая стена элементов

ВОПРОС: А что, если построить Периодическую таблицу Менделеева из кубиков, сделанных из соответствующих элементов?

– Энди Коннолли

ОТВЕТ: Есть люди, которые коллекционируют химические элементы. Они пытаются собрать как можно больше физических образцов и раскладывают их в ящички, составленные в виде таблицы Менделеева.

Из 118 элементов таблицы три десятка – такие как гелий, углерод, алюминий, железо или серу – можно купить в чистом виде в магазинах. Еще несколько десятков можно раздобыть, разломав какой-нибудь прибор (например, образец америция можно найти в детекторе дыма). Еще что-то можно заказать в Интернете.

В общем и целом вам, возможно, удастся собрать примерно 80 элементов или 90, если вы готовы немножко рискнуть своим здоровьем, безопасностью и репутацией законопослушного гражданина. Остальные слишком радиоактивны или недолговечны, чтобы можно было собрать в одном месте за один раз больше нескольких атомов.

А что, если вам это все же удалось бы?


В Периодической таблице Менделеева 7 рядов[15].



Верхние два ряда составить легко.

Третий мог бы сильно обжечь вас.

Четвертый убил бы токсичным дымом.

Пятый сделал бы все то же самое плюс облучил бы вас небольшой дозой радиации.

Шестой с грохотом взорвался бы, превратив все вокруг в облако радиоактивного и ядовитого огня и пыли.

А вот строить седьмой ряд я бы вообще не рекомендовал.


Начнем сверху. Первый ряд простой, хотя и скучный.



Кубик водорода поднялся бы кверху и растаял, как воздушный шарик без оболочки. То же случилось бы с гелием.



Второй ряд уже сложнее.


Кубик лития немедленно потемнел бы на воздухе. Бериллий довольно токсичен, так что с ним надо обращаться осторожно и стараться, чтобы его пыль не попала в воздух.

Кубики кислорода и азота будут медленно таять в воздухе, постепенно исчезая[16]. Неон также уплывет прочь.

Бледно-желтый кубик фтора немедленно стек бы на пол. Надо учесть, что фтор – самый активный окислитель во всей таблице. Почти любое вещество немедленно загорится, соприкоснувшись с ним.

Я спросил специалиста по органической химии Дерека Лоу[17], что он думает по этому поводу. Дерек сказал, что фтор не будет реагировать с неоном и у него установится что-то вроде вооруженного перемирия с хлором, но все остальное?.. Пуфф! Фтор будет создавать проблемы и встречаясь с элементами из нижних рядов в таблице, а вступая в контакт с любой жидкостью, образовывал бы чрезвычайно едкую фторную кислоту.

Если вы вдохнете даже следовое количество фтора, это сильно повредит или полностью уничтожит ваш нос, ваши легкие, рот, глаза и, в конечном итоге, все ваше тело. При работе с фтором вам точно потребовался бы противогаз. Но помните, что фтор разъедает многие материалы, из которых делают противогазы, так что последний лучше сначала протестировать. В общем, удачи и вперед, к третьему ряду!



Больше всего неприятностей в третьем ряду следует ожидать от фосфора. Чистый фосфор может иметь несколько форм. Красный фосфор более или менее безопасен. Белый вспыхивает при контакте с воздухом и горит жарким пламенем, которое трудно погасить. Вдобавок ко всему белый фосфор довольно ядовит[18].

Сера в обычных обстоятельствах не представляет проблемы, разве что неприятно пахнет. Однако тут у нас сера зажата между горящим фосфором (слева) и фтором с хлором (справа). При контакте с газообразным фтором сера, как и многие вещества, начинает гореть.

Инертный аргон тяжелее воздуха, так что он просто растекся бы по земле. Но не будем волноваться по поводу аргона, у нас сейчас есть проблемы посерьезнее.

Горение приведет к появлению разнообразных кошмарных химических соединений с названиями наподобие гексафторид серы. Если вы строите нашу стену в замкнутом помещении, то, скорее всего, уже задохнулись от ядовитого дыма, а ваш дом, возможно, сгорел до основания.

И это всего лишь третий ряд. Так вперед же, к четвертому!



Мышьяк – это звучит страшно. И страх этот вполне обоснован. Мышьяк токсичен практически для всех сложных форм жизни. Иногда подобная паника по поводу химических веществ со страшными названиями не обоснована: в нашей еде и воде присутствуют следовые количества мышьяка, и мы с ними отлично справляемся. Но сейчас не тот случай.

Горящий фосфор (к которому теперь присоединился горящий калий, который также склонен к спонтанному самовозгоранию) может поджечь мышьяк, высвободив большое количество триоксида мышьяка. Это довольно ядовитая штука. Не советую ее вдыхать.

Весь этот ряд тоже неважно пахнет. Селен и бром будут яростно вступать в реакции, и Лоу сказал мне, что по сравнению с запахом горящего селена запах серы – «это как духи от Шанель».

Если алюминий переживет этот пожар, с ним произойдет странная вещь. Плавящийся на один ряд ниже галлий потечет на алюминий, нарушив его структуру и сделав его непрочным и мягким, как мокрая бумага[19].



Горящая сера прольется на бром. При комнатной температуре этот элемент представляет собой жидкость, и это его свойство разделяет только еще одно простое вещество – ртуть. И то, и другое – довольно противные штуки. Разброс токсических веществ, возникших к этому моменту в результате горения, уже неисчислимо велик. Однако если вы наблюдаете опыт с безопасного расстояния, у вас есть шансы выжить.

В пятом ряду есть кое-что интересное – технеций-99, наш первый радиоактивный кирпичик.

Технеций – самый легкий элемент из тех, что не имеют стабильных изотопов, и он практически не встречается в природе. Его название и говорит о том, что он был получен искусственно. Доза радиации, которую излучает куб из технеция объемом в один литр, не будет смертельной, если просто вставить его в нашу периодическую стену, но все же она весьма значительна. Если вы проведете весь день, надев на голову полый куб из технеция или вдыхая технециевую пыль, этот элемент вполне может вас убить.

Если не считать технеция, пятый ряд будет во многом похож на четвертый.



Вперед, к шестому ряду! Как бы осторожны вы ни были до сих пор, шестой ряд точно вас убьет.


Этот вариант Периодической таблицы несколько больше, чем тот, к которому вы, вероятно, привыкли, так как мы добавили лантаноиды и актиноиды в 6 и 7-й ряды. (Обычно эти элементы показывают отдельно от общей таблицы, чтобы не делать ее слишком широкой.)


Шестой ряд Периодической таблицы содержит несколько радиоактивных элементов, включая прометий, полоний, астат и радон. Астат – самый проблемный элемент этого ряда. Мы даже не знаем, как он выглядит, поскольку, по словам Лоу, «эта штука просто отказывается существовать». Астат настолько радиоактивен (его период полураспада измеряется часами), что любой крупный кусок астата быстро испарился бы от производимого им самим жара. Химики подозревают, что у этого куска была бы черная поверхность, но на самом деле этого никто не знает.

Для работы с астатом не существует инструкций по безопасности. Но если бы они существовали, там было бы снова и снова запекшейся кровью нацарапано одно только слово «НЕТ!»

Наш куб недолгое время содержал бы больше астата, чем было синтезировано за всю историю химии. Я говорю «недолго», потому что он немедленно превратился бы в столб раскаленного газа. От одного только жара все находящиеся рядом получили бы ожоги третьей степени, а здание, в котором вы все это проделываете, было бы полностью уничтожено. Облако горячего газа быстро поднялось бы в небо, излучая жар и радиацию.

Сила взрыва была бы как раз такой, чтобы привлечь к вашей лаборатории внимание максимального количества проверяющих. Будь взрыв чуть слабее, вам бы, возможно, удалось его скрыть. Будь он сильнее – и в городе не осталось бы ни одного чиновника, которому можно было бы сдать заполненные документы.

Пыль и обломки, покрытые астатом, полонием и другими радиоактивными элементами, посыпались бы из ядерного облака, сделав окружающие кварталы абсолютно непригодными для обитания.

Уровень радиации был бы крайне высоким. Как известно, для того чтобы один раз моргнуть, требуется несколько сотен миллисекунд, поэтому вы получили бы летальную дозу радиации, в буквальном смысле не успев моргнуть глазом.

Такую причину смерти называют «крайне острым радиоактивным отравлением», то есть вы бы попросту сварились.

Но седьмой ряд был бы еще хуже!



В самом низу таблицы есть некоторое количество странных элементов, которые называют трансурановыми. Долгое время у них были «имена-болванки» вроде «унунуний» и все в таком роде, но постепенно они получают настоящие названия.

Однако торопиться тут не стоит, потому что большая часть этих элементов настолько нестабильна, что их можно получить только в ускорителе частиц и они не могут существовать дольше нескольких минут. Если бы у вас вдруг оказалось 100 000 атомов ливермория (116-й элемент), спустя секунду остался бы один, и он тоже исчез бы через несколько сотен миллисекунд.

Но как это ни печально для нашего проекта, трансурановые элементы не уходят тихо и незаметно. Они исчезают в ходе радиоактивного распада. И большая их часть распадается на составляющие, которые, в свою очередь, тоже распадаются. Кубик любого элемента с достаточно большим порядковым номером распался бы за секунды, высвободив при этом огромное количество энергии.

Результат был бы не просто похож на ядерный взрыв, собственно, это и был бы ядерный взрыв. Однако, в отличие от бомбы, в нашем случае мы имели бы дело не с цепной, а с обычной реакцией. Все произошло бы мгновенно.



Поток высвободившейся энергии немедленно превратил бы вас и всю остальную таблицу в плазму. Происходящее напоминало бы взрыв ядерного заряда средней мощности, однако радиоактивное заражение было бы гораздо, гораздо хуже – на землю выпал бы настоящий салат из всего содержимого Периодической таблицы, и при этом элементы с невероятной скоростью превращались бы один в другой.

Грибовидное облако поднялось бы над городом. Верхушка его под действием собственного жара достигла бы стратосферы. Если ваша лаборатория находится в густонаселенной зоне, то число жертв в первые же секунды после взрыва было бы колоссальным, однако долговременные последствия в результате заражения оказались бы еще хуже.

Причем это было бы не какое-то там заурядное, обыденное радиоактивное заражение[20], нет, это было бы похоже на ядерную бомбу, которая продолжает и продолжает взрываться. Обломки, излучающие больше радиации, чем вся чернобыльская катастрофа, покрыли бы весь земной шар. Целые регионы были бы уничтожены, и их дезактивация заняла бы столетия.

В общем, собирать коллекцию, безусловно, очень весело, но когда речь заходит о химических элементах, не пытайтесь собрать их все.


Прыг-скок!

ВОПРОС: А что, если все жители Земли встали бы рядом и разом подпрыгнули, а потом одновременно приземлились?

– Томас Беннет (и многие другие)

ОТВЕТ: Это один из самых популярных вопросов, которые задают на моем сайте. На него уже отвечали другие, включая Science Blogs и The Straight Dope. Они неплохо описывают кинетические аспекты ситуации. Однако это только часть истории.

Давайте приглядимся внимательнее.

Сначала предположим, что все население Земли магическим образом собралось в одном месте.



Вся эта толпа займет площадь, сопоставимую с площадью штата Род-Айленд. Впрочем, не вижу необходимости использовать обтекаемые обороты типа «площадь, сопоставимая с площадью»: это наш сценарий, и мы можем позволить себе любую степень точности. Итак, все человечество и в самом деле собралось в штате Род-Айленд.



Часы бьют полдень, и все одновременно подпрыгивают.



Как уже отмечалось в других источниках, сам прыжок не особенно повлияет на нашу планету. Земля весит больше, чем населяющие ее люди, примерно в 10 миллиардов раз. В среднем человек подпрыгивает в лучшем случае где-то на высоту полуметра. Даже если бы Земля была абсолютно жесткой и мгновенно отреагировала, мы ее оттолкнули бы меньше чем на диаметр атома.

И тут мы все снова приземляемся…



Теоретически это означает, что земной шар получит немало энергии, но она распространится по достаточно большой площади, так что все мы разве что оставим отпечатки наших ног на земле. Небольшое давление распространится по континентальной коре Северной Америки и угаснет без особых последствий.

Звук всех этих ступней, ударяющихся о землю, прозвучит как громкий хлопок, который продлится несколько секунд.

В конце концов все стихает. Проходят еще секунды.



Люди смущенно переглядываются. Кто-то кашляет.



Наконец, кто-то первым достает из кармана телефон. Через несколько секунд на свет извлекаются все пять миллиардов телефонов, существующих на Земле. Все они, даже те, которые работают в местной сети, показывают одну из версий сообщения «нет сигнала». Сотовые сети рухнули под беспрецедентной нагрузкой. Тем временем за пределами Род-Айленда начинают останавливаться брошенные на произвол судьбы машины и механизмы.

Аэропорт в Уорике, Род-Айленд, способен обслужить несколько тысяч пассажиров в день. Если предположить, что отправка человечества из штата была организована заранее (то есть были в том числе организованы специальные службы, призванные обеспечить аэропорты штата топливом), то аэропорту в Уорике придется работать с эффективностью 500 % в течение многих лет, но толпа практически не уменьшится. В ней даже не появится прорех.

Если включить в работу все близлежащие аэропорты, то и тогда ситуация не слишком изменится. То же касается местной системы железных дорог. Толпы людей штурмуют грузовые корабли у причалов порта Провиденс, но запасти достаточно воды и еды для долгого путешествия весьма непросто.



Полмиллиона автомобилей штата Род-Айленд мобилизованы. Спустя мгновение автострады I-95, I-195 и I-295 превращаются в самую крупную пробку в истории планеты. Большая часть машин намертво застрянет, но нескольким счастливчикам удастся выбраться с автострады и искать объезда по проселочным дорогам.

Некоторые доберутся до Нью-Йорка или Бостона, прежде чем у них закончится топливо. Поскольку электричества к этому моменту уже, скорее всего, не будет, проще бросить свою машину и угнать другую, чем найти работающую заправку. Кто вас остановит? Все полицейские остались в Род-Айленде.

Своими краями толпа распространится в южный Массачусетс и в Коннектикут. Два случайных человека в толпе, вероятно, не будут знать языка друг друга, и практически никто не будет знаком с местностью. Штат Род-Айленд превратится в хаотическую мешанину возникающих и тут же рушащихся социальных связей. Жестокость станет обыденностью. Все голодны и все хотят пить. Магазины опустошаются. Питьевую воду достать непросто, и еще сложнее ее доставить…

В течение нескольких недель Род-Айленд превратится в кладбище миллиардов людей. Выжившие расселятся по всему миру и попытаются построить новую цивилизацию на свежих руинах предыдущей. Наш вид выжил, но популяция сильно уменьшилась. Орбита Земли осталась совершенно без изменений – планета вращается точно так же, как и до того, как весь вид Homo sapiens разом подпрыгнул.

Зато теперь мы знаем ответ.


Моль кротов

ВОПРОС: А что, если собрать моль кротов[21] в одном месте?

– Шон Райс

ОТВЕТ: Последствия будут просто кошмарными.

Сначала несколько определений. Моль – это единица измерения. Однако это не стандартная единица. В сущности, это просто число – как «дюжина» или «миллиард». Если у вас есть моль чего-то, это значит, что у вас есть 602 214 129 000 000 000 000 000 этих штук (обычно записывается как 6,022x10??). Это число столь велико[22], потому что его используют для подсчета молекул, которых обычно очень много.

Крот – это норное млекопитающее. Существует несколько видов кротов, и некоторые из них поистине ужасающие создания[23].

Так как же будет выглядеть моль кротов – 602 214 129 000 000 000 000 000 зверьков разом?



Давайте начнем с самых диких допущений. Вот каков был бы приблизительно ход моих мыслей еще до того, как я взял бы в руки калькулятор. Давайте попытаемся ощутить порядок этого числа – а оно столь огромно, что по сравнению с ним числа 10, 1 и 0,1 кажутся совсем близкими одно к другому, практически равными. Давайте допустим, что они равны.



Крот достаточно маленькое животное, чтобы я мог взять его и подбросить в воздух [источник не указан]. Допустим, что предмет, который я могу подбросить, весит один фунт. Допустим далее, что один фунт равен одному килограмму. Число 602 214 129 000 000 000 000 000 выглядит в два раза длиннее триллиона, то есть это примерно триллион триллионов. Насколько я помню, триллион триллиона килограммов – это как раз столько, сколько весит наша планета.


…если кто-нибудь у вас поинтересуется: я не утверждал, что вычислять подобным образом правильно.


В общем, ясно, что мы имеем дело с кучей кротов размером примерно с планету. Это довольно грубая оценка, так как она может колебаться на несколько порядков в обе стороны.

Давайте подсчитаем точнее.

Восточноамериканский крот (Scalopus aquaticus) весит около 75 граммов, то есть моль таких кротов равен:

(6,022 x 10??) x 75 г ? 4,52 x 10?? кг

Это чуть больше половины массы нашей Луны.

Млекопитающие по большей части состоят из воды. Объем килограмма воды – один литр, так что если кроты весят 4,52x10?? килограмма, то их объем примерно равен 4,52x10?? литра. Вы наверняка заметили, что мы игнорируем зазоры между кротами. Через секунду вы поймете, почему.

Кубический корень из 4,52x10?? л 3562 км, то есть речь идет о сфере с радиусом в 2210 км или кубе с гранями по 2561 км.

Если бы эти кроты оказались на поверхности Земли, они бы покрыли ее слоем толщиной 80 км, верхняя граница которого проходила там, где (прежде) заканчивалась атмосфера.



Этот душный океан сдавленного мяса уничтожил бы своей тяжестью большую часть жизни на планете, так что проделывать все это на Земле, очевидно, плохая затея.

Вместо этого давайте выведем наших кротов в межпланетное пространство. Гравитация соберет их в сферу. Мясо не очень хорошо сжимается под давлением, так что под влиянием гравитации кроты лишь слегка уменьшатся в объеме, и у нас получится кротовая планета чуть больше Луны.

Ускорение свободного падения на поверхности планеты кротов составит примерно 1/16 от земного, приблизительно как на поверхности Плутона. Планета сначала была бы слегка теплой – температура, вероятно, чуть выше комнатной, а затем гравитационное сжатие нагреет внутреннее пространство на несколько градусов.



И тут произойдет нечто весьма странное.

Планета кротов будет представлять собой гигантскую сферу из мяса. У нее много латентной энергии (в кротовой планете будет достаточно калорий, чтобы обеспечить нынешнее население Земли энергией на 30 миллиардов лет вперед). Обычно, когда органика разлагается, она выделяет большую часть этой энергии в виде тепла. Но в глубине планеты давление будет выше 100 мегапаскалей, и этого достаточно, чтобы уничтожить все бактерии и стерилизовать таким образом останки кротов, просто не останется микроорганизмов, которые могли бы разлагать ткани.

Ближе к поверхности, где давление будет ниже, разложению будет препятствовать почти полное отсутствие кислорода. Без кислорода обычное разложение невозможно, и единственные бактерии, которые смогут обеспечить этот процесс, анаэробные, то есть те, которым кислород не требуется. Анаэробное разложение не очень эффективно, но выделяет довольно много тепла. Если этому процессу ничто не будет препятствовать, он сможет раскалить планету до температуры кипения.

Но разложение вскоре начнет контролировать себя само. Мало какие бактерии могут выжить при температурах выше 60 °C, так что, когда температура повысится, бактерии начнут умирать и разложение замедлится. По всей планете трупики кротов будут постепенно превращаться в кероген, органический материал, который в конце концов, будь планета еще горячее, стал бы нефтью.

Внешняя поверхность планеты будет излучать тепло в космос, остынет и заледенеет. Поскольку кроты в буквальном смысле образуют вокруг планеты меховое манто, то, когда замерзнет верхний слой, он изолирует внутреннюю часть планеты и замедлит потерю тепла. Однако потоками тепла в жидком ядре в основном будет управлять конвекция. Потоки горячего мяса и пузыри газов наподобие метана, а также воздух из легких скончавшихся кротов станут периодически пробиваться сквозь замерзшую кору кротовой планеты. В результате возникнут вулканические выбросы, настоящие гейзеры смерти, извергающие в пространство тела кротов, из которых состоит наша планета.

После нескольких веков или тысячелетий кротовая планета успокоится и охладится в достаточной степени, чтобы промерзнуть насквозь. Внутреннее ядро окажется под таким давлением, что вода в нем кристаллизируется в экзотические формы льда, такие как лед III и лед V, а в конце концов лед II и лед IX[24].

Как ни крути, получается довольно печальная картинка. К счастью, возможен и другой вариант развития событий.

Я не знаю, каков размер всемирной популяции кротов (или в принципе объем биомассы мелких млекопитающих), но наугад прикинем, что на каждого человека на Земле приходится хотя бы пара десятков мышей, крыс, полевок и прочих мелких зверьков.

С другой стороны, в нашей галактике должны быть миллиарды планет, пригодных для обитания. При их колонизации мы бы точно завезли с собой мышей и крыс. Если бы даже одна планета из сотни была заселена мелкими грызунами в количестве, сопоставимым с земным, то спустя несколько миллионов лет совсем недолго по меркам эволюции общее количество когда-либо живших грызунов превысило бы число молекул в моле.

В общем, хотите развести моль кротов – постройте себе космический корабль…


Фен в ящике

ВОПРОС: А что, если засунуть постоянно работающий фен в воздухонепроницаемый ящик размером метр на метр на метр?

– Dry Paratroopa

ОТВЕТ: Обычный фен потребляет 1875 Вт энергии.

Все эти 1875 Вт должны быть на что-то потрачены. Что бы ни происходило в ящике, но если внутри него потребляется 1875 Вт электроэнергии, то в конце концов они должны преобразоваться в 1875 Вт тепловой энергии.

Полезно иметь в виду, что это касается любого устройства, потребляющего энергию. Люди беспокоятся, оставив дома в розетке какое-нибудь зарядное устройство: не потребляет ли оно энергию? На этот вопрос легко ответить, потрогав прибор, включенный в сеть: если зарядка не теплая на ощупь, значит, она потребляет электричества меньше, чем на пенни в день. Маленькая зарядка для телефона, если она холодная на ощупь, потребляет меньше, чем на пенни в год. Это верно почти для любого электрического устройства[25].

Но вернемся к нашему ящику.

Тепло пойдет от фена в ящик. Допустим, что наш фен нельзя сломать, уничтожить или разрушить. Тогда ящик будет разогреваться изнутри до тех пор, пока его наружная поверхность не достигнет температуры примерно 60 °C. При этой температуре ящик будет излучать тепло с той же скоростью, с которой фен его выделяет, и система будет находиться в равновесии.


Он теплее моих родителей! Теперь он – мои новые родители.


Температура, при которой будет достигнуто равновесие, может быть чуть меньше, если на ящик веет ветерок или если ящик стоит на влажной или металлической поверхности, которая хорошо отводит тепло.

Если ящик сделан из металла, он будет достаточно горячим, чтобы обжечь вашу руку, если вы положите ее на ящик дольше, чем на пять секунд. Если у нас деревянный ящик, то в течение некоторого времени на нем можно будет держать руку, но есть опасность, что те его части, которые соприкасаются с феном, рано или поздно вспыхнут.

Внутри ящик будет похож на духовку. Температура, которой он достигнет, зависит от толщины стенок: чем более толстые стенки у ящика и чем хуже они проводят тепло, тем выше будет температура. Понадобятся не слишком толстые стенки, чтобы создать температуру, при которой фен сгорит.

Но предположим еще раз, что наш вообще не поддается разрушению. И если уж у нас есть такая классная штука, как неразрушимый фен, было бы просто глупо ограничивать его мощность жалкими 1875 Вт.



Если наш фен будет потреблять (и излучать) в десять раз больше – 18 750 Вт, то температура поверхности ящика превысит 200 °C (это как сковородка на слабом огне).



Интересно, насколько нам хватит шкалы на переключателе.


Пока что на шкале пугающе много места.


Теперь поверхность ящика раскалилась до 600 °C – достаточно, чтобы она начала светиться красным.



Если наш ящик сделан из алюминия, ящик начнет плавиться внутри. Если из свинца, то он начнет плавиться и снаружи. Если он из дерева, то пожар вам обеспечен. Но совершенно неважно, что происходит вокруг фена, – сам-то он у нас не поддается разрушению.



Двухмегаваттного лазера достаточно, чтобы уничтожить летящую ракету.

При температуре 1300 °C ящик станет примерно таким же горячим, как жидкая лава.



Добавим-ка еще.


Фен, возможно, не предназначался для этого…


Теперь в ящик течет 18 мегаватт энергии.



Поверхность ящика достигла температуры 2400 °C. Будь он сделан из стали, она бы уже расплавилась. Если же наш ящик изготовлен из чего-то вроде вольфрама, то он сможет протянуть чуть дольше.


Еще чуть-чуть, и придется остановиться.


Мощности 187 мегаватт достаточно, чтобы ящик раскалился добела. Немногие материалы могут уцелеть в подобных условиях, так что придется предположить, что наш ящик тоже не поддается разрушению.


Пол превратился в лаву.


К сожалению, про пол этого не скажешь.

Предположим, что прежде чем ящик прожег пол насквозь, кто-то бросит рядом с ним воздушный шарик, наполненный водой. Струя пара немедленно вышвырнет ящик через входную дверь на улицу[26].



Так, теперь у нас тут 1875 гигаватт (я соврал, сказав, что мы остановимся). Если верить фильму «Назад в будущее», сейчас у фена достаточно мощности, чтобы он смог путешествовать во времени.



Ящик светится ослепительно ярко, и подойти к нему ближе, чем на несколько сотен метров, нельзя из-за сильного жара. Вокруг разрастается озеро лавы. Все, что находится в радиусе 50–100 метров, вспыхивает. Столб жара и дыма поднимается в воздух. Периодические взрывы газа под ящиком подбрасывают его в воздух, и там, где он приземляется, начинаются новые пожары и возникают новые озера лавы.

Но мы продолжаем поворачивать переключатель.

При мощности 18,7 гигаватт происходящее вокруг ящика начинает напоминать запуск космического корабля. Ящик бросают из стороны в сторону мощные восходящие потоки, которые он сам же и создает.

В 1914 году Герберт Уэллс описывал похожие приспособления в книге «Освобожденный мир»: бомба, которая вместо того, чтобы взорваться один раз, взрывается непрерывно – медленно разгорающийся ад, превращающий города в костры, которые невозможно потушить. Эта мрачная история предвосхищает развитие ядерного оружия 30 лет спустя.

Теперь наш ящик парит в воздухе. Каждый раз, вновь приближаясь к земле, он мгновенно разогревает поверхность, и стремительно расширяющийся раскаленный воздух снова подбрасывает его вверх.

Энергия 1875 тераватт сравнима с энергией взрыва склада динамита размером с большой дом, при этом склад продолжает взрываться ежесекундно.




В окрестностях ящика бушуют огненные бури – гигантские пожары, которые раздувает созданная ими же система потоков раскаленного воздуха.

Мы взяли новый рубеж: теперь фен, как ни невероятно это звучит, потребляет больше энергии, чем все остальные электроприборы на планете, вместе взятые.

Ящик, парящий высоко над поверхностью земли, каждую секунду излучает энергию, сравнимую с мощностью ядерного испытания «Тринити»[27].

К этому моменту дальнейший ход событий ясен. Эта штука будет летать по атмосфере, пока не уничтожит планету.

Давайте попробуем что-то изменить.

Переключим мощность на ноль, когда ящик будет пролетать над Северной Канадой. Быстро остывая, ящик начнет падать, пока в облаке пара не плюхнется в Большое Медвежье озеро.



А затем…

В данном случае это 11 петаватт.


Короткое отступление:

Официальный рекорд скорости объекта, созданного человеком, поставил зонд «Гелиос-2», который разогнался примерно до 70 км/с на орбите вокруг Солнца. Но возможно, что на самом деле этот титул по праву принадлежит металлической крышке люка весом две тонны.

Эта крышка закрывала шахту подземного ядерного полигона в Неваде во время испытаний в 1957 году. Когда в шахте был взорван заряд мощностью в одну килотонну, шахта превратилась, по сути, в ствол ядерной пушки, всей своей мощью вытолкнувшей вверх крышку люка. Камера, нацеленная на крышку, сделала только один кадр, прежде чем крышка исчезла из поля зрения, то есть она двигалась как минимум со скоростью 66 км/с. Эту крышку потом так и не нашли.

Так вот, хотя 66 км/с – это в шесть раз больше, чем вторая космическая скорость, маловероятно, чтобы крышка достигла космического пространства (что бы ни гласили по этому поводу разные популярные спекуляции). Расчеты показывают, что крышка либо была полностью уничтожена при столкновении с воздухом, либо в конце концов замедлилась и упала обратно на землю. Но если бы крышке все-таки удалось выйти с такой скоростью в космос, она смогла бы покинуть Солнечную систему лет через двадцать.


Когда мы снова включим фен, наш ящик, бултыхающийся в озере, переживет нечто аналогичное. Раскаленный пар под ним начнет расширяться, и когда ящик станет подниматься в воздух, вся поверхность озера обратится в пар. Пар, раскаленный потоком излучения до состояния плазмы, будет ускорять ящик все больше и больше.


Фотография из архива командира МКС Хэдфилда


Вместо того чтобы врезаться в плотный слой воздуха, как крышка от люка, ящик пролетит через сферу расширяющейся плазмы, которая окажет ему очень мало сопротивления. Затем выйдет из атмосферы и продолжит лететь, медленно угасая, превращаясь из второго солнца в тусклую звездочку. Да, большая часть Северо-Западных территорий Канады сгорит, зато сама Земля уцелеет!



Представьте, кое-кто все равно останется этим недоволен.

Странные (и тревожные) вопросы из папки «Входящие» сайта «А что, если?»

ВОПРОС: А что, если бы в чернобыльский реактор во время аварии сбросили антиматерию? Он прекратил бы плавиться?

– Эй-Джей

ВОПРОС: Можно ли плакать так долго, что наступит полное обезвоживание организма?

– Карл Уилдэрмут

Последний свет человечества

ВОПРОС: А что, если бы все люди одновременно исчезли с лица Земли? Через сколько времени погаснет последний искусственный источник света?

– Алан

ОТВЕТ: На титул «последнего огонька» будет много претендентов.

Прекрасная книга Алана Вейсмана «Земля без людей» подробно описывает, что произошло бы с домами, дорогами, небоскребами, фермами и домашними животными, если бы люди внезапно исчезли. Научно-популярный сериал «Жизнь после людей» посвящен аналогичной теме. Однако ни в книге, ни в сериале нет ответа на данный конкретный вопрос.

Начнем с очевидного: большая часть искусственных источников света долго не протянет, потому что основные электромагистрали довольно быстро разрушатся. Тепловые электростанции, которые обеспечивают большую часть электричества на Земле, требуют постоянной подачи топлива, а для этого именно люди должны принимать какие-то решения.

В отсутствие людей уменьшится потребность в энергии, но термостаты в наших квартирах будут продолжать работать. Угольные шахты и нефтяные вышки отключатся в первые же несколько часов, остальные предприятия подхватят у них эстафету. Эту ситуацию сложно решить даже с участием людей. В результате начнется нарастающая лавина отключений, которая приведет к полному параличу всех основных энергосетей.



Однако немало электричества исходит из источников, которые не зависят от основных сетей тепловых электростанций. Давайте взглянем на некоторые из них и посмотрим, когда они могут отключиться.

Дизельные генераторы

Многие люди, живущие, к примеру, на далеких островах, получают энергию при помощи дизельных генераторов. Эти устройства будут работатать, пока у них не закончится топливо, а этот срок в большинстве случаев составит от нескольких дней до нескольких месяцев.

Геотермальные электростанции

Источники электроэнергии, которым вообще не нужно топливо, поставляемое людьми, окажутся в более выигрышной ситуации. Например, геотермальные станции, использующие тепло недр планеты, некоторое время обойдутся без вмешательства людей.

Однако инструкция по эксплуатации геотермальной станции на острове Свартсенги в Исландии гласит, что раз в шесть месяцев необходимо заменять масло в трансмиссиях и смазывать все электрические моторы и сочленения. Без обслуживающего персонала, способного это сделать, некоторые станции проработают еще несколько лет, но в конце концов все механизмы разрушатся.

Ветряные турбины

Машинам, действие которых основано на силе ветра, повезет больше. Ветряки специально конструируют таким образом, чтобы они не требовали постоянного ухода, по той простой причине, что их много и залезать на них большая морока.

Некоторые ветряные турбины могут работать очень долго без вмешательства людей. Гедсерский ветряк в Дании был установлен в конце 1950-х и вырабатывал энергию в течение 11 лет без единого ремонта. Современные турбины обычно рассчитаны на 30 000 часов (3 года) непрерывной работы без технического обслуживания, и, без сомнения, некоторые из них проработают еще десятилетия. И хотя бы в одном из них наверняка будет светиться хотя бы один жидкокристаллический экран на панели какого-нибудь индикатора.

Но в конце концов большая часть ветряных турбин остановится по той же причине, что и геотермальные станции: их движущиеся части заклинит без смазки.

Гидроэлектростанции

Генераторы, превращающие энергию падающей воды в электричество, продолжат какое-то время работать. Документальный сериал «Жизнь после людей» показывал интервью с оператором электростанции на плотине Гувера, который сказал, что, если бы все сотрудники ее покинули, она проработает еще несколько лет на автопилоте. Рано или поздно станция, скорее всего, станет жертвой либо засоров в системе забора воды, либо тех же механических поломок, что убьют ветряки и геотермальные станции.

Батарейки

Огоньки, использующие электричество батареек, погаснут в течение одного-двух десятилетий. Даже если батарейка не используется, она все равно постепенно разряжается. Некоторые типы батарей проживут дольше остальных, но даже те из них, которые, согласно рекламе, приспособлены для длительного хранения, смогут удерживать заряд всего десять-двадцать лет.



Есть несколько исключений. В Кларендонской библиотеке Оксфордского университета есть колокольчик на батарейке, который работает с 1840 года. Его звон практически не слышен, а на каждый удар молоточка тратится микроскопическая доля заряда. Никто толком не знает, какая именно батарея используется в колокольчике, потому что никто ни хочет разбирать его.



К сожалению, никакого света колокольчик не дает.

Ядерные реакторы

Долговечность ядерного реактора – непростой вопрос. Перейдя в режим низкой мощности, реактор может работать почти бесконечно, так высока удельная энергоемкость ядерного топлива.

Как сказано в одном интернет-комиксе:



К сожалению, на практике реакторы все равно проработают недолго. Как только хоть что-нибудь пойдет не так, ядро реактора автоматически перейдет в режим остановки. Возможно, это произойдет довольно быстро; причиной могут стать разные факторы, но наиболее вероятным будет отключение подачи энергии на реактор.

Может показаться странным, что для работы энергетической станции требуется внешний источник энергии, но каждая часть управляющей системы ядерного реактора спроектирована так, чтобы любой сбой немедленно заставил реактор остановиться. Как только извне перестанет поступать энергия – из-за того ли, что внешняя электростанция перестанет работать, или у запасных генераторов закончится топливо, – реактор отключится.

Космические зонды

Из всех творений человека наши космические аппараты могут оказаться самыми долговечными. Некоторые из них останутся на орбите на миллионы лет, но их электрическое оборудование не проживет так долго.

Пройдут столетия, и наши марсоходы будут погребены глубоко в пыли. К тому времени многие спутники рухнут обратно на землю, сойдя с орбиты. Спутники GPS на дальних орбитах протянут дольше, но со временем даже самые стабильные орбиты будут искажены под влиянием Луны и Солнца.

Многие космические аппараты работают за счет солнечных панелей, другие за счет энергии радиоактивного распада.

Марсоход «Кьюриосити», к примеру, использует энергию тепла, выделяемого кусочком плутония: этот радиоактивный материал находится в контейнере, укрепленном на вертикальном штоке.



«Кьюриосити» мог бы получать электроэнергию в течение более чем ста лет. В конце концов напряжение станет слишком слабым для его работы, но скорее всего, другие детали машины износятся еще раньше.

Итак, «Кьюриосити» выглядит многообещающе. Есть одна проблема – у него отсутствуют какие-либо осветительные приборы.

Строго говоря, у «Кьюриосити» есть фары – он использует их, чтобы подсвечивать образцы и проводить спектроскопию. Однако они включаются только тогда, когда проходят измерения. Без соответствующих инструкций (которые должны дать люди) у него не будет повода их включать.

Если на борту нет людей, космическому аппарату не очень-то нужны источники света. Зонд «Галилео», который исследовал Юпитер в 1990-е, был снабжен несколькими светодиодами в механизме устройства, записывавшего информацию о полете. Но поскольку они излучали волны в инфракрасном диапазоне, называть их «огнями» не вполне корректно. Да и в любом случае «Галилео» больше нет – как и было запланировано, зонд врезался в Юпитер в 2003 году[28].

На других наших аппаратах тоже имеются источники света. Например, некоторые спутники GPS используют ультрафиолетовые светодиоды, чтобы контролировать статическое напряжение на поверхности оборудования, и эти элементы работают от солнечных батарей. Теоретически они могут работать столько, сколько будет светить Солнце. К сожалению, большая их часть не протянет хотя бы столько, сколько «Кьюриосити», и рано или поздно будет уничтожена в результате столкновения с космическим мусором.

Но солнечные батареи используются не только в космосе.

Солнечная энергия

Телефонные будки для экстренных вызовов, стоящие вдоль дорог в отдаленных местностях, часто работают на солнечной энергии. Обычно они оборудованы фонариком, который горит по ночам.



Так же как и ветряки, эти заброшенные в глуши устройства довольно хлопотно обслуживать, поэтому они спроектированы в расчете на длительную работу. Если солнечные панели периодически очищать от пыли и мусора, они проработают так же долго, как и электроника, которая к ним подключена.

Провода и электрические контакты рано или поздно разъест коррозия, но солнечные панели, находящиеся в сухом месте и подключенные к качественной электронике, смогут спокойно вырабатывать энергию в течение столетия, особенно если ветер или дождь время от времени будут очищать их от пыли.



Если следовать четкому определению понятия «свет», то огни, получающие питание от солнечных батарей в удаленных уголках планеты, могли бы оказаться последним уцелевшим источником искусственного света[29].

Но есть еще один претендент, и он довольно странный.

Эффект Вавилова – Черенкова

Обычно радиоактивность невидима.

Когда-то циферблаты часов покрывали слоем радия, чтобы они светились. Однако светилась не сама радиоактивность, а фосфоресцирующая краска, наносившаяся поверх радия и испускавшая свет под действием излучения. С годами краска осыпалась, и хотя эти часы все еще радиоактивны, они уже не светятся.

Однако циферблаты – не единственный радиоактивный источник света.

Когда радиоактивные частицы проходят через материалы наподобие воды или стекла, они светятся, поскольку движутся быстрее скорости света в этой среде (но, конечно, медленнее скорости света в вакууме). Это называется эффектом Вавилова – Черенкова, его пример – узнаваемое голубое свечение ядра ядерного реактора.

Некоторые из наших радиоактивных отходов, например цезий-137, расплавляют и смешивают со стеклом, затем охлаждают в плотные бруски, которые обматывают изоляцией для безопасной транспортировки и хранения.

В темноте они светятся синим.

Цезий-137 имеет период полураспада 30 лет, значит, спустя двести лет они все еще будут светиться с мощностью в 1 % от изначальной радиоактивности. Свет будет угасать со временем, но сохранит тот же синий цвет.

Вот и ответ. Спустя столетия в безлюдных бетонных бункерах будет по-прежнему светиться синим светом наш радиоактивный мусор.


Пулеметный ракетный ранец

ВОПРОС: А что, если построить ракетный ранец, использовав для этого автомат, направленный стволом вниз?

– Роб Б.

ОТВЕТ: Я был несколько удивлен, когда узнал, что эту идею вполне можно реализовать! Но чтобы сделать все правильно, придется договариваться с русскими.

Принцип здесь довольно простой. Если вы стреляете вперед, отдача толкает вас назад. Следовательно, если стрелять вниз, отдача должна подбросить вас вверх.

Прежде всего нам придется ответить на вопрос, сможет ли автомат в принципе поднять собственный вес? Если он, допустим, весит 4 кг, а сила его отдачи – всего 3 кг, автомат не сможет оторваться от земли, не говоря уже о том, чтобы заодно поднять человека.

В инженерном деле соотношение веса и отдачи называется тяговооруженностью. Если она меньше единицы, аппарат не сможет подняться в воздух. Ракета-носитель «Сатурн-5» обладала тяговооруженностью, примерно равной 1,5.

Я не большой эксперт в области огнестрельного оружия (несмотря на то, что вырос на юге США), поэтому для того, чтобы ответить на вопрос Роба, я связался со своими приятелями в Техасе[30].


Примечание: ПОЖАЛУЙСТА, не пытайтесь проделать это дома!



Как выясняется, у автомата Калашникова АК-47 тяговооруженность составляет примерно 2. Это значит, что, если поставить его автомат стволом и каким-то образом зафиксировать спусковой крючок в нажатом состоянии, автомат должен подняться в воздух, одновременно выпуская очередь.

Это верно не для всех видов автоматического оружия. Например, американской винтовке М-60, скорее всего, не хватит отдачи, чтобы оторваться от земли.

Сила тяги, которую развивает ракетный двигатель (или стреляющий автомат Калашникова), зависит, во-первых, от массы выброса, а во-вторых, от того, насколько быстро он ее выбрасывает. Тяга – это произведение двух этих величин.

Если АК-47 выпускает десять пуль весом по 8 граммов каждая за одну секунду, а начальная скорость каждой пули – 715 м/с, то тяга составит:



Так как заряженный АК-47 весит всего 4,8 кг, то он, по идее, должен взлететь и начать ускоряться вверх.



В действительности тяга окажется даже где-то на 30 % выше. Все дело в том, что автомат выбрасывает не только пули, но и горячий газ, и продукты сгорания пороха. Насколько это все повысит тягу – зависит от конкретного оружия и патронов. Общая эффективность также зависит от того, сбрасываете ли вы гильзы на землю по мере подъема, или берете их с собой. Я попросил своих техасских приятелей взвесить для моих расчетов несколько гильз. В ответ они сообщили мне, что не смогли найти весы, и тогда я подсказал им, что, учитывая объем их арсенала, им, в сущности, просто нужно найти кого-то еще, у кого весы есть[31].

В общем, АК-47 смог бы взлететь, но у него не хватило бы тяги, чтобы поднять в воздух что-либо тяжелее белки.

Можно попробовать использовать несколько автоматов сразу. Если вниз будут стрелять два ствола, они создадут в два раза больше тяги. Если каждый автомат может поднять плюс к своей массе еще 2,5 кг, то два автомата поднимут 5 кг.

Уже понятно, к чему все идет:



В космос вы сегодня не попадете.


Если сложить вместе достаточное количество направленных вниз «калашниковых», то вес пассажира почти перестает иметь значение: он распределится по числу стволов и на каждый из последних придется ничтожная доля этого веса. Поскольку наше устройство, по сути дела, представляет собой пучок из множества отдельных автоматов, летящих параллельно, то чем больше будет количество «калашниковых», тем больше тяговооруженность нашего аппарата будет приближается к тяговооруженности отдельно взятого автомата:



Но есть одна проблема: боеприпасы.

В магазине АК-47 30 патронов. При десяти выстрелах в секунду это даст нам жалкие три секунды ускорения.

Это время можно увеличить, сделав магазин больше, но только до определенного момента: как выясняется, набивать в магазин больше 250 патронов не имеет смысла. Причина этого кроется в классической проблеме ракетостроения – топливо делает вас тяжелее.

Каждая пуля весит 8 граммов, а патрон (то есть пуля с гильзой и пороховым зарядом) весит более 16 граммов. Если в магазине будет больше 250 патронов, «калашников» станет слишком тяжелым и не сможет взлететь.

Итак, наш ранец должен состоять из большого количества АК-47 (минимум 25, но в идеале хотя бы 300), и в магазине каждого из них должно быть 250 патронов. Самые мощные варианты аппарата могли бы взлетать вверх на скорости до 100 м/с, поднимаясь на высоту полукилометра с лишним.

Вот мы и ответили на вопрос Роба: да, имея достаточное количество автоматов, взлететь можно.

Однако наша связка «калашниковых» – явно не самый совершенный реактивный ранец. Можно ли придумать что-то поинтереснее?

Мои техасские друзья предложили мне на выбор целый ряд различных пулеметов и автоматов, и я рассчитал результаты для каждого из них. Некоторые оказались весьма неплохи. Скажем, пулемет MG-42 (эта штука потяжелее, чем АК-47) может похвастаться чуть большей тяговооруженностью, чем «калашников».

Однако потом мы решили замахнуться на большее.

Авиационная пушка GAU-8 Avenger выпускает до 60 фунтовых снарядов в секунду. Отдача этой пушки эквивалентна почти 5 т, и это просто невероятно, учитывая, что ее устанавливают на штурмовике А-10 Thunderbolt, два двигателя которого вместе создают тягу всего в 4 т. Если установить на самолет две таких пушки и одновременно выстрелить из обеих вперед, одновременно включив двигатели на полную мощность, то пушки победят, и вы полетите назад.

А если установить GAU-8 на мою машину, включить нейтральную передачу и начать стрелять назад, то я нарушил бы любые ограничения скорости меньше чем за три секунды.


«Вообще мне гораздо интереснее, как вам это удалось».


Но как бы хорошо эта пушка ни подходила для ракетного ранца, русские создали установку, которая сработает еще лучше. Зенитная автоматическая пушка Грязева – Шипунова 6К30ГШ весит примерно вдвое меньше, чем GAU-8, и при этом ее скорострельность еще выше. Тяговооруженность этого орудия приближается к 40, а значит, если вы, сидя на ней верхом, направите ее вертикально вниз и выстрелите, она не просто взлетит в облаке смертоносных осколков, но вы еще и испытаете бы при этом ускорение в 40g.

Это уже слишком. На самом деле, даже когда пушка надежно закреплена на самолете, отдача представляет собой проблему:

Отдача… все еще наносила ущерб самолету. Скорость стрельбы была уменьшена до 4000 выстрелов в минуту, но это не очень помогло. Посадочные фары почти всегда ломались после выстрела… Очередь из более чем 30 выстрелов почти непременно означала перегрев…

Грег Гебель, airvectors.net

Но если бы вам каким-то образом усадить на пушку пассажира, сделать всю конструкцию достаточно прочной, чтобы она выдержала отдачу, засунуть пушку в аэродинамический обтекатель и убедиться, что все это как следует охлаждается…



…то можно было бы прыгать через горы!

Равномерно вверх

ВОПРОС: А что, если вы внезапно начнете подниматься вверх со скоростью одного фута в секунду, то как именно вы умрете? Сначала замерзнете? Или сначала задохнетесь? Или как-то еще?

– Ребекка Б.

ОТВЕТ: Ребекка, ты не забыла надеть пальто?

Один фут (30 см) в секунду – это не так уж быстро. Это значительно медленнее, чем скорость стандартного лифта. Вам потребуется 5–7 секунд, прежде чем вы подниметесь так высоко, чтобы никто из ваших друзей уже не сможет до вас дотянуться (конечно, это зависит от их роста).

Спустя 30 секунд вы будете в 30 футах (9 метрах) над землей. А если вы сейчас пролистаете книгу до страницы 200, то узнаете, что для какого-нибудь вашего друга это последний шанс бросить вам сэндвич, бутылку воды или еще что-нибудь[32].



Через минуту или две вы будете уже парить над верхушками деревьев. В общем и целом вам будет столь же комфортно, как и на земле. Если день ветреный, возможно, станет прохладнее, благодаря более устойчивому ветру над верхней границей леса[33].



Спустя 10 минут вы подниметесь выше всего на земле, кроме самых высоких небоскребов, а спустя 25 минут вы окажетесь уже выше шпиля небоскреба Эмпайр-Стейт-билдинг.



Воздух на такой высоте примерно на 3 % более разреженный, чем на поверхности земли. К счастью, ваше тело постоянно имеет дело с подобными перепадами давления. Уши может заложить, но больше вы ничего особенного не заметите.

Давление воздуха очень быстро меняется вместе с высотой. Как ни удивительно, даже когда вы стоите на земле, изменение давления воздуха поддается измерению даже на относительной высоте всего около метра. Если в вашем смартфоне имеется барометр (как и во многих современных моделях), вы можете скачать соответствующее приложение и наглядно увидеть разницу в давлении на уровне вашей головы и на уровне ступней.

Фут в секунду – это довольно близко к километру в час, так что через час вы будете примерно в километре над Землей. К этому моменту вам уже точно станет холоднее. Если на вас пальто, то все пока будет в порядке, хотя ветер начнет заметно усиливаться.

Спустя еще где-то два часа (и два километра) температура вокруг вас приблизится к нулю. Ветер тоже, скорее всего, будет усиливаться. Если у вас есть открытые участки кожи, возникает реальная опасность обморожения.

К этому моменту давление воздуха упадет ниже давления, которое поддерживается в кабине авиалайнера[34], и последствия этого станут более заметными. Однако если на вас нет теплого пальто, то более серьезной проблемой будет холод.

В течение следующих двух часов температура упадет ниже нуля по любой шкале[35]. Предположим, что вы еще не погибли от нехватки кислорода, тогда в какой-то момент вы станет жертвой переохлаждения. Но когда именно?

Главными научными авторитетами в вопросе замерзания насмерть (и это неудивительно) являются канадцы. Самую популярную модель, описывающую выживание человека на морозе, разработали Питер Тикуисис и Джон Фрим из Института медицины окружающей среды в Онтарио.

Согласно этой модели, основным фактором, от которого зависит, когда именно вы замерзнете насмерть, будет одежда. Если вы голый, то, скорее всего, падете жертвой переохлаждения где-то к шестому часу, то есть еще до того, как в окружающем вас воздухе закончится кислород[36]. Но если вы как следует закутались, то вам грозит обморожение, однако вы можете продержаться еще достаточно долго…

…чтобы достичь живым Зоны смерти.



Примерно на уровне 8000 метров – это выше всего, кроме самых высоких гор, – содержание кислорода в воздухе становится слишком низким. Приближаясь к этой высоте, вы будете испытывать ряд симптомов, включающих, вероятно, спутанность сознания, головокружение, нарушение зрения и тошноту.

При приближении к Зоне смерти уровень кислорода в вашей крови резко упадет. Ваши вены предназначены для того, чтобы нести кровь с низким содержанием кислорода к легким, чтобы восполнить его содержание. Но в Зоне смерти содержание кислорода в воздухе настолько мало, что кровь при газообмене с воздухом легких будет терять кислород, вместо того, чтобы насыщаться им.

Результатом будет быстрая потеря сознания и смерть. Это произойдет где-то в районе семи часов после взлета: шансы, что вы проживете восемь часов, крайне невелики.


Она умерла так же, как жила, – поднимаясь вверх со скоростью один фут в секунду. В смысле, как жила в последние несколько часов.


И вот, спустя два миллиона лет ваше промерзшее насквозь тело, все еще летящее сквозь пространство со скоростью фут в секунду, выйдет через гелиосферу в межзвездное пространство.

Клайд Томбо, астроном, открывший Плутон, умер в 1997 году. Часть его праха поместили в американский космический зонд «Новые горизонты», который в июле 2015 года пролетит мимо Плутона и отправится дальше, за пределы Солнечной системы.



Нет сомнений, что ваше гипотетическое путешествие со скоростью фут в секунду будет холодным, неприятным и в конечном счете смертельным. Зато когда Солнце спустя четыре миллиарда лет превратится в красного гиганта и поглотит Землю, во всей Вселенной не останется ни одного человека, кроме вас и Клайда.

И это тоже неплохо.

Странные (и тревожные) вопросы из папки «Входящие» сайта «А что, если?»

ВОПРОС: Можно ли, учитывая современные познания и возможности человечества, построить новую звезду?

– Джефф Гордон

ВОПРОС: С какими сложностями логистики вы столкнетесь, если решите создать армию человекообразных обезьян?

– Кевин

ВОПРОС: Если бы у людей были колеса и они могли бы летать, как бы мы отличали их от самолетов?

– Аноним

Субмарина на орбите

ВОПРОС: А что, если запустить на орбиту атомную подлодку? Как долго она сможет там продержаться?

– Джейсон Лэтбери

ОТВЕТ: Сама субмарина будет в порядке, но вот у команды возникнут проблемы.

Подлодку не разорвет. Корпусу подлодки приходится выдерживать на глубине наружное давление от 50 до 80 атмосфер, так что корпус спокойно выдержат внутреннее давление воздуха в одну атмосферу.

Скорее всего, корпус окажется воздухонепроницаемым. Хотя водонепроницаемая изоляция необязательно означает, что воздух не сможет через нее проникнуть, тот факт, что вода не может просочиться через обшивку под давлением в 50 атмосфер, говорит о том, что быстро, во всяком случае, воздух через нее не пройдет. На подлодке может быть установлено несколько специализированных односторонних клапанов, выпускающих воздух, но скорее всего, субмарину удастся полностью изолировать от окружающей среды.

Более серьезная проблема, с которой столкнется команда, очевидна – отсутствие воздуха.

Атомные подлодки используют электричество, чтобы извлекать кислород из воды. В космическом пространстве воды нет [источник не указан], так что вырабатывать воздух не получится. У подлодки достаточный запас кислорода, чтобы команда могла прожить как минимум несколько дней, но затем у моряков начнутся проблемы.

Чтобы сохранить тепло, можно включить реактор, но членам команды придется очень точно рассчитать, как долго он должен работать, поскольку океан холоднее космоса.

Строго говоря, это не так: все знают, что в космосе очень холодно. Причина, по которой космический корабль может перегреться, заключается в том, что космическое пространство не так хорошо отводит тепло, как вода, так что температура на лодке в космосе поднимается быстрее, чем на лодке в океане.

Но если вы хотите еще большего педантизма, то знайте: это все-таки правда. Океан холоднее космоса.

Межзведное пространство действительно очень холодное, но зона поблизости от Солнца – и вблизи Земли – на самом деле очень горячая! Причина, по которой кажется, что это не так, связана с тем, что в космосе наше понятие температуры не работает. Космос кажется холодным потому, что он такой пустой.

Температура – это мера средней кинетической энергии молекул. В космосе у каждой отдельной молекулы кинетическая энергия в среднем высокая, но этих молекул так мало, что они на вас просто не влияют.

Когда я был ребенком, у отца в подвале была мастерская, и я помню, как он работал на точильном станке. Когда металл касался шлифовального круга, с него летел сноп искр, осыпавших руки и одежду отца. Я не мог понять, почему это не причиняло ему вреда – в конце концов, эти сияющие искры были раскалены до нескольких тысяч градусов.



Позднее я узнал, что искры не причиняли отцу вреда потому, что были крохотными, и ничтожное количества тепла, которое в них было, тело человека могло поглотить без вреда для себя, при этом нагревался лишь крохотный кусочек кожи.

Горячие молекулы в космосе подобны искрам в мастерской моего отца – они могут быть горячими или холодными, но они в любом случае такие маленькие, что прикосновение к ним не особенно изменит температуру вашего тела[37]. Ваше нагревание и остывание связано с тем, сколько тепла вы производите и как быстро оно уходит в вакуум.

В отсутствие окружающего воздуха, который отражал бы назад излучаемое вами тепло, вы будете терять это тепло куда быстрее, чем обычно. С другой стороны, в отсутствие воздуха, который оттягивал бы тепло с поверхности тела, вы будет терять не так много тепла за счет конвекции. Для большинства космических кораблей с людьми на борту последнее обстоятельство имеет большее значение. Самая сложная задача не в том, чтобы сохранить тепло, а в том, чтобы не слишком сильно разогреться.

Атомная подводная лодка, очевидно, способна поддерживать внутри корпуса температуру, пригодную для жизни, даже когда внешний корпус охлаждается в океане до 4 °C. Однако если корпус должен поддерживать ту же температуру, находясь в открытом космосе, задача усложняется: в тени Земли подлодка будет терять тепло со скоростью примерно в 6 мегаватт. Это больше, чем 20 киловатт, которые отдает команда, и даже больше, чем несколько сотен киловатт тепла, которая субмарина будет получать, выйдя из тени и оказавшись на прямом свету Солнца, так что подводникам придется запустить реактор, чтобы сохранить тепло[38].

Чтобы спуститься с орбиты, субмарине нужно будет замедлиться настолько, чтобы столкнуться с атмосферой. Без ракет (ракетных двигателей) у нее нет возможности это сделать.



Хорошо-хорошо – строго говоря, на подводной лодке есть ракеты.



К сожалению, ракеты в пусковых шахтах лодки направлены таким образом, что не смогут подтолкнуть лодку. Ракета – реактивный снаряд, то есть у нее очень маленькая отдача. Когда ружье выбрасывает пулю, оно подталкивает пулю, разгоняя ее до нужной скорости. С ракетой не так – в ней просто поджигают топливо, и она летит. Выпущенные ракеты не смогут свести субмарину с орбиты.

А вот незапущенные могли бы.

Если баллистические ракеты современной атомной подлодки вытащить из пусковых шахт, перевернуть и снова установить в шахтах соплами вверх, каждая из них вполне могла бы изменить скорость субмарины на 4 метра в секунду.

Типичный маневр для схождения с орбиты требует примерно 100 м/с ?V (изменения скорости), а значит, 24 ракет класса «Трайдент», установленных на подлодке класса «Огайо», может как раз хватить, чтобы свести ее с орбиты.

И тогда, поскольку у подлодки нет абляционной теплозащиты и она аэродинамически неустойчива на сверхзвуковых скоростях, она неизбежно развалится в воздухе.



Если бы вы забились в правильный уголок субмарины и были пристегнуты к креслу, рассчитанному на ускорение, есть крохотный, крохотный, крохотный шанс, что вы пережили бы стремительный спуск. Но тогда вам пришлось бы выпрыгнуть с парашютом до того, как обломки вместе с вами обрушатся на землю.



Если вы как-нибудь попытаетесь это сделать (хотя я вам не рекомендую), у меня есть один невероятно полезный совет.

Не забудьте отключить детонаторы ракет.


Раздел коротких ответов

ВОПРОС: А что, если бы мой принтер мог печатать настоящие деньги? Как бы это повлияло на мировую экономику?

– Дерек О’Брайан

ОТВЕТ: На листе бумаги А4 можно уместить четыре стодолларовые купюры.



Если ваш принтер печатает одну страницу (с обеих сторон) в минуту, в цвете и высоком разрешении, получится 210 миллионов долларов в год. Этого достаточно, чтобы сделать вас очень богатым, но недостаточно, чтобы нанести урон мировой экономике. Поскольку в обращении одновременно находятся 7,8 миллиарда стодолларовых купюр, а продолжительность жизни купюры составляет примерно 90 месяцев, в год приходится допечатывать около миллиарда купюр. Ваши дополнительные два миллиона бумажек в год вряд ли смогут что-то изменить.

ВОПРОС: А что, если взорвать атомную бомбу в эпицентре урагана? Будет ли ураган мгновенно уничтожен?

– Руперт Бейнбридж (и сотни других людей)

ОТВЕТ: Этот вопрос задают часто. Как выясняется, Национальное управление океанических и атмосферных исследований, частью которого является Национальный центр по изучению ураганов, об этом тоже частенько спрашивают – так часто, что они даже опубликовали официальный ответ.

Советую прочитать его целиком[39], но думаю, что достаточно и последней фразы первого абзаца:

«Совершенно очевидно, что это не лучшая идея».

Меня радует мысль, что подразделение правительства США опубликовало свое официальное мнение по поводу возможности ядерной бомбардировки ураганов.

ВОПРОС: А что, если бы все мы установили маленькие турбинные генераторы на водосточных трубах своих домов и офисов? Сколько энергии мы смогли бы выработать? Получилось бы когда-нибудь выработать столько энергии, чтобы установка генераторов окупилась?

– Дэмиен

ОТВЕТ: В очень дождливой местности, например на Юго-Восточной Аляске, может выпадать 4 м осадков в год, и водяные турбины могут быть весьма эффективны. Если площадь дома 1500 кв. футов (140 м?) и его водосточные трубы будут начинаться в пяти метрах над землей, он выработает меньше одного ватта энергии от дождя, и максимальная экономия энергии составит:



Самый дождливый час в истории наблюдений, согласно данным, имеющимся на 2014 год, зафиксирован в 1947 году в Холте, штат Миссури, где за 42 минуты выпало около 30 см осадков. За эти 42 минуты наш гипотетический дом мог бы выработать до 800 ватт электричества, и этого было бы достаточно, чтобы работали все электрические приборы и бытовая техника в доме. Все остальное время мощность даже близко не подойдет к этой отметке.

Если установка генератора стоит 100$, то жители самого дождливого места в США – города Кечикан на Аляске – смогли бы окупить его стоимость меньше чем за столетие.

ВОПРОС: А что, если дать каждой звезде во Вселенной уникальное имя длиной в одно слово? Насколько длинными получатся названия, если использовать только удобочитаемые сочетания букв?

– Шеймус Джонсон

ОТВЕТ: Во Вселенной около 300 000 000 000 000 000 000 000 звезд[40]. Если считать, что слово удобочитаемо, если в нем чередуются гласные и согласные (для создания удобочитаемых слов есть способы и получше, но для условных расчетов этот подойдет), то каждая новая пара гласный+согласный, которую вы будете добавлять к имени, позволит вам дать имена 105 новым звездам (в латинском алфавите 21 согласных и 5 гласных букв). Поскольку информационная плотность цифр и букв близка – 100 вероятностей на знак, – можно предположить, что в самых длинных названиях будет столько же знаков, сколько в числе, обозначающем количество звезд.



Обожаю делать расчеты, которые включают в себя подсчет количества знаков в числе, записанном на бумаге (что, по сути, представляет собой просто логарифмические вычисления). И это работает – хотя на первый взгляд кажется, что это совершенно неправильно.

ВОПРОС: Иногда я езжу в школу на велосипеде. Зимой это не очень-то приятно, потому что очень холодно. С какой скоростью мне пришлось бы ехать, чтобы согреться до температуры космического корабля, входящего в атмосферу?

– Дэвид Ней

ОТВЕТ: Космический корабль, входящий в атмосферу, нагревается, поскольку сжимается находящийся перед ним воздух (а не за счет трения о воздух, как принято считать).

Чтобы повысить температуру слоя воздуха перед вашим телом на 20 °С (скажем, от нуля до комнатной температуры), нужно ехать со скоростью 200 м/с.

Самые быстрые наземные средства передвижения над уровнем моря, приводимые в движение мускульной силой человека, – это лигерады (лежачие велосипеды) с аэродинамическими обтекателями. Их максимальная скорость – около 40 м/с, скорость, на которой человек едва-едва развивает тягу, достаточную для преодоления возникающего лобового сопротивления воздуха.

Поскольку лобовое сопротивление пропорционально квадрату скорости, будет сложно преодолеть этот предел. Чтобы ехать на велосипеде со скоростью 200 м/с, потребуется примерно в 25 раз больше сил, чем если вы разгоняетесь до 40 м/с.

На этих скоростях не нужно волноваться по поводу температуры окружающей среды. Быстрый расчет показывает, что, если бы ваше тело выполняло такой объем нагрузок, его температура достигла бы смертельного уровня за несколько секунд.

ВОПРОС: Сколько места физически занимает Интернет?

– Макс Л.

ОТВЕТ: Есть много способов оценки объема информации, которая хранится в Интернете, но есть один любопытный вариант: просто посмотреть, какой объем занимают носители информации, которые мы (как вид в целом) покупаем.

Электронная промышленность производит около 650 миллионов жестких дисков в год. Если большая их часть – диски 3,5 дюйма, то в секунду производится восемь литров (2 галлона) жестких дисков.

А значит, все жесткие диски, произведенные за последние годы (при том, что объемы их производства постоянно растут), едва заполнят один большой нефтеналивной танкер. Так что, если считать таким образом, Интернет занимает меньше места, чем танкер.

ВОПРОС: А что, если прикрепить взрывчатку С-4 к бумерангу? Будет ли такое оружие эффективным или эта идея и в самом деле такая глупая, как выглядит?

– Чэд Мачевски

ОТВЕТ: Оставим в стороне вопросы аэродинамики – мне просто интересно: а какое тактическое преимущество вы предполагаете получить в том случае, если промахнетесь и бумеранг со взрывчаткой прилетит обратно?


Все, что вы хотели знать про молнию…

Прежде чем продолжить, я хочу еще раз напомнить: я не эксперт по безопасности во время грозы.

Я рисую картинки в Интернете. Я люблю смотреть, когда что-нибудь горит или взрывается, а значит, не всегда могу действовать в ваших интересах. Настоящие специалисты по безопасности во время грозы – это ребята из Национальной погодной службы США.

Ладно. С этим разобрались…

Чтобы ответить на нижеследующие вопросы, нам нужно понять, куда с наибольшей вероятностью ударит молния. Чтобы узнать это, есть отличный способ, и я вам прежде всего о нем расскажу: прикатите откуда-нибудь воображаемый[41] шар диаметром 60 м, установите его на местности и обратите особое внимание на точку, в которой он касается земли.

В этом разделе я отвечу на несколько разных вопросов, касающихся молнии. Говорят, что молния бьет в самую высокую точку на местности. Это одно из тех раздражающих своей неточностью утверждений, которые сразу вызывают массу вопросов. Самую высокую – в пределах какого радиуса? Я хочу сказать, что не все же молнии бьют в Эверест. Но будет ли молния выбирать самого высокого человека в толпе? Самый высокий человек, которого я знаю, это, вероятно, канадский писатель, программист и автор комиксов Райан Норт. Стоит ли мне стоять рядом с ним во время грозы – или это небезопасно? И вообще – стоит ли мне стоять рядом с Райаном из каких бы то ни было соображений?…

Наверное, мне все-таки стоит отвечать на вопросы, а не задавать их.

Так как же молния выбирает цель?

Удар молнии начинается с ветвящегося электрического заряда – «лидера молнии», который спускается с тучи. Он идет вниз со скоростью, составляющей десятки или даже сотни километров в секунду и преодолевает несколько километров до земли за несколько десятков миллисекунд.

У лидера довольно низкая сила тока – порядка 200 А. Этого достаточно, чтобы вас убить, но это ничто по сравнению с тем, что будет дальше. После того как лидер достигает земли, электрический заряд облака и земли выравнивается, высвобождая где-то около 20 000 А. Это и есть та ослепляющая вспышка, которую мы видим. Она поднимается обратно со скоростью, сопоставимой со скоростью света, преодолевая расстояние меньше чем за миллисекунду[42].

То место на земле, куда, как мы видим, «бьет» молния, – это та точка, где лидер впервые коснулся поверхности. Лидер спускается через атмосферу маленькими скачками, прокладывая себе путь к (обычно) положительно заряженной поверхности земли. Однако в момент, когда лидер «решает», куда ему устремиться дальше, он «чувствует» этот заряд только в радиусе нескольких десятков метров от себя. Если в этом радиусе имеется что-то, соединенное с положительно заряженной землей, заряд немедленно устремляется туда, в остальных случаях он перемещается в более или менее случайном направлении, и процесс повторяется.

Вот тут мы и используем 60-метровую воображаемую сферу. Это способ определить, какие точки лидер почувствует в первую очередь – точки, на которые он может перепрыгнуть в следующий (последний) шаг.



Чтобы разобраться, куда может ударить молния, давайте покатим по земле нашу воображаемую сферу диаметром 60 м[43]. Огромный шар перекатывается через деревья и здания, однако не проходит сквозь них (и не подминает их). Места, где поверхность шара соприкасается с чем-либо – с верхушкой дерева, столбом изгороди, игроком на поле для гольфа, – и есть вероятные мишени молнии.

Это значит, что можно рассчитать «тень» молнии вокруг объекта высотой h на плоской поверхности.



Тень – это зона, в которой лидер ударит в самый высокий объект, а не в землю вокруг него:



Учтите, это не значит, что в тени вы в безопасности – часто даже наоборот. После того как заряд ударил в высокий объект, он уходит в землю. Если вы касаетесь земли рядом, он может пройти через ваше тело. Из 28 людей, убитых молнией в США в 2012 году, 13 стояли под деревом или рядом с ним.

А теперь давайте посмотрим, каковы будут возможные пути молнии в сценариях, которые описаны нижеследующими вопросами.

ВОПРОС: А что, если искупаться в бассейне в грозу? Опасно ли это?

ОТВЕТ: Достаточно опасно. Вода представляет собой проводник, но это не самая большая проблема: хуже то, что, когда вы плаваете, ваша голова выступает над большой плоской поверхностью. Но даже если молния ударит в воду рядом с вами, это все равно плохо. 20 000 А распространятся по воде во все стороны, в основном, по ее поверхности, но какой именно разряд вы получите в той или иной точке бассейна, рассчитать сложно.

Я предполагаю, что вы будете в довольно серьезной опасности, если находитесь не дальше десятка метров от места удара – и даже дальше, если вода в бассейне пресная, поскольку разряд будет просто счастлив «срезать» путь через вас.

А что произойдет, если вы будете принимать душ в тот момент, когда в вас ударит молния? Или стоять под водопадом?

Брызги не будут представлять опасности – это просто капли воды, висящие в воздухе. Ванна у вас под ногами и лужица воды в ней, имеющая прямой контакт с металлической выпускной трубой, – вот где настоящая опасность.


ВОПРОС: А что, если в лодку или в самолет, в которых вы находитесь, попала молния? А в подводную лодку?


ОТВЕТ: Лодка без кабины опасна примерно в той же степени, что и поле для гольфа. Лодка, оборудованная закрытой кабиной и системой защиты от молнии, примерно так же безопасна, как автомобиль. Подлодка безопасна примерно так же, как подводный сейф (не стоит путать подводный сейф с сейфом на подводной лодке, сейф на подводной лодке куда безопаснее, чем подводный сейф). Современные авиалайнеры, как правило, более или менее нормально переживают удар молнии – они сконструированы так, чтобы молния не повредила ни пассажирам, ни электронике самолета.

ВОПРОС: А что, если бы вы меняли лампочку на вершине радиовышки в тот момент, когда ударила молния? Или делали обратное сальто? Или стояли на графитовом поле? Или смотрели прямо на молнию?

ОТВЕТ:


ВОПРОС: А что, если бы молния попала в летящую пулю?

ОТВЕТ: Пуля не повлияет на траекторию молнии. Пришлось бы как-то рассчитать момент выстрела так, чтобы пуля оказалась в середине разряда молнии, когда случится обратный разряд.

Центр разряда молнии не больше нескольких см в диаметре. Пуля, выпущенная из АК-47, имеет длину примерно в 26 мм и движется со скоростью 700 мм в миллисекунду.

Пуля сделана из свинца в медной оболочке. Медь – замечательный проводник электричества, и немалая часть 20 000 А могла бы пройти через нее.

Как ни странно, пуля перенесла бы это довольно хорошо. Если бы она не двигалась, поток быстро нагрел бы металл и расплавил его. Но поскольку она будет быстро двигаться, то выйдет из потока прежде, чем тот нагреет ее больше чем на несколько градусов. Пуля продолжила бы лететь в цель, практически не изменив траекторию. Возникнут довольно любопытные электромагнитные силы, созданные магнитным полем вокруг разряда и его прохождением через пулю, но ни одна из них, согласно моим расчетам, сильно не изменит картины.


ВОПРОС: А что, если бы вы обновляли BIOS на своем компьютере во время грозы и в вас бы ударила молния?

ОТВЕТ:


Странные (и тревожные) вопросы из почтового ящика «А что, если?»

ВОПРОС: Можно ли остановить извержение вулкана, разместив под ним ядерную бомбу?

– Томаш Грушка

ВОПРОС: Мой друг уверен, что звук распространяется в космосе. Это ведь не так, правда?

– Аарон Смит

Мозг и компьютер

ВОПРОС: А что, если бы все человечество бросило заниматься текущими делами и занялось вычислениями? Какой вычислительной мощности можно было бы достичь? Можно ли сравнить ее с производительностью современных компьютеров и смартфонов?

– Матеуш Кнорпс

ОТВЕТ: С одной стороны, люди и компьютеры думают совершенно по-разному, так что сравнивать их – все равно что сравнивать яблоки с апельсинами



Но с другой стороны, яблоки, конечно, лучше[44]. Давайте попробуем напрямую сравнить человека и компьютер, выполняющих одно и то же задание.



Несложно придумать задачу, которую один-единственный человек решит быстрее, чем все компьютеры мира (хотя с каждым днем это становится все труднее). Например, люди все еще лучше, чем компьютеры, угадывают, что именно произошло на картинке:


Чтобы проверить эту теорию, я послал эту картинку моей матери и спросил, что, по ее мнению, произошло. Она немедленно ответила: «Ребенок разбил вазу, а кот изучает обстановку»[45].

Она мудро отвергла альтернативные версии, включая следующие:

• вазу разбил кот;

• кот выпрыгнул из вазы на ребенка;

• кот гнался за ребенком, а тот попытался взобраться на комод при помощи веревки;

• в дом забрался дикий кот, и кто-то кинул в него вазу;

• в вазе была спрятана мумия кота, но она ожила, когда ребенок коснулся ее волшебной веревкой;

• веревка, на которой висела ваза, оборвалась, и кот пытается собрать осколки;

• ваза взорвалась, и на звук прибежали ребенок и кот. Ребенок надел шляпу для защиты от последующих взрывов;

• ребенок и кот ловили змею. Ребенок наконец поймал ее и завязал узлом.

Все компьютеры мира не смогли бы вычислить верный ответ быстрее, чем одна моя мать. Но это потому, что компьютеры не запрограммированы определять подобные вещи[46], тогда как наш мозг в течение миллионов лет эволюции тренировался, пытаясь определить, что делают другие мозги неподалеку (и зачем они это делают).

Так что мы могли бы легко выбрать задачку, которая создаст преимущество для человека, но это не очень честно: компьютеры ограничены тем, как мы их программируем, так что мы по определению уже обладаем преимуществом.

Давайте лучше посмотрим, как мы можем потягаться с ними на их территории.

Сложность микрочипов

Вместо того чтобы придумывать новое задание, мы просто применим к людям те же тесты на производительность, которые применяем к компьютерам. Обычно это операции с числами с плавающей запятой, сохранение и вызов чисел, манипулирование буквенными последовательностями и базовые логические построения.

Специалист по искусственному интеллекту Ханс Моравек утверждает, что человек, выполняющий тесты на производительность для компьютерных чипов вручную, используя карандаш и бумагу, может выполнить одно задание в полторы минуты[47].

Тогда получается, что процессор в смартфоне средней мощности может производить вычисления в 70 раз быстрее, чем все население Земли. Для современного чипа персонального компьютера эта цифра равняется 1500.



Итак, в каком году обычный персональный компьютер обошел совокупную вычислительную способность человечества?

1994

В 1992 году численность населения Земли составляла 5,5 миллиарда человек, то есть их общая вычислительная способность, согласно нашему тесту, составляла бы около 65 MIPS (миллионов инструкций в секунду).

В том же году корпорация «Интел» выпустила популярный процессор 486DX, который в своей базовой конфигурации имел производительность в 55–60 MIPS. К 1994 году новые «пентиумы» производства «Интел» достигали значений 70 и 80 MIPS, оставив человечество далеко позади.

Вы можете возразить, что мы несправедливы по отношению к компьютерам. В конце концов, мы все время выставляем один компьютер против всего человечества. А как насчет всего человечества против всех компьютеров?

Это сложно рассчитать. Мы можем легко найти результаты тестов производительности для разных компьютеров, но как рассчитать количество инструкций в секунду для, скажем, чипа, установленного в игрушке Furby?



Большая часть компьютеров в мире построена на микрочипах, которые не предназначены для подобных тестов. Если предположить, что все люди на земле обучены делать вычисления для тестов производительности, сколько усилий нам нужно будет потратить, чтобы модифицировать каждый чип для выполнения этих тестов?

Вместо этого можно оценить общую мощность всех вычислительных машин мира, подсчитав количество транзисторов. Оказывается, процессоры 1980-х и современные процессоры имеют близкое соотношение транзисторов к MIPS – примерно 30 транзисторов на инструкцию в секунду.

Статья Гордона Мура (автора «закона Мура») дает оценку общей численности транзисторов, производимых ежегодно с 1950-х. График выглядит примерно так:



Используя наше соотношение, мы можем перевести число транзисторов в общую вычислительную мощность. Получается, что современный ноутбук, тест производительности которого дает результаты в десятки тысяч MIPS, обладает большей вычислительной мощностью, чем все компьютеры мира в 1965-м. Исходя из этой оценки, год, в котором общая вычислительная сила компьютеров превзошла общую вычислительную силу человечества, – это 1977-й.

Сложность нейронов

Конечно же, заставлять людей выполнять тесты на производительность при помощи бумаги и карандаша – невероятно глупый способ определять их вычислительную мощность. Если измерять сложность, наши мозги гораздо изощреннее, чем любой суперкомпьютер, правда же?

Правда. По большей части.

Существуют проекты, в которых пытаются использовать суперкомпьютеры, чтобы полностью симулировать мозг на уровне отдельных синапсов[48]. Если посмотреть, сколько процессоров и времени требуют эти симуляции, можно получить количество транзисторов, требующихся для того, чтобы сравняться в сложности с человеческим мозгом.

Результаты тестирования японского суперкомпьютера Kcomputer в 2013 году показывают, что для «создания» одного мозга понадобится 1015 транзисторов[49]. По этой оценке, только в 1988 году общее количество логических цепей в мире сравнялось со сложностью одного мозга… и общая сложность всех цепей все еще ничтожна по сравнению с общей сложностью всех мозгов. По оценкам, основанным на законе Мура, и исходя из наших расчетов, компьютеры не обойдут человечество раньше 2036 года[50].

Почему эти тесты нелепы

Два описанных способа оценки мозга представляют собой две крайности.

Первый, тест Dhrystone, сделанный на бумаге, требует, чтобы человек вручную симулировал отдельные операции, которые выполняет компьютерный чип, после чего выясняется, что быстродействие человека составляет примерно 0,01 MIPS.

Второй тест, в котором деятельность нейронов человеческого мозга симулируется при помощи суперкомпьютера, показывает, что быстродействие человека составляет примерно 50 000 000 000 MIPS.

Тогда уж лучше совместить эти способы. Это некоторым странным образом даже логично. Если мы предполагаем, что наши компьютерные программы столь же неэффективно симулируют деятельность человеческого мозга, как и человеческие мозг симулирует работу компьютерных чипов, тогда, возможно, более точной оценкой производительности мозга будет среднее геометрическое этих двух чисел.



Итоговый результат показывает, что быстродействие человеческого мозга составляет порядка 30 000 MIPS – примерно вровень с компьютером, на котором я сейчас печатаю эти слова. А еще это значит, что уровень сложности всех компьютеров мира превысил общую неврологическую сложность человечества в 2004 году.

Муравьи

В своей статье «Закону Мура – 40 лет» Гордон Мур делает интересное наблюдение. Он указывает, что, согласно данным биолога Э. Уилсона, в мире от 1015 до 1016 муравьев. Для сравнения, в 2014 году в мире было примерно 1020 транзисторов, или десять тысяч транзисторов на одного муравья.

Мозг муравья может содержать четверть миллиона нейронов и тысячи синапсов на каждый нейрон, из чего следует, что общая сложность мозгов всех муравьев аналогична общей сложности мОзгов всего человечества.

Так что не стоит слишком волноваться, что компьютеры догонят нас по сложности. В конце концов, мы догнали муравьев, а их это мало беспокоит. Конечно, все выглядит так, словно мы захватили власть на планете, но если бы мне пришлось делать ставки, кто из нас все еще будет существовать спустя миллион лет – приматы, компьютеры или муравьи, я знаю, на кого бы поставил.


Планета Маленького принца

ВОПРОС: А что, если бы какой-нибудь астероид был очень маленьким, но сверхмассивным – можно ли было бы на нем и в самом деле жить, как Маленький принц?

– Саманта Харпер

Ты съела мою розу? Может быть.


ОТВЕТ: «Маленький принц» Антуана де Сент-Экзюпери – это история о страннике с дальнего астероида. Это простая, грустная, проникновенная и запоминающаяся история. Принято думать, что это детская книжка, но сейчас сложно определить, для какой аудитории она на самом деле предназначалась. В любом случае свою аудиторию она нашла – это одна из самых продающихся книг в истории.



«Маленький принц» был опубликован в 1943 году. В то время мы еще не знали точно, как выглядят астероиды, так что про них было особенно интересно писать. Даже лучшие телескопы позволяли увидеть самые крупные астероиды только в виде светящихся точек. Отсюда, собственно, их название – слово астероид означает «подобный звезде».

Наше первое представление о том, как выглядят астероиды, сформировалось в 1971 году, когда космический зонд «Маринер-9» слетал на Марс и сфотографировал по дороге спутники Марса Фобос и Деймос. Наше современное представление об астероиде как о картофелине, усеянной кратерами, основано именно на этих фотографиях, ведь Фобос и Деймос считаются застрявшими на орбите Марса астероидами.



До 1970-х годов в научной фантастике обычно предполагалось, что маленькие астероиды должны быть круглыми, как планеты. Экзюпери в «Маленьком принце» пошел еще дальше, представив астероид маленькой планетой с гравитацией, воздухом и розой. Нет смысла пытаться критиковать это с научной точки зрения, потому что 1) это история не про астероиды и 2) она начинается с притчи о том, как глупы взрослые, воспринимающие все слишком буквально.

Вместо того чтобы с помощью науки критиковать сказку, задумаемся, какие любопытные подробности мы могли бы добавить. Если бы действительно существовал сверхплотный астероид с достаточной поверхностной гравитацией, чтобы по нему можно было ходить, у него были бы весьма интересные свойства.

Если бы астероид имел радиус 1,75 м, то для того, чтобы гравитация у его поверхности была приближена к земной, ему нужно было бы иметь массу около 500 миллионов тонн. Это примерно равно общей массе всех людей на Земле.

Стоя на поверхности астероида, мы испытывали бы влияние приливных сил. Ноги ощущались бы более тяжелыми, чем голова, и все это создавало ощущение, словно нас слегка вытягивают: как будто вы растягиваетесь на резиновом гимнастическом шаре или лежите на карусели головой к центру.



Вторая космическая скорость на поверхности будет составлять примерно 5 м в секунду. Это медленнее, чем спринтерский забег, но все же довольно быстро. В принципе, если вы не способны забросить мяч в баскетбольную корзину, вы не сможете выбраться с этого астероида, просто подпрыгнув.



Однако странность второй космической скорости (скорости освобождения) заключается в том, что неважно, в какую сторону вы направляетесь. Если вы разгонитесь быстрее второй космической, то, направляясь в любую сторону (не считая направления к центру планеты), вы выйдете за пределы гравитации. Это значит, что вы сможете покинуть астероид, разбежавшись горизонтально и продолжая бежать по прямой, когда закругляющаяся поверхность планеты уйдет у вас из-под ног.



Если вы не будете двигаться достаточно быстро, чтобы выбраться за пределы действия гравитации астероида, вы выйдете на орбиту вокруг него. Ваша скорость составит примерно 3 метра в секунду – типичная скорость утренней пробежки.



Однако это будет очень странная орбита.

Действие приливных сил будет весьма разнообразным. Если вы протянете руку к поверхности планеты, ее начнет притягивать с гораздо большей силой, чем остальные части вашего тела. А если вы коснетесь поверхности одной рукой, то все остальные части тела гравитация будет толкать вверх, и вам будет казаться, что они теперь весят меньше. По сути дела, каждая часть вашего тела будет пытаться выйти на свою собственную орбиту.

Крупный объект на орбите под влиянием подобных сил – например, спутник – обычно распадается на кольца. С вами этого не произойдет, но ваша орбита будет весьма сложной.

Подобные орбиты были описаны в одной статье Раду Руджеску и Даниеле Мортари. Расчеты этих ученых показали, что крупные, удлиненные спутники вращаются вокруг центрального объекта по необычным маршрутам. Даже их центр тяжести движется не по традиционной эллиптической орбите – некоторые из них выходят на пятиугольную орбиту, другие вращаются хаотично и в конце концов врезаются в свою планету[51].

Подобный анализ мог бы на самом деле иметь практическое применение. В течение многих лет поступали предложения использовать длинные вращающиеся тросы для того, чтобы перемещать грузы в гравитационное поле или из него, что-то вроде свободно плавающего в пространстве космического лифта. Подобные тросы могли бы доставлять грузы на поверхность Луны, или поднимать их с нее, или подхватывать космические корабли с границы атмосферы Земли. Нестабильность орбит многих тросов – главная проблема такого проекта.

Что касается обитателей нашего сверхплотного астероида, им пришлось бы быть осторожными – при слишком быстром беге возникает серьезный риск выйти на орбиту, где вы сделаете сальто и вас укачает.

К счастью, вертикальные прыжки не представляли бы сложности.


Болельщики баскетбольной команды «Кливленд кавальерс» с уважением относятся к французской детской литературе, однако разочарованы решением Маленького Принца подписать контракт с клубом «Майами Хит».

Стейк из поднебесья

ВОПРОС: С какой высоты нужно уронить стейк, чтобы он упал на землю уже поджаренным?

– Алекс Лахей

ОТВЕТ: Надеюсь, вы любите стейки с кровью. И вам, возможно, придется разморозить стейк после того, как вы подберете его с земли.

Предметы очень сильно разогреваются, когда возвращаются из космоса. Когда объект входит в атмосферу, воздух не может убраться с его пути достаточно быстро и сжимается перед объектом – а сжатие воздуха приводит к его нагреванию. Вы почувствует нагрев от сжатия примерно тогда, когда скорость объекта превысит 2 М (вот почему передние кромки крыльев «Конкорда» были покрыты теплостойким защитным материалом).

Когда австрийский парашютист Феликс Баумгартнер прыгнул с высоты почти 39 км, он развил скорость в 1 М на высоте около 30 км. Этого достаточно, чтобы нагреть воздух на несколько градусов, но температура окружающего воздуха была настолько низкой, что перемены Баумгартнер не почувствовал (в верхней точке прыжка было около –40°, та волшебная точка, в которой не нужно уточнять, говорим ли мы о шкале Фаренгейта или Цельсия, – тут они совпадают).

Чтобы получить ответ на вопрос Алекса, я решил провести серию симуляций падения стейка с разных высот.

Стейк весом 8 унций (228 г) размерами и формой напоминает хоккейную шайбу, так что я рассчитывал коэффициент лобового сопротивления моего стейка, исходя из данных, приведенных на стр. 74 книги «Физика хоккея» (эти данные автор, Ален Хаше, получил собственноручно при помощи лабораторного оборудования)[52]. Стейк, конечно, не хоккейная шайба, но в конце концов выяснилось, что погрешность в расчете коэффициента сопротивления не слишком сильно влияет на результат.

Поскольку для ответа на все ваши вопросы мне часто приходится анализировать поведение необычных объектов в экстремальных физических условиях, то подходящие данные подчас можно найти только в отчетах о военных исследованиях, проводившихся в США во времена холодной войны (похоже, правительство США осыпало деньгами все проекты, хотя бы приблизительно похожие на разработки в сфере вооружений).

Чтобы оценить, насколько воздух нагреет падающий стейк, я посмотрел исследования, посвященные нагреву носового конуса межконтинентальных ракет в момент их возвращения в атмосферу. Наиболее полезными оказались «Прогнозы аэродинамического нагрева обтекателей тактических ракет» и «Температурные диаграммы для летательных аппаратов, входящих в плотные слои атмосферы».

Под конец я должен был выяснить, насколько быстро тепло распространяется по стейку. Я начал с того, что почитал статьи специалистов пищевой промышленности, которые описывали теплопроводность различных кусков мяса. Мне потребовалось некоторое время, пока я не сообразил, что есть гораздо более простой способ узнать, какое время и температура требуются для наиболее эффективного разогрева разных слоев стейка – заглянуть в поваренную книгу.

Отличная книга Джеффа Поттера «Кулинария для гиков» включает в себя в том числе и отличное введение в науку приготовления мяса, где объясняется, какие диапазоны температур хороши для стейка и почему. «Наука хорошей кулинарии» тоже оказалась полезной.

Собрав воедино все данные, я выяснил, что стейк будет быстро ускоряться, пока не достигнет высоты примерно в 30–50 км, после чего воздух станет достаточно плотным, чтобы снова замедлить падение куска этого куска мяса.

Скорость падающего стейка будет постепенно снижаться по мере возрастания плотности воздуха. Вне зависимости от того, с какой скоростью он летел, когда достиг нижних слоев атмосферы, он быстро замедлится до равновесной скорости – то есть такой, при которой сила сопротивления воздуха будет равна силе гравитации. Вне зависимости от того, с какой высоты изначально был сброшен стейк, ему потребуется шесть или семь минут, чтобы пролететь последние 25 км до земли.

На протяжении большей части этих 25 км температура окружающего воздуха будет ниже нуля, а значит, стейк проведет шесть или семь минут под воздействием ураганного ледяного встречного ветра. Даже если этот кусок поджарится за время падения, то когда он приземлится, его, возможно, потребуется разморозить.

В момент, когда стейк наконец достигнет земли, его скорость будет равновесной – около 30 м/с. Чтобы представить себе эту скорость, вообразите, что стейк был брошен питчером Национальной бейсбольной лиги. Если стейк в этот момент будет хотя бы отчасти заморожен, он легко может расколоться. Однако если он приземлится в воду, в грязь или в палую листву, то вероятно, все будет в порядке[53].

Стейк, который уронили с высоты 39 км, в отличие от Феликса Баумгартнера, скорее всего, не преодолеет звуковой барьер. Он также и не нагреется до сколько-нибудь заметной степени. В конце концов, и скафандр Феликса не был обожжен, когда он приземлился.



Стейк, вероятно, сможет пережить преодоление звукового барьера. Не считая опыта Феликса Баумгартнера, известны истории о пилотах, которые катапультировались на сверхзвуковых скоростях и оставались в живых.

Чтобы преодолеть звуковой барьер, нужно уронить стейк с высоты 50 км. Но этого все еще недостаточно, чтобы поджарить его.

Для этого надо подняться еще выше.

Если уронить стейк с высоты 70 км, он будет падать достаточно быстро, чтобы на некоторое время разогреться от столкновения с воздухом до 176 °С. К сожалению, разреженный воздух на такой высоте будет горячим не больше минуты, и каждый, у кого есть хоть какой-то опыт готовки, скажет вам, что кусок мяса, который поместили в духовку, разогретую до 176 °С, всего на 60 секунд, ни за что не прожарится.

При падении с высоты 100 км – что формально считается границей между атмосферой и космическим пространством – ситуация не намного улучшится. Стейк на полторы минуты разгонится на более чем 2 М, и его поверхность, скорее всего, подрумянится, но жар слишком быстро сменится ледяным дыханием стратосферы, чтобы мясо могло по-настоящему прожариться.

На сверх– и гиперзвуковых скоростях вокруг стейка сформируется ударная волна, которая поможет защитить его от ускоряющегося встречного ветра. Точная характеристика этого ударного фронта (и, следовательно, степень его влияния на стейк) зависит от того, как именно двести граммов сырого мяса ведут себя на сверхзвуковых скоростях. Я порылся в литературе, но не нашел соответствующих исследований.

Поэтому я могу лишь предположить, что на более низких скоростях возникнут определенные завихрения, которые заставят стейк беспорядочно кувыркаться, тогда как на гиперзвуковой скорости он будет сдавлен и примет форму полустабильного сфероида. Однако все это совершенно произвольные догадки. Если у кого-то из вас есть возможность поместить стейк в аэродинамическую трубу и смоделировать в ней гиперзвуковую скорость, чтобы получить более точные результаты, пожалуйста, пришлите мне потом видео.

Если уронить стейк с высоты 250 км, температура становится выше: 250 км – это окрестности околоземной орбиты (нижняя точка орбиты космического аппарата Гагарина была на 80 км ниже). Однако стейк, поскольку мы роняем его из неподвижного положения, двигается не так быстро, как объект, входящий в атмосферу с орбиты.

В этом сценарии стейк достигает максимальной скорости в 6 М, и внешняя его поверхность может приятно подрумяниться. Внутри мясо, увы, все еще сырое (и это еще при условии, что стейк не войдет в зону турбулентности и не разорвется на куски).

На бо?льших высотах жар будет вполне серьезным. Ударная волна перед стейком создаст температуру в несколько тысяч градусов (Фаренгейта или Цельсия – неважно). Проблема с такой температурой заключается в том, что жар полностью сожжет поверхность стейка, обуглив ее.



Обугливание – нормальное следствие того, что мясо находится в огне. При этом на гиперзвуковых скоростях хрупкий обуглившийся слой будет срываться встречным потоком воздуха, обнажая следующий слой мяса, который, в свою очередь, будет тоже обугливаться и сдуваться (в космических технологиях такой процесс выгорания поверхности называют абляцией).

Но даже на такой высоте жар все еще не будет столь продолжительным, чтобы стейк успел полностью прожариться[54]. Но мы можем пробовать все более и более высокие скорости или продлить время прожарки, роняя стейк с орбиты под разными углами.

Но если температура будет достаточно высокой в течение достаточно долгого времени, стейк постепенно будет уменьшаться, поскольку верхний слой будет снова и снова обугливаться и сдуваться. И если какая-то часть стейка все же доберется до земли, внутри мясо все еще будет сырым.

Вот почему нам надо уронить стейк не куда-нибудь, а на Питтсбург.

Как гласит одна история (вероятно, выдуманная), сталевары в Питтсбурге готовили стейки, выкладывая их на раскаленные металлические листы, вышедшие прямиком из литейного цеха. При этом поверхность стейка прожаривалась, а внутренняя часть оставалась сырой. Предположительно, отсюда и пошел термин «по-питтсбургски с кровью» (Pittsburgh rare).

Итак, вышвырните стейк из ракеты на орбите, отправьте поисковую команду его подобрать, отряхните его, разогрейте, срежьте обугленные части – и можете запускать в него зубы!

Только убедитесь, чтобы в нем не было сальмонеллы – не говоря уже о штамме «Андромеда»…

Забить вратаря в ворота

ВОПРОС: А что, если послать шайбу с такой силой, чтобы она забила вратаря в ворота?

– Том

ОТВЕТ: Это невозможно.

Проблема не столько в том, что ударить по шайбе с достаточной силой нелегко – в этой книге мы не интересуемся подобными ограничениями. Человек с клюшкой не может разогнать шайбу быстрее 50 м/с, но давайте предположим, что по шайбе бьет хоккейный робот, или электромагнитная катапульта, или сверхзвуковая газовая пушка.

Проблема, если вкратце, заключается в том, что вратарь – тяжелый, а шайба – нет. Голкипер в полной экипировке весит примерно в 600 раз больше, чем шайба. Даже самый быстрый прострел обладает меньшим импульсом, чем десятилетний ребенок, катающийся на коньках со скоростью порядка полутора километров в час.

Хоккеист может также обладать довольно сильным сцеплением с поверхностью. Игрок, несущийся по льду на полной скорости, может полностью затормозить за несколько метров, а это значит, что сцепление весьма сильное (это также означает, что, если хоккейную площадку медленно наклонять, то все игроки съедут на одну сторону лишь когда наклон достигнет 50 градусов; но это, конечно придется проверить экспериментально).

Благодаря изучению статистики столкновений, содержащейся в записях хоккейных матчей, и консультациям с одним знатоком хоккея, я прикинул, что шайба весом в 165 граммов должна будет двигаться со скоростью между 2–8 М, чтобы вбить вратаря спиной в ворота. Эта скорость должна быть больше, если вратарь был готов к удару, и может быть меньше, если шайба ударит его под углом снизу вверх.

Бросить объект со скоростью в 8 М – это само по себе не очень сложно. Один из лучших методов для этого – уже упомянутая газовая пушка, в которой, по сути дела, используется тот же принцип, что и в пневматическом пистолете.

Однако хоккейная шайба, летящая со скоростью 8 М, столкнется со множеством трудностей: во-первых, воздух перед ней будет сжиматься и быстро нагреваться. При этом шайба будет двигаться недостаточно быстро, чтобы ионизировать воздух и оставлять за собой, подобно метеору, сияющий след, но при этом достаточно быстро для того, чтобы ее поверхность начала плавиться или обугливаться.

Сопротивление воздуха довольно быстро замедлит шайбу, и даже если ее скорость в момент подачи составляла 8 М, то к моменту, когда шайба достигнет ворот, эта скорость уменьшится в несколько раз. Но даже на скорости 8 М шайба, скорее всего, не пройдет тело вратаря насквозь. Вместо этого она при столкновении с ним взорвется с силой большого фейерверка или небольшого заряда динамита.

Если вы похожи на меня, то, впервые увидев этот вопрос, наверное, представили себе, как шайба пробивает сквозное отверстие во вратаре, как это изображают в мультиках. Но мы просто не очень хорошо интуитивно оцениваем поведение разных материалов на сверхскоростях.

Возможно, самое точное представление о, том, что случится, вы получите, если изо всех сил запустите очень спелый помидор в торт с кремом.



Вот на что примерно это будет похоже.

Истребление простуды

ВОПРОС: А что, если бы все люди на планете не подходили близко друг к другу в течение пары недель? Вымерли бы за это время вирусы простуды?

– Сара Эварт

ОТВЕТ: А стоит ли оно того? Обычную простуду вызывают разнообразные вирусы, но наиболее частые ее виновники – риновирусы[55]. Эти вирусы оккупируют клетки в вашем носу и горле и используют их, чтобы порождать новые вирусы. Спустя несколько дней ваша иммунная система замечает вирусы и уничтожает их[56], но в среднем это происходит уже после того, как вы заразили еще одного человека[57]. Зато у вас на многие годы после выздоровления будет иммунитет к этому конкретному риновирусу.



Если бы Сара поместила нас всех в карантин, вирусы простуды, которыми мы заражены, не смогли бы перейти на новых хозяев. Но способна ли наша иммунная система уничтожить все вирусы в нашем теле до последнего?

До того как ответить на этот вопрос, оценим практические последствия подобного карантина. Мировой валовой продукт составляет более 72 миллиардов долларов в год, из чего следует, что перерыв в экономической деятельности на несколько недель будет стоить несколько миллиардов. Шок, который мировая экономическая система испытает от всемирной «паузы», может легко спровоцировать глобальный коллапс.



Всемирных запасов еды, возможно, хватит, чтобы обеспечить нас на четыре-пять недель карантина, но всю эту еду пришлось бы заранее поровну распределить. Откровенно говоря, не знаю, чем бы мне помог 20-дневный запас зерна, окажись я в полном одиночестве где-то посреди чистого поля.



С всемирным карантином связан и другой вопрос: насколько далеко друг от друга мы сможем разойтись? Мир велик [источник не указан], но и людей в нем немало [источник не указан].

Если разделить всю сушу на Земле поровну, на каждого придется чуть больше 2 га, так что ближайший человек будет находиться в 77 метрах.



Дистанция в 77 метров, вероятно, достаточна для предотвращения передачи риновирусов, но это решение будет иметь определенные последствия. Довольно значительная часть суши не слишком подходит для того, чтобы простоять там недель пять. Многим из нас[58] придется застрять в Сахаре, а еще кому-то[59] – в Антарктиде.

Более практичным – хотя необязательно более дешевым – решением будет выдать всем костюмы биологической защиты. Таким образом, мы могли бы перемещаться и общаться, и даже отчасти вести нормальную экономическую деятельность:



Но оставим в стороне практичность и обратимся собственно к вопросу Сары: поможет ли это?

Чтобы узнать ответ, я поговорил с профессором Иэном Маккеем, специалистом-вирусологом из Австралийского исследовательского центра инфекционных заболеваний университета Квинсленда.

Доктор Маккей сказал, что с чисто биологической точки зрения эта идея и в самом деле имеет смысл. Он добавил также, что иммунная система человека полностью уничтожает риновирусы и другие РНК-вирусы, ответственные за респираторные инфекции. Более того, похоже, что мы и другие животные не передаем друг другу риновирусы, а значит, нет животных, которые могли бы послужить резервуаром возбудителей человеческой простуды. Если риновирусам не хватает людей, между которыми они могли бы перемещаться, они вымирают.

Мы уже знаем, как это вымирание вирусов происходит в изолированных популяциях. На уединенном архипелаге Сент-Килда на северо-западе Шотландии в течение столетий проживала всего сотня человек. Жители островов, на которые заходило лишь несколько кораблей в год, периодически страдали от необычного синдрома, который они называли cnatan-na-gall – «кашель чужестранца». В течение нескольких столетий кашель, словно по часам, поражал весь остров всякий раз, когда прибывало новое судно.

Точная причина этих эпидемий неизвестна[60], но скорее всего за многие из них ответственны риновирусы. Каждый раз, когда прибывал корабль, он привозил новый штамм вируса. Этот штамм распространялся по острову, заражая буквально всех жителей. Спустя несколько недель все жители вырабатывали свежий иммунитет к этому штамму, и поскольку вирусу больше некуда было деться, он вымирал.



Подобное же очищение от вирусов, вероятно, произойдет и в любой другой маленькой и изолированной популяции – например в группе моряков, выживших после кораблекрушения.

Если бы все люди были изолированы друг от друга, сценарий островов Сент-Килда повторился бы в масштабах всего вида Homo sapiens. Спустя неделю-другую все мы вылечились бы от простуды, и у здоровой иммунной системы было бы достаточно времени, чтобы избавиться от вирусов.

К сожалению, есть одна проблема, и ее достаточно, чтобы разрушить весь наш план: не у всех из нас здоровый иммунитет.



У большинства людей риновирусы в организме будут полностью уничтожены примерно за десять дней. Все обстоит иначе у тех, у кого иммунная система серьезно ослаблена. Например, у пациентов с пересаженными органами, чей иммунитет подавлялся искусственным путем, обычные инфекции, включая риновирусные, могут длиться неделями, месяцами и даже годами.

Сравнительно небольшая группа людей с ослабленным иммунитетом послужит надежным убежищем для риновирусов. Шансов уничтожить эти вирусы немного: вирусу достаточно выжить всего в нескольких носителях, чтобы распространиться и вновь захватить мир.

Так что план Сары не только вызовет масштабный кризис цивилизации, но и не уничтожит риновирусы[61]. Однако это, может быть, к лучшему!

Хотя в простуде мало радости, ее отсутствие может оказаться еще хуже. В своей книге «Планета вирусов»[62] писатель и популяризатор науки Карл Циммер пишет, что дети, которые не болели в детстве простудой, по мере взросления начинают страдать разнообразными иммунными заболеваниями. Возможно, слабые инфекции тренируют и настраивают нашу иммунную систему.

С другой стороны, простуда – отвратительная штука. Помимо того, что это вообще неприятно, некоторые исследования показывают, что заражение риновирусами также ослабляет нашу иммунную систему и делает нас более уязвимыми для будущих инфекций.

Подводя итоги: я не стал бы стоять пять недель посреди пустыни ради того, чтобы навсегда избавиться от простуд. Но если когда-нибудь появится вакцина от насморка, я – первый в очереди.


Наполовину пустой стакан

ВОПРОС: А что, если стакан воды внезапно станет в буквальном смысле наполовину пустым?

– Витторио Иаковелла

ОТВЕТ: Пессимист, скорее всего, предскажет результат точнее, чем оптимист.

Когда люди говорят, что стакан наполовину пуст, они обычно имеют в виду стакан, который поровну заполнен водой и воздухом.



Согласно известному афоризму, оптимист считает, что стакан наполовину полон, а пессимист – что он наполовину пуст. Из этого парадокса родилось невероятное количество других шуток: например, инженер видит стакан, который в два раза больше, чем он должен быть, сюрреалист видит вместо стакана жирафа, жующего галстук, и так далее.

Но что, если пустая половина стакана была бы по-настоящему пустой – то есть представляла собой вакуум?[63] Этот вакуум, безусловно, не просуществовал бы долго. Но что именно с ним произойдет – зависит от ключевого вопроса, который обычно никто не задает. Какая именно половина стакана пустая?

Давайте представим себе три по-разному заполненных стакана и проследим, что с ними будет, микросекунда за микросекундой.

В середине – обычный стакан с водой (внизу) и воздухом (наверху). Справа – такой же стакан, как и в центре, только воздух заменен вакуумом. Стакан слева наполовину заполнен водой и наполовину вакуумом, но на этот раз вакуум находится в нижней половине.



Представим, что вакуум появляется в момент t=0.

В первые микросекунды ничего не происходит. В таком временном промежутке даже молекулы воздуха практически неподвижны.



По большей части молекулы мечутся туда и сюда со скоростью несколько сотен метров в секунду. Но в любой момент времени одни из них движутся быстрее других. Самые быстрые движутся со скоростью свыше 1000 м/с. Это и будут первые молекулы, которые устремятся в вакуум в правом стакане.

Вакуум в стакане слева изолирован от воздуха, так что частицам воздуха будет непросто туда попасть. Вода, будучи жидкостью, не расширяется при увеличении объема сосуда и не сможет заполнить вакуум так же, как это делает воздух. Однако в вакууме стакана она начинает кипеть, медленно заполняя паром пустое пространство.



В то время как вода на поверхности обоих стаканов начнет мед ленно выкипать, в правом стакане воздух, устремляющийся внутрь, остановит этот процесс, прежде чем тот толком начнется. Вакуум же в стакане слева продолжит медленно заполняться туманом из водяного пара.



Спустя несколько сотен миллисекунд воздух, устремляющийся в стакан справа, окончательно заполняет вакуум и ударяется в поверхность воды, посылая ударную волну сквозь толщу жидкость. Стенки стакана слегка поддаются, но выдерживают давление и не разбиваются. Волна отражается от стенок и дна стакана и возвращается обратно в воздух, где становится частью уже имеющейся турбулентности.



Ударная волна, возникшая в результате «схлопывания» вакуума, за одну миллисекунду достигает двух других стаканов. Стакан и вода слегка деформируются, когда волна проходит через них. Через несколько миллисекунд волна достигает наших ушей и мы слышим громкий хлопок.



Примерно в это время левый стакан явно начинает подниматься в воздух.

Давление воздуха пытается сплющить стакан вместе с водой. Это явление мы называем всасывающей силой. Вакуум в стакане справа не просуществовал достаточно долго, чтобы приподнять стакан, но в левом стакане воздух не может заполнить вакуум, то левый стакан и вода в нем начинают скользить навстречу друг другу.



Кипящая вода заполнила вакуум очень небольшим количеством водяного пара. Когда пространство становится меньше, собравшийся пар оказывается все большее давление на поверхность воды. В конце концов это замедлит кипение (так же, как это происходит при повышенном атмосферном давлении).



Тем не менее левый стакан и вода теперь перемещаются слишком быстро, чтобы скопившийся пар мог как-то повлиять на этот процесс. Менее чем через десять миллисекунд после того, как пошел отсчет, они устремляются друг к другу со скоростью несколько метров в секунду. Без воздушной прослойки между ними – лишь тонкое облачко пара – вода врезается в дно стакана, как молот.

Воду практически нельзя сжать, так что удар будет очень коротким и сильным. Сила воздействия на стекло огромна, и оно разлетается вдребезги.



Этот эффект «водяного молота» или гидравлического удара (он же отвечает за звук, который иногда слышно, когда выключаешь кран в доме со старыми водопроводными трубами) можно увидеть и во время демонстрации хорошо известного фокуса: по горлышку стеклянной бутылки с водой с силой бьют – и у бутылки вылетает дно.

Когда по бутылке ударяют, ее резко толкают вниз. Жидкость в бутылке не может мгновенно отреагировать на всасывание (давление воздуха) – примерно так же, как в нашем сценарии, – и между жидкостью и дном бутылки на короткое время возникает зазор, вакуум толщиной в ничтожную долю дюйма. Когда вода снова заполняет этот вакуум, возникшая ударная волна вышибает дно бутылки.

Действующих в нашем случае сил будет более чем достаточно, чтобы расколотить даже самый толстый барный стакан.

Дно стакана под давлением воды выпадает и разбивается о стол. Вода разбрызгивается вокруг, капли и осколки стекла разлетаются во все стороны.



Тем временем отделившаяся верхняя часть стакана продолжает подниматься вверх.

Спустя полсекунды присутствующие вздрагивают от громкого хлопка и непроизвольно задирают головы, чтобы посмотреть, что будет дальше.




У стакана как раз достаточно скорости, чтобы врезаться в потолок и разлететься на осколки… которые, поскольку импульс теперь погашен, падают обратно на стол.

Урок из всего этого таков – пока оптимист уверяет, что стакан наполовину полон, а пессимист ворчит, что он наполовину пуст, физик ищет надежное укрытие.

Странные (и тревожные) вопросы из папки «Входящие» сайта «А что, если?»

ВОПРОС: Всемирное потепление угрожает повышением средней температуры, а извержение супервулкана может ее понизить. Эти две опасности могут нейтрализовать одна другую?

– Флориан Зайдль-Шульц

ВОПРОС: А что, если наткнуться на бегу на проволоку, которой режут сыр? Как быстро нужно бежать, чтобы вас разрезало пополам?

– Джон Меррилл

Инопланетные астрономы

ВОПРОС: А что, если на ближайшей пригодной к обитанию планете действительно есть жизнь? Допустим, у этой цивилизации есть примерно такие же технологии, что и у нас. Если они сейчас посмотрят на нашу звезду, что они увидят?

– Чак Х.

ОТВЕТ:



Теперь давайте попробуем дать более развернутый ответ. Прежде всего…

Радиосигналы

Фильм «Контакт» сделал популярной идею о том, что инопланетяне слушают наши радиопередачи. К сожалению, вероятность этого весьма невелика. Проблема вот в чем: космос очень большой.

Можно углубиться в физику межзвездных радиоволн[64], но к проблеме можно подойти и с точки зрения экономики: если ваш телевизионный может достигнуть другой звезды, вы тратите деньги впустую. Обслуживание передатчика стоит дорого, и существа на других планетах не покупают продукты из рекламных роликов, благодаря которым оплачивается содержание радиостанции.

Детальная картина чуть сложнее, но в целом можно сказать, что чем более совершенными становятся наши технологии, тем меньше радиопередач «утекает» в космос. Мы отказываемся от гигантских антенн и переходим на кабель, оптоволокно и сотовые сети с очень четко очерченной зоной покрытия[65].

Когда-то сигналы нашего телевидения можно было (хоть и с большим трудом) отследить из космоса, но сейчас эта возможность исчезает. Даже в конце XX века, когда наши теле– и радиопередачи, можно сказать, кричали в бездну во весь голос, этот сигнал на расстоянии всего нескольких световых лет слабел до уровня, на котором его невозможно было бы обнаружить. Потенциально обитаемые экзопланеты, которые мы обнаружили к настоящему моменту, находятся на расстоянии десятков световых лет, так что шансы, что там сейчас повторяют наши тогдашние шутки, не слишком велики.

Но теле– и радиопередачи не были самыми сильными земными радиосигналами. Их вполне могло заглушить излучение радиолокационных станций дальнего обнаружения.

Система дальнего обнаружения, это порождение холодной войны, состояла из нескольких наземных и воздушных станций, разбросанных по Арктике. Эти станции 24 часа в сутки сканировали атмосферу мощными лучами, и их операторы тревожно вглядывались в экраны, ища в приходящем эхе любые намеки на перемещение врага[66].

Мощное излучение этих радаров уходило и в космос, и этот сигнал вполне могли бы перехватить ближайшие экзопланеты, слушай они нас в тот момент, когда луч проходил по их участку неба. Однако те же технологиии, которые делают ненужными телевышки, влияют и на радиолокационные станции. Аналогичные современные системы – там, где они еще существуют, – гораздо менее масштабны тише и могут в конце концов могут быть полностью заменены новыми технологиями.



Самый мощный радиосигнал, который испускает Земля – это луч радиотелескопа обсерватории Аресибо в Пуэрто-Рико. Эта огромная тарелка может функционировать как радарный передатчик: она «освещает» радиоволнами окрестные небесные тела вроде планеты Меркурий или объектов пояса астероидов и получает от них ответные сигналы. По сути дела, это радиофонарик, которым мы освещаем планеты, чтобы лучше их видеть (да, это именно так безумно, как звучит).

Однако телескоп Аресибо передает сигнал только время от времени, и у него узкий луч. Если бы экзопланета оказалась в зоне действия луча Аресибо, а инопланетяне как раз в этот момент нацелили бы в нашу сторону свою принимающую антенну, то все, что они услышали бы, – короткий импульс радиосигнала, а затем тишина[67].


Так что гипотетические инопланетяне, вглядывающиеся в Землю, возможно, не смогут заметить нас при помощи радиоантенн.

Однако еще имеется…

Видимый свет

Тут шансов на успех больше. Наше Солнце очень яркое [источник не указан], и его лучи освещают Землю и Луну [источник не указан]. Часть этих лучей отражается обратно в космос в виде так называемого пепельного света, проходящего в том числе и через нашу атмосферу, прежде чем направиться дальше, к звездам. Этот эффект теоретически можно наблюдать с какой-нибудь экзопланеты.

Это ничего не смогло бы рассказать инопланетному наблюдателю о человечестве, но если наблюдать за Землей достаточно долго, то можно многое понять про земную атмосферу по ее отражательной способности. Возможно, инопланетным астрономам удалось бы понять, и как устроен круговорот воды на нашей планете (и наша богатая кислородом атмосфера подсказала бы им, что у нас тут происходит что-то весьма любопытное).

Так что в конце концов самый ясный сигнал, полученный с Земли, может исходить вовсе не от нас: его могут послать водоросли, которые занимались ландшафтным дизайном планеты – и видоизменением сигналов, которые она посылает в космос, – в течение миллиардов лет.


О, уже поздно. Нам пора.


Конечно, если бы мы хотели послать более ясный сигнал, мы могли бы это сделать. Проблема с радиограммой в том, что инопланетяне должны слушать эфир именно в тот момент, когда она придет.

Но мы могли бы привлечь их внимание иначе. При помощи ионных двигателей, ядерных силовых установок или просто удачного использования гравитационного поля Солнца мы могли бы послать зонд из нашей Солнечной системы с достаточной скоростью, чтобы он достиг окрестностей близлежащей звезды за несколько десятков тысячелетий. Если мы сможем разработать систему управления, которая выдержит столь длительный полет (а это само по себе непросто), то ее можно было бы использовать, чтобы направиться к любой населенной планете.

Для мягкой посадки нам пришлось бы затормозить, но для торможения требуется еще больше топлива, чем для разгона… Но ведь мы же хотим, чтобы они нас точно заметили, правильно?

И если бы эти инопланетяне в этот момент посмотрели в сторону нашей Солнечной системы, вот что они бы увидели:


Больше никакой ДНК

ВОПРОС: А что, если у какого-нибудь человека внезапно исчезнет вся ДНК? Как долг о этот человек протянет? Наверное, это жестокий вопрос…

– Нина Чарест

ОТВЕТ: Если вы лишитесь всей своей ДНК, вы немедленно станете весить на 150 г меньше.

Сбросить 150 граммов

Я не стал бы рекомендовать этот способ. Сбросить 150 г можно гораздо проще, например:

• снять футболку;

• сходить в туалет;

• обрезать волосы (если они достаточно длинные);

• стать донором, а потом пережать трубочку, когда накапает 150 мл крови, и отказаться сдавать больше;

• взять в руки шар диаметром 1 м, наполненный гелием;

• избавиться от пальцев.


Кроме того, можно потерять 150 граммов, просто проделав путешествие от полюса к экватору. У этого две причины. Во-первых, наша Земля имеет такую форму:



Если вы стоите на Северном полюсе, вы на 20 км ближе к центру Земли, чем если вы стоите на экваторе, и воздействие гравитации на полюсе ощущается сильнее.

Во-вторых, если вы стоите на экваторе, вы испытываете воздействие центробежной силы.



Из-за этих двух обстоятельств при перемещении между экватором и полюсами можно приобрести или потерять до половины процента веса вашего тела.

Причина, по которой я сосредоточился на весе, заключается в том, что, если бы ваша ДНК вдруг исчезла, потеря массы была бы не первым явлением, которое вы бы заметили. Возможно, вы испытали бы что-то вроде крошечной ударной волны из-за того, что все клетки чуточку сжались. А может, и не испытали бы.

Если бы в момент потери ДНК вы стояли на ногах, вы могли бы слегка вздрогнуть. Когда вы стоите, ваши мышцы постоянно работают, чтобы удерживать вас в вертикальном положении. Сила ваших мускулов не изменилась бы, а вот масса, которую они удерживают, – например масса ваших конечностей, – изменится. Поскольку F=ma, то отдельные части тела стали бы двигаться чуточку быстрее. После этого вы, вероятно, чувствовали бы себя вполне нормально.

Некоторое время.

Ангел-убийца

Никто никогда не терял всю свою ДНК[68], так что мы не можем сказать точно, какие именно медицинские последствия наступят и в каком порядке. Но чтобы хотя бы понять, на что это будет похоже, обратимся к отравлению грибами.

Amanita bisporigera – вид грибов, который встречается на востоке Северной Америки. Этот гриб вместе с его родственными видами называют «ангелом-убийцей».



Это маленький, белый, невинный с виду гриб. Если у вас было такое же детство, как у меня, вам наверняка говорили, чтобы вы не ели грибы, найденные в лесу. Причина этого – Amanita[69].

Если съесть «ангела-убийцу», то остаток дня вы будете чувствовать себя отлично. Однако ночью или следующим утром у вас появятся симптомы, напоминающие холеру, – рвота, боль в животе и понос. Затем вам полегчает.

К тому моменту, когда вы почувствуете себя лучше, спасти вас, скорее всего, уже будет нельзя. Amanita bisporigera содержит токсины альфа-, бета– и гамма-аманитина, которые связываются с ферментом, использующимся для чтения ДНК. Таким образом, эти токсины обрывают процесс, благодаря которому клетки следуют инструкциям ДНК.

Аманитины наносят непоправимый ущерб клеткам, в которые попадают. Поскольку большая часть вашего тела состоит из клеток[70], для вас это плохая новости. Смерть в результате отравления этими токсинами чаще всего происходит из-за отказа почек или печени, поскольку это первые органы, в которых аккумулируется токсин. Иногда срочная медицинская помощь и пересадка печени могут спасти пациента, но значительное число людей, отравившихся Amanita bisporigera, погибает.

Страшнее всего при этом отравлении фаза «ходячий мертвец» – это когда пострадавшему кажется, что он в порядке (или ему хотя бы становится лучше), но при этом его клеткам уже нанесены непоправимые и смертельные повреждения.

Это типичная ситуация при повреждении ДНК, и, скорее всего, мы увидим нечто подобное в случае человека, который потерял ДНК.

Еще ярче это иллюстрируется двумя другими примерами повреждения ДНК – в результате химиотерапии или облучения.

Радиация и химиотерапия

Препараты для химиотерапии – это довольно прямолинейные средства. Некоторые лекарства действуют избирательнее, чем другие, но многие из них просто прерывают деление клеток как таковое. Причина, по которой подобная терапия в большей степени убивает раковые клетки (а не любые клетки пациента), заключается в том, что раковые клетки делятся постоянно, в то время как большая часть обычных клеток – только от случая к случаю.

Но некоторые человеческие клетки все же делятся постоянно. Чаще всего это делают клетки костного мозга, – этой фабрики по производству крови.



Кроме того, костный мозг – это ключевая часть человеческой иммунной системы. Без него мы теряем возможность производить белые кровяные клетки, и иммунная защита рушится. Химиотерапия повреждает иммунную систему, и это делает раковых больных особенно уязвимыми к случайным инфекциям[71].

Есть и другие типы быстро делящихся клеток в нашем организме. Волосяные фолликулы и выстилка желудка также постоянно делятся, поэтому химиотерапия и вызывает потерю волос и тошноту.

Доксорубицин, один из самых сильных и наиболее частых препаратов для химиотерапии, работает, встраиваясь между нитями двойной спирали ДНК и спутывая их. Это похоже на то, как если бы вы капнули суперклеем на клубок пряжи – ДНК превращается в бессмысленный комок[72]. Первые побочные эффекты доксорубицина проявляются спустя несколько дней после начала терапии – это тошнота, рвота и диарея (и это логично, поскольку лекарство убивает клетки пищеварительной системы).

Потеря ДНК тоже вызвала бы смерть клеток и, вероятно, аналогичные симптомы.

Радиация

Большие дозы гамма-радиации также повреждают ДНК. Лучевая болезнь – вот, пожалуй, тип повреждения, который больше всего напоминает сценарий Нины. Клетки, наиболее чувствительные к радиации, как и в случае химиотерапии, находятся в костном мозге, за ними следуют клетки пищеварительного тракта[73].

Лучевая болезнь, как и отравление «ангелом-убийцей», имеет латентный период. Это период, когда тело все еще работает, но новые белки не могут синтезироваться и наступает коллапс иммунной системы.

В случае сильной лучевой болезни основной причиной смерти будет отказ иммунной системы. Без резерва лейкоцитов тело неспособно сопротивляться инфекциям, и обычные бактерии могут проникнуть в организм и творить в нем что хотят.

Итого

Потеря вашего ДНК, скорее всего, приведет к болям в желудке, тошноте, головокружению, быстрому кризису иммунной системы и смерти в течение нескольких дней или часов либо от быстрой системной инфекции, либо от отказа жизненно важных органов.



С другой стороны, хотя бы один плюс во всем этом ужасе есть. Если мы когда-нибудь окажемся в антиутопии, где правительство в духе будет собирать нашу генетическую информацию, чтобы следить за нами и контролировать нас…

…то вы останетесь невидимым.


Всепланетный самолет

ВОПРОС: А что, если попытаться пролететь на обычном земном самолете над разными объектами Солнечной системы?

– Глен Кьякьери

ОТВЕТ: Вот наш самолет[74]:



Нам придется использовать электрический двигатель, потому что двигатель внутреннего сгорания может работать только там, где есть зеленые растения. В мирах, где нет растений, кислород не задерживается в атмосфере – он соединяется с другими элементами, образуя, к примеру, углекислый газ или ржавчину. Растения исправляют это, захватывая углекислый газ и выделяя обратно в воздух кислород. Для работы двигателя внутреннего сгорания необходимо, чтобы в воздухе присутствовал кислород[75].


А вот и наш пилот:



Вот что произойдет, если наш самолет запустить над поверхностью 32 крупнейших объектов Солнечной системы:



Поскольку в большинстве случаев у этих небесных тел нет атмосферы, то самолет тут же свалится на их поверхность. Однако в нескольких случаях падение с высоты километра или меньше будет достаточно медленным, чтобы пилот остался цел – в отличие, впрочем, от систем, призванных обеспечить его выживание.

У девяти объектов атмосфера достаточно плотная, чтобы имело смысл о ней говорить: это Земля (разумеется!), затем Марс, Венера, планеты-гиганты, а также спутник Сатурна Титан и Солнце.

Давайте посмотрим, что произойдет с самолетом на каждом из этих небесных тел.

Солнце

Эту идею вы вряд ли назовете удачной. Если самолет у и удастся подлететь достаточно близко к поверхности Солнца, чтобы хоть как-то ощутить наличие атмосферы, он в буквальном смысле слова испарится меньше чем за секунду.

Марс

Чтобы посмотреть, что произойдет с нашим самолетом на Марсе, обратимся к X-Plane.

X-Plane – самый продвинутый симулятор полетов в мире, результат 20-летнего труда одного из самых преданных энтузиастов авиации[76] и сообщества его единомышленников. Этот симулятор позволяет моделировать обтекающие потоки воздуха для каждой части самолета в процессе полета, что делает его ценным инструментом для исследований, так как позволяет создавать виртуальные модели новых самолетов – и новые условия для них. В частности, если задать в настройках программы пониженную гравитацию, разреженную атмосферу, а заодно и уменьшить радиус планеты, то можно смоделировать полет на Марсе.

X-Plane говорит нам, что полет на Марсе сложен, но не невозможен. В НАСА это знают, и там обдумывали идею исследовать Марс при помощи самолета. Сложность заключается в том, что для того, чтобы удержаться в такой разреженной атмосфере, надо лететь по-настоящему быстро: только для отрыва от поверхности надо разогнаться до 1000 км/ч, а инерция при движении будет такой, что изменить курс будет почти невозможно – даже если вам удастся повернуть корпус самолета, он все равно продолжит двигаться в первоначальном направлении. Создатель X-Plane сравнил пилотирование марсианского самолета с управлением сверхзвуковым океанским лайнером.

Наша одномоторная «Сессна-172» не справится с такой задачей. Даже если этот самолетик запустить с высоты 1 км, он не сможет разогнаться достаточно быстро для того, чтобы удержаться в воздухе, и рухнет на марсианскую поверхность со скоростью почти 60 м/с (220 км/ч). Падая с 4–5 км, самолет мог бы разогнаться достаточно для того, чтобы спланировать – на скорости, составляющей примерно половину скорости звука в земном воздухе. Пережить такую посадку не смог бы никто.

Венера

К сожалению, X-Plane не может смоделировать поистине адские условия, которые царят вблизи поверхности Венеры. Но теоретические расчеты дадут нам примерное представление о том, на что был бы похож этот полет. Вкратце: самолет полетит неплохо, однако при этом сразу загорится, потом перестанет лететь, а потом перестанет быть самолетом.

Атмосфера на Венере в 60 раз плотнее, чем на Земле. Она достаточно плотная, чтобы наша «Сессна» смогла взлететь на скорости, близкой к скорости вашей обычной утренней пробежки. К сожалению, эта атмосфера настолько горячая, что в ней расплавился бы свинец. В течение первых же секунд с бортов потечет краска, системы самолета станут отказывать одна за другой, и он медленно спланирует на поверхность, разваливаясь в воздухе под воздействием высоких температур.

Гораздо лучшая идея – лететь над венерианскими облаками. Хотя условия близ поверхности Венеры ужасают, верхние слои ее атмосферы на удивление похожи на земные. На высоте 55 км человек смог бы выжить, если у него будет кислородная маска и защитный костюм: воздух имеет комнатную температуру, а давление примерно такое же, какое бывает в земных горах. Костюм же необходим, чтобы защитить вас от серной кислоты[77].

Кислота – это не слишком приятно, однако зона прямо над облаками все равно неплохо подходит для самолета, если только у него нет незащищенных металлических деталей, которые кислота могла бы разъесть. И если этот самолет в принципе способен летать в условиях постоянно бушующего 12-балльного урагана (простите, я забыл упомянуть об этом раньше), то…

В общем, Венера – ужасное место.

Юпитер

Наша «Сессна» не сможет лететь над Юпитером, потому что здесь слишком сильная гравитация. Мощность, которая потребуется для горизонтального полета в атмосфере Юпитера, должна быть в три раза выше, чем на Земле. Начиная с комфортного давления, примерно равному земному на уровне моря, мы разогнались бы среди бушующих ветров до 275 м/с (965 км/ч), погружаясь все глубже в слои аммиачного и водяного льда, пока нас и наш самолетик не раздавило бы. Поверхности, на которую можно было бы упасть, у Юпитера просто нет: атмосфера этого газового гиганта просто плавно переходит в жидкое тело планеты по мере того как вы погружаетесь в нее все глубже.

Сатурн

Тут ситуация немного лучше, чем на Юпитере. Меньшая сила гравитации, в общем и целом близкой к земной, и чуть более плотная (но все еще разреженная) атмосфера означают, что мы сможем продержаться несколько дальше, прежде чем нас из-за холода и сильнейшего ветра постигнет та же судьба, что и на Юпитере.

Уран

Уран – это странный на вид шар равномерно синего цвета. На этой планете тоже бушуют сильные ветра и царит жгучий холод, но это самая дружелюбная из газовых планет, и возможно, здесь на нашей «Сессне» мы смогли бы немного полетать. На самом Уране смотреть, наверное, особенно не на что, зато можно полюбоваться на его узкие кольца в небе.

Нептун

Если вы все же планируете полет вокруг одного из ледяных гигантов, я бы порекомендовал скорее Нептун[78], а не Уран. Здесь хотя бы есть облака, на которые можно полюбоваться, прежде чем вы замерзнете насмерть или ваш самолет развалится из-за турбулентности.

Титан

Лучшее мы приберегли напоследок. Когда речь заходит о самолетах, то Титан может оказаться даже более подходящим местом, чем Земля. Атмосфера у него плотная, но сила гравитации невелика, и хотя здешний воздух плотнее земного в четыре раза, давление в атмосфере выше всего на 50 %. Меньшая, чем на Луне, гравитация означает, что летать будет легко. Нашу «Сессну» можно бы поднять в воздух при помощи педальной тяги.

На самом деле, человек мог бы взлететь на Титане, используя лишь силу собственных мускулов. Человеку с дельтапланом удалось бы взлететь и парить исключительно с помощью: гигантских плавательных ласт, надетых на ноги, или даже просто размахивая искусственными крыльями. Физические затраты минимальны – возможно, потребуется не больше усилий, чем при ходьбе.

Оборотной стороной медали (а оборотная сторона всегда имеется) будет холод. На Титане температура составляет 72 градуса по шкале Кельвина, что примерно соответствует температуре жидкого азота. Исходя из расчетов, основанных на стандартах обогрева легкомоторных самолетов, я бы предположил, что кабина «Сессны» на Титане будет терять тепло со скоростью два градуса в минуту.

Батареи смогут поддержать тепло еще некоторое время, но в итоге самолет замерзнет и рухнет на поверхность. Космический зонд «Гюйгенс», который падал на Титан с практически опустошенными батареями и делал в процессе увлекательные снимки, замерз насмерть всего через несколько часов, проведенных на поверхности: после приземления он смог отправить на Землю лишь одну фотографию.

Если бы люди нацепили искусственные крылья, чтобы взлететь, мы могли бы воплотить миф об Икаре в его титановой версии: крылья замерзли бы, развалились, и мы разбились бы насмерть.

Но я никогда не считал историю Икара назиданием о том, что для человечества существуют пределы. Я считаю ее просто предупреждением о том, что есть пределы использования воска в качестве клея. Проблема холода на Титане – всего лишь инженерная проблема. Если нашу «Сессну-172» соответствующим образом оборудовать и правильно подобрать источники тепла, она вполне могла бы полететь над Титаном – и мы тоже.


Странные (и тревожные) вопросы из папки «Входящие» сайта «А что, если?»

ВОПРОС: Какова общая питательная ценность (калории, жиры, витамины, минералы и т. д.) тела среднего человека?

– Джастин Ризнер

ВОПРОС: До какой температуры должна быть нагрета цепная пила (или другой режущий инструмент), чтобы моментально прижечь раны, которые она же и нанесет?

– Сильвия Галлахер

Да пребудет с вами Сила!

ВОПРОС: А что, если измерить Силу, которую может вырабатывать магистр Йода? Какова будет ее мощность?

– Райан Финни

ОТВЕТ: Я (конечно же!) не буду брать в расчет приквелы.

Самым ярким проявлением Силы в оригинальной трилогии была сцена, когда Йода поднял звездный истребитель X-wing Люка из болота. Если говорить исключительно с точки зрения перемещения физических объектов, это было самым энергозатратным действием с использованием Силы на протяжении всей трилогии.

Энергия, необходимая, чтобы поднять объект на заданную высоту, равна массе объекта, умноженной на ускорение свободного падения и эту высоту. Сцена с истребителем позволяет рассчитать минимальный уровень пиковой выходной мощности Йоды.

Сначала нужно понять, насколько тяжел этот летательный аппарат. Масса X-Wing нигде официально не указана, зато известна его длина – 12,5 м. Истребитель F-22 имеет длину 19 м и вес 19 700 кг. Исходя из этого соотношения, можно оценить массу X-Wing примерно в 5 тонн.



Дальше нам нужно знать, как быстро он поднимался из болота. Я просмотрел соответствующие кадры фильма и замерил время, за которое X-Wing поднялся из воды.



Передняя посадочная опора поднимается из воды примерно за три с половиной секунды, и я оценил ее длину примерно в 1,4 м (основываясь на сцене из фильма «Новая надежда», когда кто-то из команды протискивается мимо нее), из чего можно заключить, что X-wing поднимался со скоростью 0,39 м/с.

Наконец, нам надо знать силу гравитации на Дагобе. Здесь, думал я, мы точно окажемся в тупике, потому что, несмотря на весь фанатизм поклонников «Звездных войн», вряд ли где-то есть подробное описание геофизических характеристик каждой планеты этой вселенной. Правильно?

Не, неправильно. Я недооценил фэндом. На сайте Wookieepeedia выложен как раз такой каталог, и он сообщает нам, что поверхностная гравитация на Дагобе составляет 0,9 g. Если принять в расчет массу X-Wing и скорость подъема, это дает нам пиковую выходную мощность:



Этого достаточно, чтобы обеспечить электроэнергией квартал домов в пригороде; примерно такова же (25 лошадиных сил) мощность электрической модификации автомобиля Smart. С учетом нынешних цен на электричество Йода обойдется нам примерно два доллара в час.



Но телекинез – только один из вариантов проявления Силы. А как насчет той молнии, которой Император пытал Люка? Она никак не объясняется с точки зрения физики, но катушки Тесла способны демонстрировать аналогичные эффекты и использовать где-то 10 киловатт, что примерно приравнивает Императора к Йоде. (Обычно катушки Теслы используют много коротких импульсов. Если Император поддерживает постоянную электрическую дугу, как при электросварке, мощность может измеряться и в мегаваттах.)

Так как насчет Люка? Я исследовал сцену, где он впервые использует Силу, чтобы вытащить световой меч из снега. Здесь сложно рассчитать цифры, но я просмотрел сцену по кадрам и приблизительно оценил пиковую выходную мощность Люка в 400 Вт. Это малая доля от 19 кВт Йоды, и Люк поддерживал ее только в течение доли секунды.

Так что, похоже, Йода – лучшее, что у нас есть в качестве источника энергии. Но с учетом того, что всемирное потребление электроэнергии превышает 2 тераватта, потребуется сто миллионов Йод, чтобы удовлетворить наши запросы. С учетом всего этого переход на энергию Йоды, скорее всего, не стоит трудов, хотя эта энергия и была бы «зеленой» – в самом буквальном смысле слова.


Транзитные штаты

ВОПРОС: Над каким штатом США чаще всего пролетают самолеты?

– Джесси Рудерман

ОТВЕТ: Когда речь заходит о «транзитных» (flyover) штатах, обычно в воображении предстают эти большие прямоугольные штаты там, на Западе, над которыми мы пролетаем из Нью-Йорка в Лос-Анджелес или из Сан-Франциско в Чикаго, никогда не приземляясь по дороге.

Но над каким штатом действительно пролетает самое большое количество самолетов? Очень многие полеты проходят вдоль Восточного побережья – легко себе представить, что перелетов над Нью-Йорком больше, чем над штатом Вайоминг.

Чтобы выяснить, какие штаты в самом деле можно назвать транзитными, я просмотрел более 10 000 воздушных маршрутов, выясняя, над какими штатами проходит каждый из них.

Удивительно, но штат, над которым пролетает больше всего самолетов, не приземляясь и не взлетая с его территории, это…


…Вирджиния.

Результат меня удивил. Я вырос в Вирджинии и точно никогда не думал о ней как о «транзитном» штате. Этот факт тем более удивителен, что в Вирджинии есть несколько крупных аэропортов – в частности, именно на ее территории расположены два из них, обслуживающих Вашингтон – Национальный аэропорт имени Рейгана и международный аэропорт Даллеса, то есть большая часть полетов в федеральную столицу и из нее не засчитываются в качестве транзитных перелетов над Вирджинией, поскольку эти самолеты садятся в самой Вирджинии.

Вот карта штатов США, раскрашенная в зависимости от количества ежедневных перелетов:



Сразу за Вирджинией следуют Мэриленд, Северная Каролина и Пенсильвания. Над этими штатами транзитных перелетов куда больше, чем над остальными.

Так почему же все же Вирджиния? Тут есть несколько факторов, но главный из них – это международный аэропорт Хартс-филд – Джексон в Атланте, самый загруженный в мире. Через Хартсфилд – Джексон проходит больше пассажиров и рейсов, чем через аэропорты Токио, Лондона, Пекина, Чикаго или Лос-Анджелеса. Это хаб (домашний аэропорт) авиакомпании «Дельта», которая до недавнего времени была крупнейшей в мире, а это значит, что пассажиры, летящие «Дельтой», часто делают пересадки в Атланте.

Благодаря большому количеству полетов из Атланты на северо-запад США, 20 % всех полетов из Атланты пересекают Вирджинию, а 25 % – Северную Каролину, внося немалый вклад в статистику обоих штатов.



Однако Атланта – не главный фактор. Аэропорт, рейсы которого чаще всего пролетают над Вирджинией, оказался для меня сюрпризом: международный аэропорт Пирсон в Торонто! На первый взгляд, это несколько неожиданная отправная точка для рейсов, пересекающих Вирджинию, и тем не менее самолеты из крупнейшего аэропорта Канады летят над Вирджинией чаще, чем рейсы из нью-йоркских аэропортов им. Кеннеди и Ла-Гвардия вместе взятых.



Отчасти причина победы Торонто заключается в том, что из него летит много прямых рейсов на Карибские острова и в Южную Америку, и по пути они пересекают воздушное пространство США[79]. Кроме того, Торонто – основная точка отправления транзитных рейсов не только через Вирджинию, но и через штаты Западная Вирджиния, Пенсильвания и Нью-Йорк. Эта карта показывает, рейсы из какого аэропорта чаще всего пролетают над тем или иным штатом:


Соотношения транзитов

Еще один возможный способ определить «самый транзитный» штат – выяснить, у какого штата самое высокое соотношение транзитных «пролетов» над ним и рейсов, для которых он является конечной точкой. По этому критерию «самыми транзитными» оказываются по большей части наименее населенные штаты. В первую десятку предсказуемо входят Вайоминг, Аляска, Монтана, Айдахо, Северная и Южная Дакота.

Однако самый высокий показатель тем не менее оказался у довольно неожиданного штата: это Делавэр.

Но этому быстро нашлось весьма логичное объяснение: в Делавэре нет больших аэропортов.

В принципе, в Делавэре есть несколько аэродромов, в том числе авиабаза Довер (DOV) и аэропорт Ньюкасл (ILG), и при этом только последний можно считать гражданским аэропортом. Однако после того как в 2008 году прекратила полеты компания Skybus Airlines, отсюда больше не летают самолеты[80].

Самые «не-транзитные» штаты

Меньше всего транзитных перелетов – над Гавайями, что логично. Этот штат состоит из крохотных островов посреди самого большого из океанов: нужно сильно постараться, чтобы попасть в эту зону.

Из 49 материковых штатов США меньше всего перелетов проходит над Калифорнией. Это меня удивило, так как Калифорния сильно вытянута вдоль побережья, и, казалось бы, многие рейсы, пролетевшие над Тихим океаном, должны ее пересекать.

Однако после 11 сентября 2001 года, когда в качестве оружия были использованы лайнеры с полным грузом топлива, Федеральное управление гражданской авиации США постаралось сократить число самолетов, летящих над территорией США с полными баками. Большая часть международных рейсов, которые могли бы пролететь транзитом над Калифорнией, вместо этого имеют стыковку в одном из местных аэропортов.

Перелеты под штатами

И наконец, ответим на горааздо более странный вопрос: а под каким штатом чаще всего пролетают самолеты? Точнее, про какой штат Америки можно сказать, что самолеты чаще всего пролетают над территорией, расположенной на строго противоположной от него стороне земного шара?

Это, как выясняется, как раз Гавайи.

Причина, по которой такой маленький штат побеждает в этой номинации, заключается в том, что по другую сторону Земли от большей части территории США находится Индийский океан, над которым пролегает очень мало коммерческих маршрутов. Гавайи, с другой стороны, находятся «напротив» Ботсваны – государства в южной Африке. По сравнению с другими континентами, над Африкой летает не так уж много самолетов, но все же этого достаточно, чтобы Гавайи победили.

Бедная Вирджиния… Мне, как человеку, который здесь вырос, тяжело принять новость о том, что Вирджиния – это штат, мимо которого чаще всего пролетают мимо. Но теперь я заставлю себя иногда – хотя бы изредка – смотреть в небо и махать рукой.

А если вы обнаружите себя на борту рейса 104 из Йоханнесбурга, ЮАР, в Лагос, Нигерия (ежедневно в 09:35), не забудьте посмотреть вниз, помахать рукой и прошептать: «Алоха!».

Падение с помощью гелия

ВОПРОС: А что, если я выпрыгну из самолета, имея при себе гигантский сдутый воздушный шар и пару баллонов гелия, и во время падения надую шар? Как долго нужно падать, чтобы он успел замедлить меня для безопасного приземления?

– Колин Роу

ОТВЕТ: Как бы нелепо это ни звучало, подобное возможно.

Падать с большой высоты опасно [источник не указан]. Воздушный шар в самом деле мог бы спасти вас, хотя обычный шарик с вечеринки, ясное дело, с задачей не справится.

Если воздушный шар будет достаточно большим, гелий даже не понадобится. Оболочка шара сможет сработать как парашют и замедлить вас до скорости, которая уже не будет смертельной.



Понятно, что для выживания необходимо избежать приземления на высокой скорости. Как было написано в одной медицинской статье:

Разумеется, очевидно, что скорость или высота падения сами по себе не наносят повреждений… Однако быстрое изменение ускорения, как, например, после падения на бетон с высоты десяти этажей, – это другой случай.

По сути дела, это просто длинный пересказ старой фразы «убивает не падение, а внезапная остановка в конце».

Чтобы шар, наполненный воздухом (а не гелием), сработал как парашют, он должен иметь диаметр от 10 до 20 м, поэтому надуть его при помощи переносных баллонов не получится – он слишком большой. Огромный вентилятор мог бы накачать в шар воздуха из атмосферы, но раз уж мы дошли до такого, то лучше просто использовать парашют.

Гелий

Гелий упрощает ситуацию.

Нужно не так уж много гелия, чтобы поднять в воздух человека. В 1982 году некий Ларри Уолтерс пролетел над Лос-Анджелесом в шезлонге, привязанном к метеозондам, и в конце концов достиг высоты в несколько километров. Пролетев через воздушное пространство международного аэропорта Лос-Анджелеса, Уолтерс приземлился, прострелив несколько шаров из пневматического ружья[81].

Сразу по приземлении Уолтерс был арестован, хотя было не совсем понятно, какое обвинение ему следует предъявить. Инспектор по безопасности Федерального управления гражданской авиации США заявил «Нью-Йорк таймс»: «Мы знаем, что он нарушил какой-то пункт федерального закона об авиации, и выдвинем обвинение, как только решим, какой именно».

Относительно небольшого шара с гелием – точно меньшего диаметра, чем парашют, – хватит для того, чтобы замедлить ваше падение, однако если сравнивать этот шар с детским воздушным шариком, он все равно будет гигантским. Самые крупные баллоны с гелием, которые можно взять напрокат, вмещают чуть больше 7 000 л, и вам потребуется как минимум десять таких баллонов, чтобы наполненный шар смог выдержать ваш вес.

Причем надувать шар пришлось бы очень быстро. Баллоны со сжатым гелием гладкие и обычно довольно тяжелые, а это значит, у них высокая равновесная скорость. У вас было бы всего несколько минут на то, чтобы все их опустошить (как только баллон опустеет, его можно сбросить вниз).

Обойти эту проблему, начав падать с большей высоты, не получится. Как мы уже выяснили в истории со стейком, верхние слои атмосферы сильно разрежены и любой объект, падающий из стратосферы, будет очень сильно ускоряться, пока не достигнет нижних слоев атмосферы, а затем весь оставшийся путь будет падать медленно. Это верно для всех объектов, от маленьких метеоритов до Феликса Баумгартнера.

Но если бы вы смогли надуть шар быстро (например, подключив к нему несколько баллонов одновременно), вам удалось бы замедлить свое падение. Только не используйте слишком много гелия, иначе обнаружите себя парящим на высоте 5 км, словно второй Ларри Уолтерс.



Пока я делал расчеты для ответа на вопрос Колина, я умудрился несколько раз добиться того, что у меня на дифференциальных уравнениях зависла программа Mathematica. Кроме того, мой IP-адрес заблокировали в поисковой системе Wolfram \ Alpha из-за слишком большого количества запросов. Форма апелляции для заблокированных пользователей требует объяснить, с чем было связано такое количество запросов, и я честно написал: «Я вычислял, сколько баллонов с гелием нужно взять напрокать, чтобы надуть шар, который можно было бы использовать как парашют при падении из реактивного самолета».

Прости, Wolfram.

Все на выход!

ВОПРОС: А что, если запустить все человечество в космос? Хватит ли у нас для этого энергии?

– Адам

ОТВЕТ: Есть много научно-фантастических фильмов, в которых показано, как человечество из-за загрязнения, перенаселения или ядерной войны покидает Землю.

Но запуск человека в космос – дело непростое. Если не рассматривать сценарии массового сокращения численности популяции Homo sapiens, то возможно ли чисто технически отправить все человечество в космос? Давайте даже оставим в стороне вопрос о том, куда мы направляемся: предположим, что искать новый дом человечеству не обязательно, но оставаться на Земле больше нельзя.



Чтобы рассчитать такую возможность, начнем с минимальной энергии, которая для этого потребуется: 4 гигаджоуля на человека. Неважно, как мы выйдем в космос: при помощи ракеты, пушки, космического лифта или лестницы, – перемещение человека (или любого другого объекта) весом 65 кг за пределы земной гравитации потребует как минимум этого количества энергии.

Что такое 4 гигаджоуля? Это около мегаватт-часа – столько электричества типичный дом в США потребляет за месяц-два. Это эквивалентно количеству энергии, содержащемуся в 90 кг бензина или микроавтобусе, полном пальчиковых батареек.



Четыре гигаджоуля, умноженные на семь миллиардов человек, – это 2,8x1018 Дж, или 8 петаватт-часов, то есть примерно 5 % ежегодного мирового потребления энергии. Много, но вполне реально.

Однако 4 гигаджоуля – это лишь минимум. На практике все зависит от выбранного нами способа транспортировки. Используй мы, например, ракеты, потребовалось бы значительно больше энергии. Это связано с фундаментальной проблемой ракетостроения – ракета, помимо прочего, вынуждена поднимать и собственное топливо.

Вернемся на секунду к 90 кг бензина, поскольку они помогут проиллюстрировать эту центральную проблему космонавтики.

Если мы хотим запустить космический корабль весом 65 кг, нам нужна энергия, которую могут выделить примерно 90 кг топлива. Мы загружаем это топливо на борт – и теперь наш корабль весит 155 кг. Корабль весом 155 кг требует 215 кг топлива, так что мы загружаем еще 125 кг…

К счастью, от этого бесконечного порочного круга – добавлению новых 1,3 кг топлива на каждый уже добавленный килограмм – спасает тот факт, что нам нет необходимости тащить все это топливо с собой до самой орбиты. Оно сгорает в полете, наша ракета становится все легче и ей требуется все меньше топлива. Формула, показывающая, сколько топлива нам нужно сжечь, чтобы двигаться с заданной скоростью, называется формулой Циолковского:



где mнач и mкон – это масса корабля с топливом до и после того, как оно будет отработано, а ?сож – это удельный импульс топлива (для ракетного топлива составляет от 2,5 до 4,5 км/с).

Для нас важно соотношение между Av (скорость, с которой мы хотим двигаться) и ?сож (скорость, с которой топливо выходит из ракеты). Чтобы покинуть Землю, ?v должно быть выше 13 км/с, а ?сож – порядка 4,5 км/с, что дает соотношение массы топлива к массе корабля по меньшей мере e13/4,5 ? 20. Если это соотношение равно x, то для запуска одного килограмма массы нам потребуется ех килограммов топлива.

Это означает, что по мере возрастания х этот объем будет становиться все больше. В результате для преодоления гравитации Земли кораблю весом в одну тонну, использующему обычное ракетное топливо, потребуется 20–50 тонн топлива. Таким образом, для запуска в космос всего человечества (общий вес – около 400 млн тонн) потребуются десятки миллиардов тонн топлива. Это много: используй мы углеводородное топливо, это была бы немалая часть оставшихся в мире запасов нефти. И мы еще не учли вес самого корабля, еды, воды и наших домашних животных[82]. Кроме того, нам потребуется топливо, чтобы построить эти корабли, доставить людей к месту запуска и так далее. Это не то чтобы совершенно невозможно, но все-таки уже выходит за рамки реального.

Но ракеты – не единственный вариант. Как бы безумно это ни звучало, возможно, лучше было бы попробовать 1) буквально залезть в космос с помощью троса или 2) вылететь с планеты на волне ядерного взрыва. Это вполне возможные – пусть и крайне рискованные – способы запуска, и оба они обсуждались с самого начала космической эры.



Первый подход – это «космический лифт», излюбленный сюжет авторов научной фантастики. Иде я заключается в том, что мы прицепляем трос к спутнику, находящемуся на достаточно далекой орбите, чтобы трос туго натянулся за счет центробежной силы. Затем мы можем отправить людей вверх по веревке при помощи обычного электричества и моторов, работающих от солнечной энергии, ядерных генераторов или чего-нибудь еще, что окажется наиболее эффективным. Сложнее всего с инженерной точки зрения подобрать для троса достаточно прочный материал – он должен быть прочнее любого материала, имеющегося в нашем распоряжении сегодня. Есть надежда, что материалы, созданные на основе углеродных нанотрубок, предоставят нам нужную прочность, так что наш немалый список инженерных задач пополняется еще одной, которую мы собираемся решить с помощью приставки «нано».

Второй вариант – использование импульса ядерного взрыва, что на удивление эффективно для быстрого перемещения большого количества материи. Вкратце идея состоит в том, что вы бросаете у себя за спиной ядерную бомбу и летите на ударной волне. На первый взгляд может показаться, что корабль должен испариться при взрыве, но выясняется, что, если у него есть защитный экран хорошо продуманной конструкции, то энергия взрыва будет отражена прежде, чем она успеет что-либо разрушить. Если бы эту технологию удалось сделать достаточно надежной, то можно было бы поднимать на орбиту целые городские кварталы и – теоретически – осуществить наш проект.

Инженерные идеи, которые лежат в основе этой технологии, считались достаточно убедительными для того, чтобы в 1960-х годах правительство США действительно попыталось построить такой корабль. Эти работы получили название «Проект «Орион», ими руководил Фримен Дайсон и история проекта изложена в отличной одноименной книге, которую написал сын Фри-мена Джордж. Сторонники импульса ядерного взрыва как космической технологии по-прежнему весьма разочарованы, что проект был прекращен еще до стадии строительства первого прототипа. Их оппоненты утверждают, что если как следует вникнуть в эту ужасающую идею – засунуть гигантский ядерный арсенал в ящик, подбросить высоко в атмосферу и несколько раз взорвать, – то становится ясно, что проект и так зашел слишком далеко.

Так что ответ на вопрос Колина следующий: хотя отправить одного человека в космос сравнительно легко, на все человечество придется максимум наших ресурсов и, возможно, уничтожить планету.

Маленький шаг для человека, но большой – для всего человечества.

Странные (и тревожные) вопросы из папки «Входящие» сайта «А что, если?»

ВОПРОС: В фильме «Тор» главный герой вращает молот так быстро, что возникает сильный торнадо. Возможно ли это?

– Давор

ВОПРОС: Если не целоваться всю жизнь, а потом использовать всю накопленную силу в одном поцелуе, какая у него будет всасывающая сила?

– Джонатан Линдстрем

ВОПРОС: Сколько ядерных ракет нужно запустить в Соединенные Штаты Америки, чтобы полностью превратить их в пустыню?

– Аноним.

Самооплодотворение

ВОПРОС: Я читал, что сперматозоиды пытаются создать из стволовых клеток костного мозга. А что, если создать сперматозоиды из стволовых клеток женщины и она же от них забеременеет? Кем она будет приходиться своему ребенку?

– Р. Скотт ЛаМорт

ОТВЕТ: Чтобы создать человека, нужно объединить два набора ДНК.



У человека эти два набора содержатся в сперматозоиде и яйцеклетке, причем в каждой такой клетке содержится случайный образец ДНК родителя (о случайности – чуть ниже). У Homo sapiens эти клетки принадлежат двум разным людям. Однако это необязательно должно быть так. Стволовые клетки, которые могут сформировать любой вид ткани, можно использовать для создания сперматозоидов (или яйцеклеток).

Пока что еще никому не удалось создать полноценный сперматозоид из стволовых клеток[83]. В 2007 году группа исследователей смогла превратить стволовые клетки костного мозга в сперматогониальные клетки – предшественники сперматозоидов. Исследователи не смогли добиться того, чтобы клетки полностью развились в сперматозоиды, но все же это был шаг вперед. В 2009 году та же исследовательская группа опубликовала статью, в которой утверждалось, что сделан последний шаг и созданы полностью функционирующие сперматозоиды.

С этой статьей есть две проблемы.

Во-первых, ученые не утверждали, что в самом деле создали сперматозоиды. В статье лишь говорилось, что получились спермоподобные клетки, но журналисты не обратили на это внимания. Во-вторых, статья была позже отозвана научным журналом, который ее опубликовал: выяснилось, что два абзаца статьи представляют собой плагиат.

И все же, несмотря на обе эти проблемы, похоже, что идея не так уж неправдоподобная, так что ответ на вопрос Р. Скотта надо начинать с самого начала.

Проследить пути движения генетической информации может быть довольно сложно. Чтобы проиллюстрировать это, давайте посмотрим на крайне упрощенную модель, которая может быть знакома поклонникам ролевых игр.

Хромосомы: издание «Подземелья и драконы»

ДНК человека расположена в 23 сегментах, которые называются хромосомами. У каждого человека есть два варианта каждой хромосомы – одна от отца, одна от матери.

В нашей упрощенной версии ДНК вместо 23 хромосом будет 7. Каждая хромосома человека содержит множество генетической информации, но в нашей модели одна хромосома будет отвечать за что-то одно.

Мы используем версию системы d 20 для ролевой игры Dungeons&Dragons, для описания параметров персонажа. Каждая ДНК содержит 7 хромосом:


1. STR

2. CON

3. DEX

4. CHR

5. WIS

6. INT

7. ПОЛ


Шесть из них – классические параметры из ролевых игр: сила (STR), телосложение (CON), ловкость (DEX), харизма (CHR), мудрость (WIS) и интеллект (INT). Последняя хромосома определяет пол.

Вот образец «нити» ДНК:



В нашей модели каждая хромосома содержит один фрагмент информации. Это либо параметр (номер, обычно от 1 до 18), либо множитель. Последняя, хромосома, определяющая пол, как и в настоящей человеческой генетике, может иметь значение X или Y

Как и в реальной жизни, каждый человек обладает двумя наборами хромосом, один от отца и один от матери. Представьте, что ваши гены выглядят так:



Сочетание этих двух наборов параметров определяет характеристики человека. Вот простое правило по совмещению параметров в нашей системе.

Если у вас есть число на обоих вариантах хромосомы, в качестве параметра берется то, которое больше. Если на одной из хромосом – число, а на другой – множитель, параметр равен числу, умноженному на множитель. Если на обеих хромосомах представлен множитель, вы получаете параметр, равный единице[84].

Вот каким оказался бы наш гипотетический персонаж:



Когда от обоих родителей наследуется цифровой параметр, результат может быть весьма впечатляющим! Телосложение нашего персонажа – сверхчеловеческий параметр 24. В общем, не считая низкого уровня Мудрости, у персонажа все параметры отличные.

Теперь давайте представим, что этот персонаж (назовем ее Элис) встретил кого-то еще (назовем его Боб).

У Боба тоже отличные параметры:



Если у них будет ребенок, каждый предоставит ему свою нить ДНК. Но нити Элис и Боба будут случайным сочетанием нитей их собственных матери и отца. Каждый сперматозоид и каждая яйцеклетка представляют собой случайную комбинацию хромосом из каждой нити. Так что давайте представим, что Боб и Элис создают следующие половые клетки:



Если эти сперматозоид и яйцеклетка объединятся, параметры ребенка будут такими:



Дочь Элис и Боба унаследует Силу матери и Мудрость отца. У нее также будет сверхчеловеческий Интеллект, спасибо отличному параметру 14, полученному от Элис, и множителю, унаследованному от Боба. Ее Телосложение, с другой стороны, гораздо слабее, чем у обоих родителей, поскольку множитель 2 от матери не так уж сильно мог повлиять на параметр 5, предоставленным отцом.

И у Элис, и у Боба был множитель в хромосоме, отвечающей за харизму. Поскольку два множителя дают параметр 1 (в том случае, если бы и Элис, и Боб предоставили множитель), то ребенок будет совершенно лишен харизмы. К счастью, шансы на это составляют всего 1 из 4.

Если бы у ребенка был множитель на обеих нитях, параметр превратился бы в 1. К счастью, поскольку множители довольно редкая вещь, то шансы, что они совпадут у двух случайных людей, невелики.

А теперь посмотрим, что было бы, если бы Элис родила ребенка от самой себя.

Сначала она произвела бы пару половых клеток, которые провели бы случайный отбор дважды:



Затем выбранные нити достались бы ребенку:



Ребенок гарантированно будет девочкой, поскольку некому предоставить Y-хромосому.

У ребенка также есть проблема – три из семи параметров, Интеллект, Ловкость и Телосложение, представлены одной и той же хромосомой с обеих сторон. В случае Ловкости и Телосложения это не проблема, так как у Элис высокие значения этих параметров, но в Телосложении она унаследовала множитель с обеих сторон, что дает ей параметр 1.

Если кто-то рожает ребенка самостоятельно, это сильно повышает вероятность того, что ребенок унаследует одну и ту же хромосому дважды, следовательно, два раза получит множитель. Шансы, что у ребенка Элис будет двойной множитель, равны 58 % – сравните это с шансом 25 % при ребенке от Боба.

В общем, если родить ребенка от самой себя, 50 % хромосом будут иметь один и тот же параметр с обеих сторон. Если этот параметр – единица или множитель, у вашего ребенка будут какие-то проблемы, даже если их не было у вас. Это состояние – наличие одинакового генетического кода на обеих нитях хромосомы – называется «гомозиготностью».

Люди

Самое часто встречающееся генетическое заболевание человека, вызванное инбридингом (близкородственным скрещиванием), – это спинальная мышечная атрофия. Она вызывает постепенную гибель клеток спинного мозга и часто приводит к смерти или тяжелой инвалидности.

Спинальная мышечная атрофия – результат аномальной версии гена пятой хромосомы. У одного из 50 людей есть эта аномалия, то есть один из 100 людей передаст ее своему ребенку, следовательно, один из 10 000 людей (100 x 100) унаследует дефективный ген от обоих родителей[85].

Если же родить ребенка от самого себя, шанс спинальной мышечной атрофии возрастает до 1 из 400, поскольку если у родителя есть этот ген (шанс 1 из 100), то с вероятностью 1 из 4 именно он достанется ребенку.

«Один из четырехсот» звучит не так уж плохо, но спинальная мышечная атрофия – это только начало.

ДНК – это сложно

ДНК – исходный код для самой сложной машины в известной нам Вселенной. Каждая хромосома содержит невероятное количество информации, и взаимодействие между ДНК и клеточными механизмами вокруг нее крайне сложно, с огромным количеством движущихся деталей и правилами в духе игры «Мышеловка»[86]. Даже если назвать ДНК «исходным кодом», это будет сильным упрощением: самые сложные программные разработки кажутся карманными калькуляторами по сравнению с ДНК.

У человека каждая хромосома влияет на множество вещей посредством разнообразных мутаций и вариаций. Некоторые из этих мутаций, как та, что отвечает за спинную мышечную атрофию, похоже, абсолютно негативны – у мутации нет положительных сторон. Если сравнивать с нашей ролевой системой, это как хромосома, в которой Сила равна единице. Если другая хромосома будет нормальной, вам достанется нормальный параметр, то есть вы станете скрытым носителем.

Другие мутации, как, например, ген серповидной анемии на хромосоме 11, могут принести и пользу, и вред. Люди, у которых этот ген представлен на обеих хромосомах, страдают от серповидной малярии. Однако, если он есть только на одной из хромосом, они получают неожиданный бонус – повышенный иммунитет к малярии.



Это как множитель «x2» в нашей ролевой системе. Один такой ген сделает вас сильнее, а два – как и два множителя – приведут к серьезному заболеванию.

Эти два заболевания иллюстрируют одну из причин, по которой необходимо генетическое разнообразие. Мутации появляются сплошь и рядом, но наши избыточные хромосомы позволяют смягчить этот эффект. Избегая инбридинга, популяция снижает вероятность того, что редкие и опасные мутации окажутся в одном геноме, на обеих хромосомах.

Коэффициент инбридинга

Биологи используют показатель, называемый коэффициентом инбридинга, чтобы оценить процент ДНК, который может оказаться идентичным. Ребенок от родителей, которые не являются родственниками, имеет коэффициент 0, тогда как ребенок, у которого идентичный двойной набор хромосом, имеет коэффициент 1.

Это подводит нас к ответу на первоначальный вопрос. Ребенок от родителя, который произвел самооплодотворение, будет неким подобием клона родителя, но с серьезными генетическими болезнями. У родителя будут все гены, которые есть у ребенка, но у ребенка не будет всех генов родителя, и половина хромосом ребенка окажется в паре с собственной копией.



Значит, у ребенка будет коэффициент инбридинга, равный 0,5. Это очень много – такой коэффициент будет у ребенка после трех поколений последовательных браков сестер и братьев. Согласно книге Д. С. Фальконера «Введение в количественную генетику», коэффициент инбридинга, равный 0,5, приведет к снижению IQ в среднем на 22 пункта и к отставанию роста к десяти годам на 10 см. Есть немалый шанс, что эмбрион просто не доживет до родов.

Подобный инбридинг часто можно было наблюдать в королевских семьях, которые пытались сохранить свою кровь «чистой». Для европейской династии Габсбургов, семейства европейских правителей середины второго тысячелетия, было характерно множество браков между двоюродными братьями и сестрами, и кульминацией стало рождение Карла II.

Коэффициент инбридинга Карла был равен 0,254, то есть уровень инбридинга оказался выше, чем если бы его родители были братом и сестрой (0,25). Он страдал серьезными физическими и умственными нарушениями и был странным (и в общем неэффективным) королем. Однажды он велел выкопать из могил тела своих родственников, чтобы на них посмотреть. Его неспособность зачать ребенка привела к концу этой королевской династии.

Самооплодотворение – рискованная стратегия, вот почему секс так популярен среди крупных и сложных организмов[87]. Иногда встречаются многоклеточные / высокоорганизованные животные, размножающиеся путем самооплодотворения[88], но это достаточно редкий случай. Обычно это происходит в обстановке, где сложно размножаться половым путем – будь-то из за нехватки ресурсов, в изолированной популяции…

…или из-за излишне самоуверенных сотрудников парка Юрского периода.


Бросок вверх

ВОПРОС: Как высоко человек может что-нибудь подбросить?

– Дейв, ирландец с острова Мэн

ОТВЕТ: Люди мастерски кидаются предметами. Честно говоря, мы в этом деле просто великолепны – ни одно животное не умеет бросаться так, как мы.

Действительно, шимпанзе швыряются своими фекалиями (и изредка камнями), но они далеко не такие меткие, как люди. Муравьиные львы разбрасывают песок, но при этом ни во что не целятся. Рыбы-брызгуны охотятся на насекомых, сбивая их метко брошенными каплями воды, но используют для этого рот, а не конечности. Жабовидные ящерицы могут брызнуть струей крови из глаз на расстояние больше 1 м. Я не знаю, почему[89] они это делают, потому что каждый раз, когда я дохожу в статье до фразы «брызнуть струей крови из глаз», я останавливаюсь и смотрю на эту фразу до тех пор, пока не понимаю, что мне нужно прилечь.



Так что, хотя существуют и другие животные, умеющие бросать предметы, только мы способны схватить первый попавшийся под руку метательный снаряд и точно поразить цель. Более того, у людей это так хорошо получается, что некоторые исследователи предположили, будто бросание камней сыграло центральную роль в развитии мозга современного человека.

Бросать предметы нелегко[90]. Чтобы бейсбольный мяч долетел до баттера, питчер должен выпустить мяч, который он бросает, в строго определенной точке траектории броска. Ошибка на полмиллисекунды в ту или другую сторону достаточна для того, чтобы мяч не попал в зону удара.

Чтобы правильно рассчитать бросок, понадобится примерно пять миллисекунд (за это время самый быстрый из возможных нервных импульсов пройдет по всей длине вашей руки). Это значит, что, когда ваша рука только еще принимает исходное положение, сигнал «выпустить мяч» уже приблизился к вашему запястью. Чтобы понять, какая тут необходима точность, представьте, что барабанщик роняет палочку с десятого этажа и попадает по своему барабану, стоящему на земле, точно в такт.



Но похоже, что мы куда лучше бросаем предметы вперед, нежели вверх[91]. Если же речь идет о максимальной высоте броска, то мы могли бы использовать снаряды, которые взлетают вверх, когда их бросают вперед: был у меня в детстве бумеранг, вечно застревавший в ветках деревьев[92]. Но мы могли бы обойти эту проблему, используя вот такое приспособление:


Устройство для попадания себе в голову бейсбольным мячом с четырехсекундной задержкой


Мы могли бы также использовать трамплин, хорошо смазанный желоб, даже пращу – любое приспособление, способное направить снаряд вверх, при этом не снижая и не увеличивая его скорость. Конечно, можно попробовать и вот так:



Я сделал основные аэродинамические расчеты для бейсбольного мяча, который бросают с разной скоростью. Высоту броска я буду исчислять в жирафах:



Средний человек, вероятно, может бросить бейсбольный мяч на высоту как минимум трех жирафов:



Неплохо натренированный человек мог бы достичь высоты пяти жирафов:



Бейсболист со скоростью подачи 130 км/ч осилит десять жирафов:



Арольдис Чепмен, рекордсмен мира по скорости подачи (зафиксированный рекорд – 105 миль в час (ок. 169 км/ч), мог бы теоретически бросить мяч на высоту 14 жирафов:



Но как насчет других снарядов, помимо мяча? Очевидно, с помощью таких инструментов, как праща, арбалет или изогнутая ракетка-хиестра для баскской пелоты, снаряды можно метать гораздо быстрее. Но давайте для ответа на этот вопрос сосредоточимся на бросках голыми руками.

Бейсбольный мяч, скорее всего, – не идеальный объект, но найти данные по скорости для других снарядов довольно сложно. К счастью, Роальд Брэдсток, британский метатель копья, как-то провел «соревнование по метанию случайных объектов», во время которого метал все подряд, начиная от дохлой рыбины и заканчивая кухонной раковиной. Опыт Брэдстока предоставляет нам массу полезных данных для расчетов[93]. В частности, мы узнаем о том, что лучший снаряд для метания – это мячик для гольфа.

Существует не так уж много документальных описаний того, как профессиональные атлеты бросают мячики для гольфа. К счастью, Брэдсток такой бросок задокументировал, и его рекорд составил 155 м. Спортсмен метал мячик с разбега, и все же у нас достаточно оснований считать, что мячик для гольфа лучше бейсбольного. С точки зрения физики, это логично: ограничивающим фактором при броске в бейсболе служит вращающий момент локтя, и мячик для гольфа, обладающий меньшим весом, позволит руке двигаться чуть быстрее.

Возможно, скорость за счет использования мячика для гольфа удастся увеличить несильно, однако вполне вероятно, что профессиональный бейсболист, у которого есть время заранее потренироваться, сможет метнуть мячик для гольфа быстрее бейсбольного мяча.

Если так, то, если учесть в расчетах торможение о воздух, Арольдис Чепмен мог бы, вероятно, метнуть мячик для гольфа на высоту шестнадцати жирафов:



Похоже, это максимально возможная высота для брошенного человеческой рукой объекта…

…если не брать в расчет способ, который позволит даже пятилетнему ребенку легко побить все эти рекорды.


Смертоносные нейтрино

ВОПРОС: Как близко надо находиться к сверхновой звезде, чтобы получить смертельную дозу нейтринного излучения?

– Др. Дональд Спектор

ОТВЕТ: Словосочетание «смертельная доза нейтринного излучения» звучит весьма странно. Увидев его в первый раз, я даже пришел в некоторое замешательство.

Если вы не имеете отношения к физике, то эти слова вас, может быть, не удивят, поэтому вот вам контекст, чтобы объяснить, почему эта мысль так удивляет меня.

Нейтрино – это призрачные частицы, которые еле-еле взаимодействуют с окружающим миром. Посмотрите на вашу руку – около миллиарда нейтрино, излученных Солнцем, проходят через нее в эту самую секунду.


Хорошо, можете перестать смотреть на свою руку.


Причина, по которой вы не замечаете поток нейтрино, заключается в том, что эти частицы по большей части игнорируют обычную материю. В среднем только один нейтрино из этого огромного потока раз в несколько лет столкнется с одним из атомов вашего тела[94].

Нейтрино настолько призрачные создания, что вся Земля для них проницаема: практически весь солнечный поток нейтрино проходит через нее, не испытывая никаких затруднений и без всяких последствий. Чтобы отслеживать нейтрино, люди строят гигантские резервуары, в которых содержатся сотни тонн материала, в надежде зафиксировать след единственного нейтрино, летящего от Солнца.

Это значит, что, когда ускоритель частиц (который производит нейтрино) хочет отправить поток этих частиц на детектор, который находится где-то в другом месте, ускорителю нужно просто прицелиться в сторону этого детектора, даже если тот находится на обратной стороне Земли!



Поэтому слова о «смертельной дозе нейтринной радиации» звучат странно – здесь объединены несопоставимые масштабы. Это как английская идиома «сбить с ног перышком»[95] или фраза «футбольный стадион, до верху полный муравьев»[96]. Если у вас есть некоторые познания в математике, то это можно сравнить с формулой ln([(x))]e – не то чтобы в ней совсем не было смысла, просто нельзя себе представить ситуацию, в которой она бы применялась[97].

Ко всему прочему, не так-то просто произвести достаточно частиц нейтрино, чтобы заставить хотя бы одну из них взаимодействовать с материей, – трудно представить себе условия, при которых их будет так много, чтобы они могли причинить вам вред.

Сверхновые как раз предоставляют нам такие условия. Задавший этот вопрос доктор Спектор, физик из колледжа Хобарт-и-Уильям-Смит, поделился со мной простым правилом оценки размера сверхновых: сколь большими они бы ни казались, в реальности они еще больше.

Вот пример для того, чтобы вы могли оценить масштаб явления. Как вы думаете, что будет ярче (с точки зрения количества энергии, доставленной на сетчатку вашего глаза): сверхновая, находящаяся на том же расстоянии от Земли, что и Солнце? Или вспышка взрыва водородной бомбы, прижатой прямо к вашему глазу?


Вы не могли бы взорвать ее поскорее? Она тяжелая.


Судя по правилу доктора Спектора, сверхновая должна быть ярче. И она действительно ярче… в миллиард раз.

Вот почему этот вопрос очень интересен – сверхновые невообразимо огромны, а нейтрино невообразимо бестелесны. В какой момент эти две невообразимости уравниваются настолько, чтобы воздействовать на человека?

Статья эксперта по радиации Эндрю Карама дает нам ответ. Она показывает, что во время взрыва некоторых сверхновых при коллапсе ядра может высвободиться 1057 нейтрино (по одному на каждый протон звезды, который при коллапсе обратился в нейтрон).

По расчетам Карама, доза нейтринной радиации на расстоянии в 1 парсек[98] будет составлять половину нанозиверта (нЗв), или 1/500 от дозы, которую вы получаете, съев один банан[99].

Смертельная доза радиации составляет примерно 4 зиверта (Зв). Используя закон обратных квадратов, мы можем рассчитать дозу радиации:



х = 0,00001118 парсек = 2,3 астрономической единицы (а.е.).


Это чуть больше, чем расстояние от Солнца до Марса.

Коллапс ядра происходит у гигантских звезд, так что если бы вы наблюдали за сверхновой с такого расстояния, вы, скорее всего, находились бы где-то во внешних слоях звезды, из которой она возникла.


Коллапс GRB 080319B был самым драматичным когда-либо наблюдавшимся событием – особенно для тех, кто парил рядом с ней на доске для серфинга.


Угроза нейтринного облучения позволяет осознать истинный размер сверхновых. Если бы вы наблюдали сверхновую на расстоянии одной астрономической единицы – и каким-то образом не сгорели бы, не испарились и не превратились в какую-то экзотическую разновидность плазмы, – даже поток призрачных нейтрино был бы достаточно плотным, чтобы вас убить.


Если перышко летит достаточно быстро, оно действительно может сбить вас с ног.


Странные (и тревожные) вопросы из папки «Входящие» сайта «А что, если?»

ВОПРОС: Некий токсин блокирует канальцевую реадсорбцию в почках, но не влияет на фильтрацию. Каковы возможные кратковременные эффекты этого токсина?

– Мэри

ВОПРОС: А что, если бы венерина мухоловка съела человека? За сколько времени он был бы полностью обезвожен и поглощен?

– Джонатан Ванг

Лежачий полицейский

ВОПРОС: На какой скорости можно столкнуться с «лежачим полицейским» и выжить?

– Мирлин Барбер

ОТВЕТ: Как ни странно, она может быть довольно высокой.

Но сначала – предупреждение: когда вы прочтете ответ, не пытайтесь переезжать искусственные неровности на высоких скоростях.

Вот несколько причин:

• вы можете сбить кого-нибудь насмерть;

• это может повредить шины, подвеску и машину в целом;

• вы читали остальные ответы в книге?



Если этого недостаточно, вот вам цитаты из медицинских журналов о повреждениях позвоночника в аналогичных ситуациях.


Рентген и компьютерная томография грудопоясничного отдела выявили компрессионные переломы у четырех пациентов… Был использован дорсальный инструментарий… Все пациенты успешно восстановились, кроме одного с переломом шеи.

Сильнее всего был раздроблен позвонок L1 (23/52, 44,2 %).

В соответствии с ранее полученными данными, ягодицы с характеристиками, приближенными к реальным, снижают собственную частоту колебаний позвоночника с ~12 до 5,5 Гц.


Последняя фраза не связана напрямую с искусственными неровностями, но я все равно решил включить эту цитату.

Обычный, небольшой «лежачий полицейский» вас не убьет

Искусственные неровности делают для того, чтобы заставить водителя притормозить. Если переехать через стандартную искусственную неровность на скорости 8 км/ч, вы просто легонько подпрыгнете, но на скорости 35 км/ч испытаете уже ощутимый толчок. Исходя из этого, можно предположить, что при переезде на скорости 140 км/ч последует пропорционально более сильный толчок, но, скорее всего, это не так.

Как показывают цитаты из медицинских журналов, иногда люди действительно получают повреждения, столкнувшись с неровностью на дороге.

Однако почти все эти несчастья происходят с очень специфической категорией пассажиров – с теми, кто сидит на жестком заднем сиденье в хвосте автобуса, идущего по плохой дороге.

Когда вы едете на машине, две вещи защищают вас от неровностей на дороге – шины и подвеска. Неважно, с какой скоростью вы наедете на «лежачего полицейского»: если он не настолько высокий, чтобы зацепить раму автомобиля, значительная часть толчка будет погашена за счет двух этих систем и вы не получите повреждений.

Однако это совершенно не обязательно пойдет на пользу этим системам. Шины могут погасить удар, но при этом лопнуть. Если неровность достаточно высока, чтобы ударить по ободу колеса, она может навсегда испортить много важных деталей машины.

Типичный «лежачий полицейский» возвышается над дорогой на 7–10 см. Это примерно равно толщине средней шины (то есть расстоянию от обода до асфальта)[100]. Значит, что, если колесо врежется в стандартную искусственную неровность, обод колеса ее не коснется – шина просто будет сжата.

Типичный легковой автомобиль можно разогнать до 200 км/ч. Если врезаться в искусственную неровность на такой скорости, вы, скорее всего, так или иначе потеряете управление и разобьетесь[101]. Однако сам удар, скорее всего, не будет фатальным.

Но если врезаться на скорости в более высокую искусственную неровность, ваша машина так легко не отделается.

Как быстро нужно ехать, чтобы гарантированно погибнуть?

Давайте прикинем, что случится, если машина едет со скоростью быстрее максимально возможной. Хотя скорость обычного седана в среднем ограничена 200 км/ч, самые быстрые спорткары могут разгоняться до 400 км/ч.

У большинства современных легковых машин имеются какие-либо электронные устройства, контролирующие обороты двигателя и искусственно ограничивающие скорость автомобиля, однако основным естественным ограничителем является сопротивление воздуха. Оно возрастает пропорционально квадрату скорости, и в какой-то момент мощности двигателя автомобиля оказывается недостаточно, чтобы ехать сквозь воздух еще быстрее.

Если бы мы действительно заставили седан ехать быстрее его максимальной скорости – например, использовав магический ускоритель, оставшийся у нас от игры в релятивистский бейсбол, – то «лежачий полицейский» был бы наименьшей из наших проблем.

Машины производят подъемную силу. Воздух, обтекающий машину, воздействует на нее целым набором разнообразных сил.


Откуда взялись все эти стрелочки?


Подъемная сила не слишком значительна при тех скоростях, которые развивают обычные машины на обычных шоссе, но на больших скоростях эта сила становится все более заметной.

У болидов «Формулы-1», снабженных аэродинамическими антикрыльями, эта сила давит сверху вниз, прижимая машину к трассе. В случае обычного седана она будет, наоборот, поднимать автомобиль.

Поклонники кузовных гонок NASCAR часто говорят о «скорости отрыва» (около 320 км/ч), когда машину заносит и она начинает крутиться на трассе. В других видах автогонок происходили драматические аварии, когда машина переворачивалась назад по ходу движения: аэродинамические обвесы не срабатывали так, как планировалось.

В общем, при скорости в 250–500 км/ч типичный седан оторвется от земли, перевернется и разобьется… прежде чем вы доберетесь до первого «лежачего полицейского».


Срочная новость: ребенок и неопознанное существо в велосипедной корзинке погибли в аварии


А что, если бы вы не дали машине взлететь? Сила ветра на такой скорости все равно сорвала бы капот, боковые панели и двери. На еще большей скорости автомобиль развалился бы на части, а то и сгорел, как космический аппарат на входе в атмосферу.

Каково же ограничение скорости?

В штате Пенсильвания водителям добавляют по 2 доллара к штрафу за каждую милю (1,6 км/ч) превышения лимита скорости.

Таким образом, если бы вы налетели на «лежачего полицейского» в Филадельфии со скоростью в 90 % от скорости света, то мало того, что вы разрушили бы город…



…вас бы могли еще оштрафовать на 1,14 миллиарда долларов.

Потерявшиеся бессмертные

ВОПРОС: А что, если бы два бессмертных человека оказались на противоположных сторонах планеты, похожей на Землю, но необитаемой? Сколько времени им понадобится, чтобы найти друг друга? Сто тысяч лет? Миллион? Миллиард?

– Итан Лейк

ОТВЕТ: Начнем с простого ответа, из тех, что так любят физики: 3000 лет.

Именно столько времени уйдет на то, чтобы два человека нашли друг друга, если предположить, что они случайным образом бродят по планете по 12 часов в день и что они должны сблизиться на расстояние 1 км, чтобы друг друга увидеть.



Проблемы с этой моделью очевидны[102]. Во-первых, мы исходили из того, что вы всегда видите человека, если он в километре от вас. Это возможно только в идеальных условиях – человек, бредущий по горному хребту, может быть виден за километр, но в густом лесу во время дождя два человека могут пройти в нескольких метрах один от другого и не заметить друг друга.

Можно попробовать рассчитать среднюю видимость во всех частях света, но тогда мы столкнемся с другим вопросом: зачем двум странникам, которые пытаются друг друга найти, терять время в густых джунглях? Было бы разумнее оставаться на открытом, ровном пространстве, где легко и увидеть, и быть увиденным[103].



Если же мы примем в расчет психологию участников, то наша модель и вовсе развалится[104]. Почему мы предполагаем, что двое наших бессмертных станут бродить случайным образом? Оптимальная стратегия должна быть совершенно иной. А какой?

Если согласовать планы заранее, то задача упрощается. Можно договориться о встрече на Северном или Южном полюсе, или (если до полюсов не добраться) в самой высокой точке суши, или у истока самой длинной реки. Если встретиться почему-либо не удастся, можно случайным образом перемещаться между заранее согласованными вариантами локаций. У наших путешественников полно времени.

Но если у них не было шанса договориться заранее, ситуация усложняется. Не зная стратегии другого человека, как догадаться, какой должна быть ваша собственная стратегия?

Есть старая задачка, еще из тех времен, когда еще не было мобильных телефонов, и звучит она примерно так:

Предположим, вы должны встретиться с приятелем в каком-то американском городке, где никто из вас еще не был. У вас не было возможности оговорить место встречи заранее. Куда вы пойдете?

Автор загадки предполагал, что самое логичное решение – направиться в главное почтовое отделение города и там ждать у окошка выдачи междугородних посылок. Его логика заключалась в том, что в каждом городишке США только одно такое окошко и что каждый прохожий подскажет, как его найти.

Мне этот аргумент кажется несколько неубедительным. Более того, он не проходит экспериментальную проверку. Я загадал эту загадку нескольким людям, и никто из них не предложил почтовое отделение. Автор загадки ждал бы у окошка в полном одиночестве.



Нашим потерявшимся бессмертным еще сложнее, так как они ничего не знают о географии планеты, на которой находятся.

Идти вдоль береговой линии кажется куда более разумной стратегией. Большая часть человечества живет вблизи воды, и искать вдоль линии куда проще, чем прочесывать площадь. Если ваша догадка окажется неверной, вы потратите не так много времени, как если бы сначала отправились вглубь материка.

Чтобы обойти средний континент, потребуется около 5 лет (если исходить из типичного для Земли соотношения площади континентов и их береговой линии)[105].

Давайте предположим, что оба вы находитесь на одном континенте. Если вы оба идете против часовой стрелки, то сможете кружить вечно, так и не встретив друг друга. Это плохо.

Другой вариант – пройти полный круг против часовой стрелки, а затем подбросить монету. Если будет орел, снова сделать круг против часовой стрелки, если решка – по часовой. Если вы оба следуете одному алгоритму, вероятность встречи спустя несколько оборотов повышается.

Но предположение, что вы оба используете единый алгоритм, вероятно, слишком оптимистично. К счастью, есть решение получше: будьте муравьем.

Вот алгоритм, которому я бы следовал (если когда-нибудь потеряетесь со мной на одной планете, имейте это в виду!):

Двигайтесь в случайном направлении, оставляя за собой цепочку камушков, каждый из которых будет указывать на следующий. После одного дня в движении три дня отдыхайте. Периодически ставьте дату где-то на цепочке камней, которые вы оставляете за собой. Неважно как – главное, чтобы способ все время был одинаковым. Можно, например, вырезать количество дней на камне или выложить число из тех же камней.

Если вы набредете на тропу, которая покажется вам более свежей, чем те, которые вы видели до сих пор, идите по ней как можно быстрее. Если вы потеряли тропу и не можете найти ее снова, продолжайте оставлять собственный след.



Не обязательно стремиться сразу найти второго участника – для начала найдите место, где он был какое-то время назад. Конечно, есть вероятность, что вы ходите друг за другом по кругу, но если вы будете двигаться быстрее, идя по чужому следу, чем оставляя собственный, вы найдете друг друга в течение нескольких лет или десятилетий.

А если ваш партнер действует иначе – например, сидит в некоей точке и поджидает вас, – вы хотя бы повидаете массу всего интересного.


Орбитальная скорость

ВОПРОС: А что, если космический аппарат замедлять при вхождении в атмосферу до скорости всего несколько км/ч при помощи посадочных двигателей, таких же, как у марсохода? Это избавило бы нас от потребности в тепловой защите?

– Брайан

ВОПРОС: Есть ли какой-нибудь способ вернуть космический аппарат в атмосферу, не прибегая к торможению о воздух? Это позволило бы обойтись без дорогой (и относительно ненадежной) теплоизоляции корпуса…

Кристофер Мэллоу

ВОПРОС: Можно ли поднять маленькую ракету с полезной нагрузкой на такую высоту в атмосфере, что для достижения второй космической скорости ей было бы достаточно совсем маленького ракетного двигателя?

– Кенни Ван де Мэле

ОТВЕТ: Все эти вопросы связаны с одной и той же идеей, которую я уже затрагивал раньше, но сейчас хочу остановиться на ней подробнее.

Выйти на орбиту тяжело не потому, что космос очень высоко.

Выйти на орбиту тяжело потому, что лететь надо очень быстро.

Космос выглядит не так:


Изображение в уменьшенном масштабе


Космос выглядит вот так:


Да, разумеется, это изображение в натуральную величину.


Космос находится на расстоянии всего 100 км от земной поверхности. Это далековато – не хотел бы я подниматься туда по лестнице, – но не так чтобы ужасно далеко. Если вы находитесь в Сакраменто, Сиэтле, Калькутте, Хайдарабаде, Пномпене, Каире, Пекине, центральной Японии, в центре Шри-Ланки или в Портленде, то до космоса от вас ближе, чем до моря.

Попасть в космос просто[106]. Конечно, не так чтобы вы могли доехать туда на своей машине, но вообще это совершенно не проблема. Можно доставить человека в космос на ракете размером с телеграфный столб. Самолет-ракетоплан Х-15 оказался в космосе, просто набрав очень большую скорость, а затем направившись вертикально вверх[107],[108].


Сегодня вам удастся попасть в космос…и быстро вернуться обратно


Но легко только попасть в космос. Проблема в том, чтобы там остаться.

Гравитация на околоземной орбите почти равна гравитации на поверхности Земли. Международная космическая станция вовсе не находится за пределами влияния гравитации – она испытывает примерно 90 % той силы, которая воздействует на нас на поверхности.

Чтобы не упасть обратно в атмосферу, станции надо лететь по орбите, причем очень, очень быстро.

Скорость, которая требуется, чтобы остаться на орбите, составляет примерно 8 км в секунду[109]. Лишь малая толика энергии ракеты расходуется на то, чтобы подняться из атмосферы, – большая ее часть тратится на достижение орбитальной (боковой) скорости.

Этот факт подводит нас к основной проблеме орбитальных полетов: достижение орбитальной скорости требует гораздо больше топлива, чем достижение орбитальной высоты. Чтобы разогнать корабль до 8 км/с, требуются очень мощные двигатели и много топлива. Достичь такой скорости и так достаточно сложно; достичь ее с дополнительным грузом топлива, которое позже потребуется для торможения при приземлении – практически нереализуемая задача[110].

Именно с этим невероятным расходом топлива связано то обстоятельство, что каждый космический корабль, входящий в атмосферу, тормозит при помощи тепловой изоляции, а не с помощью ракетных двигателей. Врезаться в воздух – самый практичный способ затормозить. (И возвращаясь к вопросу Брайана – марсоход «Кьюриосити» не был исключением. Хотя этот аппарат и использовал маленькие ракетные двигатели, чтобы «зависнуть» над самой поверхностью планеты, сначала он все же тормозил о марсианский воздух, чтобы сбросить большую часть скорости.)

А что вообще такое 8 км/с? Насколько это быстро?

Думаю, что одна из причин некоторого недопонимания в вопросе космических скоростей заключается в оптической иллюзии: когда мы видим съемку астронавтов на орбите, нет ощущения, что они так уж быстро двигаются, – кажется, будто они медленно парят над красивым голубым шаром. Но 8 км/с – это огромная скорость. Если вы посмотрите в небо незадолго до заката, иногда можно увидеть, как мимо проплывает МКС… а затем, спустя полтора часа, вы снова увидите, как она пролетает над вами[111]. За эти 90 минут станция облетела весь земной шар.

МКС движется так быстро, что, если бы вы выстрелили из ружья, стоя на краю футбольного поля, то станция пролетела бы до другого конца поля прежде, чем ваша пуля пролетела бы 9 метров.

Давайте представим себе, как бы это выглядело, если бы вы обходили Землю спортивным шагом на скорости 8 км/с.

Для большей наглядности в представлении скорости вашего путешествия давайте использовать музыку, чтобы отмечать течение времени[112]. Предположим, вы включили песню I’m Gonna Be 500 Miles («Я прошел бы 500 миль»), записанную группой The Proclaimers в 1988 году. Темп этой песни – примерно 131,9 удара в минуту, так что представьте себе, что с каждым ударом вы перемещаетесь на 3,6 км вперед. За то время, которое потребуется, чтобы спеть первую строчку припева, можно пройти от статуи Свободы до Бронкса.

Вы будете перемещаться со скоростью при мерно 15 станций метро в минуту.



Потребуется меньше трех строчек припева (16 ударов), чтобы пересечь пролив Ла-Манш.

Кстати, продолжительность этой песни связана с любопытным совпадением. Песня I’m Gonna Be… длится 3 минуты и 30 секунд, а МКС движется со скоростью 7,66 км/с. Это значит, что, если астронавт на МКС слушает эту песню, то за время, пока она звучит…



…он пролетит почти 1000 миль.

Пропускная способность FedEx

ВОПРОС: Когда – если это вообще возможно – пропускная способность Интернета сравняется со скоростью экспресс-почты FedEx?

– Йохан Обринк

Никогда не стоит недооценивать пропускную способность фургона, нагруженного магнитными лентами и мчащегося по шоссе.

Эндрю Тэненбаум, 1981

ОТВЕТ: Если вы хотите передать несколько сот гигабайт данных, то отправить жесткий диск с информацией посредством экспресс-почты FedEx получится быстрее, чем переслать ее по Сети. Идея эта не нова – часто такой способ передачи называют «дискетной сетью» или «флоппинетом», – и даже Google пересылает большие массивы данных внутри компании именно так.

Но всегда ли это будет быстрее?

Компания Cisco оценивает нынешний мировой интернет-трафик примерно в 167 терабит в секунду. У FedEx есть 654 самолета общей грузоподъемностью в 12 000 тонн в день. Жесткий диск ноутбука (SSD) весит примерно 78 граммов и может хранить до терабайта информации.

Это значит, что FedEx способен передавать 150 экзабайт в день, или 14 петабит в секунду, – почти в тысячу раз больше, чем современные возможности Интернета.

Если деньги для вас не проблема, то вот в коробку для обуви влезет примерно 10 кг дисков – на них поместится немалая часть Интернета.



Можно повысить плотность данных, используя карты памяти microSD.



Эти флэшки размером с ноготь обладают информационной плотностью до 160 терабит на килограмм, то есть весь авиапарк FedEx, нагруженный картами microSD, может пересылать до 177 петабит в секунду, или 2 зеттабайта в день, – в тысячу раз больше, чем нынешний интернет-трафик. (Инфраструктура была бы довольно любопытной – Google пришлось бы построить огромные склады специально для операций с картами памяти.)

По оценкам Cisco, интернет-трафик растет примерно на 29 % в год. С этой скоростью мы достигнем уровня FedEx в 2040 году. Конечно, объем данных, которые можно уместить на одном носителе, к тому моменту тоже возрастет. Единственный способ догнать FedEx – если скорость передачи данных будет расти быстрее, чем вместимость систем хранения. На первый взгляд это кажется маловероятным, так как передача и хранение данных связаны напрямую – вся эта информация откуда-то берется и куда-то направляется, – но нет мы не можем точно предугадать тенденции.

Хотя FedEx – достаточно мощная компания, чтобы не отставать от Интернета в ближайшие десятилетия, нет никаких технических причин, мешающих создать технологию подключения с более высокой пропускной способностью. Уже существуют экспериментальные волоконные кабели, которые могут передавать до петабита в секунду. 200 таких кабелей обгонят FedEx.

Если нанять всю индустрию грузоперевозок США для транспортировки SD-карт, общая пропускная способность достигнет 500 экзабит – половины цеттабита – в секунду. Чтобы достичь того же через Сеть, потребуется полмиллиона таких кабелей на петабит.

В общем, похоже, что Интернет никогда не обгонит «флоппинет». Однако потенциально безграничная пропускная способность FedEx-Интернета имеет свою цену: нам придется смириться с задержкой сигнала на 80 000 000 миллисекунд.


Свободное падение

ВОПРОС: С какой точки земного шара нужно спрыгнуть, чтобы как можно дольше находиться в свободном падении? А если при этом надеть вингсьют?

– Даш Шриватса

ОТВЕТ: Самый высокий отвесный обрыв на Земле – это гора Тор (Канада), имеющая такую форму[113]:



Чтобы сделать этот сценарий чуть менее ужасным, давайте представим, что под обрывом находится яма, заполненная чем-то мягким – например, сахарной ватой, – чтобы вы могли безопасно приземлиться.


Сработает ли это? Читайте в следующей книге…


Человек, падающий, вытянувшись, как ныряльщик, обладает предельной скоростью около 55 м/с. Потребуется несколько сотен метров, чтобы так разогнаться, поэтом это расстояние вы пролетите за 26 секунд с небольшим.

Что можно сделать за 26 секунд?

Для начала, этого достаточно, чтобы пройти всю первую игру Super Mario World 1–1 (если вы идеально рассчитали время и срезали путь через трубу).

Этого также достаточно, чтобы пропустить телефонный звонок. У американского телефонного оператора Sprint время, после которого звонок переключается на автоответчик, составляет 23 секунды.

Если кто-то позвонит вам и телефон начнет звонить в момент вашего прыжка, за три секунды до того, как вы достигнете дна, включится автоответчик.



С другой стороны, если бы вы спрыгнули с утесов Мохер (Ирландия), имеющих высоту 210 м, вы бы падали всего 8 секунд (или чуть больше при достаточно сильных восходящих потоках воздуха). Это не очень долго, но, если верить Ривер Тэм[114], этого времени хватит, чтобы полностью вас обескровить при наличии соответствующего оборудования.

До сих мы предполагали, что вы падаете вертикально. Но это совершенно необязательно.

Даже без спецоборудования опытный скайдайвер – после того как он или она разгонится до максимальной скорости – может падать под углом почти 45 градусов. Падая под таким углом, можно существенно продлить свой прыжок.


АААААААААААААААААААААААААААААА – вдох – АААААААААААААААААААААААААААААААААААА


Сложно сказать, насколько именно: помимо местного рельефа, это сильно зависит от одежды, которую вы выберете. Как написано в комментарии к статье «Википедии», посвященной рекорду по прыжкам в бейсджампинге,

Рекорд самого долгого падения без вингсьюта сложно рассчитать, поскольку граница между вингсьютами и джинсами оказалась размытой после появления более продвинутых моделей последних.

Тут-то мы и рассмотрим вингсьют – нечто среднее между спортивными штанами-«парашютами» и настоящим парашютом.

Вингсьют позволяет падать гораздо медленнее. Один энтузиаст выложил в сеть результаты серии прыжков, которые показывают, что человек в вингсьюте может снижаться со скоростью всего 18 м/с – большой прогресс по сравнению с 55 м/с в свободном падении.

Даже если исключить горизонтальную составляющую полета, это затянет наше падение больше чем на минуту. Теперь времени хватит на блиц-партию в шахматы. А еще его хватит на то, чтобы спеть первый куплет (весьма уместной в данном случае) песни группы REM под названием It’s the End of the World as We Know It («Это конец мира, каким мы его знаем»), а вслед за этим – кусок (менее уместной) песни Wannabe группы Spice Girls (тот, что ближе к концу).



Если выбрать еще более высокие скалы, которые теперь стали доступны для прыжка благодаря возможности горизонтального полета, у нас будет еще больше времени.

Есть много точек, с которых, вероятно, можно совершить очень долгий прыжок в вингсьюте. Например, восьмитысячник Нангапрабат (Пакистан), может похвастаться достаточно крутой стеной высотой более 3 км. (Как ни удивительно, вингсьют отлично функционирует даже в такой разреженной атмосфере, хотя пилоту понадобилась бы кислородная маска и летел бы он чуть быстрее, чем обычно.)

На данный момент рекорд по самому долгому прыжку в винг-сьюте принадлежит американцу Дину Поттеру, который спрыгнул с горы Айгер (Швейцария) и пролетел 3 минуты и 20 секунд.

Что можно сделать за три минуты и двадцать секунд?

Представим, что мы уговорили прыгнуть Джоуи Честната и Такеру Кобаяси – чемпионов мира по скоростному поеданию пищи. Если нам удастся найти способ, который позволит управлять вингсьютом и одновременно быстро есть, и Джоуи и Такера спрыгнут с Айгера, то они смогут – теоретически – съесть на двоих 45 хот-догов, прежде чем достигнут земли…



…а это бы значило, как минимум, что они побили один из са мых странных рекордов в мире.

Странные (и тревожные) вопросы из папки «Входящие» сайта «А что, если?»

ВОПРОС: Можно ли пережить цунами, погрузившись в бассейн?[115]

– Крис Муска

ВОПРОС: А что, если ваш парашют не раскрылся, но у вас есть с собой пружинка, обладающая идеальной массой и упругостью? Можно ли спастись, держась за ее конец?

– Варадараджан Шринивасан

Спартанские стрелы

ВОПРОС: В фильме «300 спартанцев» стрелы, выпущенные из луков, затмевают солнце. Возможно ли это на самом деле? И сколько стрел для этого понадобится?

– Анна Ньюэлл

ОТВЕТ: Сделать это непросто.

Попытка № 1

Лучник может выпустить от восьми до десяти стрел в минуту. С точки зрения физики лучник – это генератор стрел с частотой в 150 миллигерц.



Каждая стрела проводит в воздухе всего несколько секунд. Если среднее время полета стрелы составляет примерно три секунды, то примерно у половины лучников в любой выбранный момент выпущенные ими стрелы будут находиться в воздухе.

Каждая стрела закрывает от поля битвы примерно 40 см? солнечного света. Поскольку лишь у половины лучников стрелы находятся в воздухе, каждый из них в каждый момент закрывает в среднем 20 см?.

Если лучники выстроены рядами, по два лучника на метр, по полтора метра между рядами, и всего есть 20 рядов (30 м), то на каждый метр строя…



…будет приходиться 18 летящих стрел.



18 стрел закроют всего 0,1 % солнечного света. Этот результат явно следует улучшить.

Попытка № 2

Для начала давайте более плотно поставим лучников. Если сделать плотность их строя равной плотности толпы на танцполе[116], мы сможем утроить количество лучников на квадратный метр. Конечно, стрелять при этом будет не слишком удобно, но я уверен, что они как-нибудь справятся.

Фронт лучников можно также удлинить до 60 м. Это дает нам плотность в 130 лучников на метр.

Как быстро они могут стрелять?

В режиссерской версии «Властелина колец» есть сцена, когда группа орков атакует Леголаса и он молниеносно прицеливается и стреляет, убивая каждого из врагов одним выстрелом, прежде чем кто-либо из них успевает до него добраться.

Орландо Блум, актер, игравший Леголаса, конечно, не умел стрелять так быстро. На самом деле он просто натягивал лук – стрелы в кадр добавили позже при помощи компьютерной графики. Поскольку такая скорость стрельбы кажется зрителям впечатляющей, но не невозможной, она может послужить удобной верхней границей для наших расчетов.

Давайте предположим, что мы натренировали наших лучников, и они могут стрелять со скоростью Леголаса: 7 стрел за 8 секунд.

В этом случае наш отряд лучников (выпускающий невероятные 339 стрел на метр) все еще закроет от поля битвы всего лишь 1,56 % солнечного света.

Попытка № 3

Давайте забудем о луках и раздадим нашим лучникам пулеметы Гатлинга, стреляющие не пулями, а стрелами. Если они будут выпускать 70 стрел в секунду, это дает нам 110 м? стрел на 100 м? поля боя! Превосходно.

Но есть проблема. Хотя общая площадь тени от стрел примерно равна 100 м?, тени отдельных стрел будут перекрывать друг друга.

Формула для расчета доли затенения земной поверхности большим количеством стрел, причем часть теней перекрывает одна другую, такова:



В любом случае, 110 м? стрел позволяют закрыть от солнца только 2/3 поля боя. Поскольку наши глаза оценивают яркость логарифмически, уменьшение яркости Солнца до одной трети от нормального уровня покажется легким потемнением, но уж точно не затмением.

Если взять еще менее реалистичную скорость стрельбы, то проблему можно решить. Пусть каждый пулемет выпускает 300 стрел в секунду – тогда они закроют от поля боя 99 % солнечного света.

Но есть способ проще.

Попытка № 4

Мы исходили из предположения, что Солнце находится прямо над нами. Именно так и показано в фильме. Но возможно, знаменитая угроза была связана с планом атаковать на рассвете.

Если бы солнце поднялось еще невысоко над горизонтом с востока, а лучники стреляли бы на север, тогда свет мог бы пройти через всю массу стрел, что теоретически позволило бы умножить эффект тысячекратно.



Конечно, ни одна из этих стрел даже случайно не попала бы в о вражеского воина. Но будем честны: было сказано только, что стрелы «затмят солнце». Не было ни слова о том, что эти стрелы в кого-то попадут.

И кто знает, может, если правильно выбрать врага, то больше ничего и не требуется.


Осушить океаны

ВОПРОС: А что, если бы на дне Бездны Челленджера[117] образовался круглый портал радиусом 10 м, ведущий прямо в открытый космос? Как менялась бы Земля по мере вытекания Мирового океана?

– Тед М.

ОТВЕТ: Сразу хочу отметить следующее: по моим приблизительным оценкам, если бы в этом сточном отверстии застрял затонувший авианосец, давления было бы достаточно, чтобы смять его и засосать внутрь. Ничего себе…

Встречный вопрос: как далеко портал транспортирует земную воду? Если противоположный конец портала находится близко от Земли, то океаны просто обрушатся обратно в атмосферу, причем по ходу этого процесса вода нагреется и превратится в пар, который сконденсируется и выпадет в виде дождя. Одна только энергия, выделившаяся при этом в атмосферу, повлияет на наш климат весьма значительно, как и огромные высотные облака пара.

Так что давайте предположим, что противоположный конец нашего портала для сброса океанов расположен подальше – например, на Марсе. (Я лично предлагаю расположить его прямо над зондом «Кьюриосити» – тогда у марсохода, наконец, появятся неопровержимые доказательства наличия воды на Красной планете.)

Что случится с Землей?

Ничего особенного. Чтобы осушить Мировой океан, потребуются сотни тысяч лет.

Даже если мы сделаем наше отверстие размером с баскетбольную площадку и вода будет вытекать через него с невероятной скоростью, океаны поистине огромны. Вначале уровень воды падал бы меньше чем на сантиметр в день. Даже более или менее приличного водоворота на поверхности не получилось бы – для этого отверстие слишком маленькое, а океан слишком глубокий. (По той же причине водоворот в ванне образуется только после того, как из нее уйдет больше половины воды.)

Но предположим, что мы ускорим вытекание, проделав не один, а множество стоков[118], тогда уровень воды станет падать быстрее.

Давайте посмотрим, как будет меняться карта.

Вот как она выглядит вначале:


Это проекция Плате-Карре (см. http://xkcd.com/977).


Вот карта после того, как уровень океана упадет на 50 м:



В общем и целом похоже на предыдущий вариант, но есть несколько небольших изменений. Шри-Ланка, Новая Гвинея, Великобритания, острова Ява и Борнео теперь имеют сухопутное сообщение с соседями. И после 2000 лет, в течение которых голландцы сдерживали напор моря, Нидерланды наконец-то оказались на высоте, в сухости и безопасности. Теперь, когда постоянная угроза катастрофического наводнения миновала, Голландия может направить высвободившуюся энергию на экспансию. Голландцы немедленно расширяют свою территорию, присвоив только что показавшиеся из-под воды земли.



Когда уровень моря падает на 100 метров, из-под воды появляется огромный новый остров у берегов Новой Шотландии – раньше он был Большой Ньюфаундлендской банкой.

В этот момент мы замечаем нечто странное – высыхают не все моря. Уровень Черного моря, например, понизится совсем чуть-чуть, а потом оно вообще перестанет уменьшаться. Дело в том, что по мере падения уровня воды некоторые внутренние моря будут отрезаны от Мирового океана и стока в Тихом океане. В зависимости от особенностей морского дна в том или ином месте потоки воды могли бы прорезать в дне более глубокий канал и течь дальше. Но все больше участков океана будут окружены сушей и перестанут высыхать.



На уровне минус 200 м карта выглядит довольно непривычно. Появляются все новые острова. Индонезия превращается в большую кляксу. Нидерланды уже захватили большую часть Европы.



Япония превратилась в перешеек, соединяющий Россию и Корейский полуостров.

Новая Зеландия обзаводится новыми островами. Нидерланды распространяются на север.



Новая Зеландия резко увеличивается в размере. Северный Ледовитый океан теперь изолирован, и уровень воды в нем перестает падать. Нидерланды выходят на перешеек, который теперь ведет в Америку.



Уровень моря понизился на 2 км. Тут и там продолжают возникать новые острова. Карибское море и Мексиканский залив отрезаны от Атлантического океана. Даже не знаю, как назвать то, что творится с Новой Зеландией…



Мировой океан обмелел на 3 километра. Многие вершины срединно-океанических хребтов – самых протяженных горных цепей на Земле – выступили на поверхность. Из воды поднимаются обширные участки новой суши.



К этому моменту большая часть океанов разделилась на изолированные участки, которые перестали мелеть. Точное расположение и размеры этих внутренних морей изобразить сложно: это лишь приблизительная картина.



Вот как выглядит карта после того, как наш портал завершит свою работу. Как ни странно, немало воды осталось, хотя большая ее часть представляет собой довольно мелкие моря, лишь несколько впадин имеют глубину 4–5 км.

Исчезновение примерно половины Мирового океана серьезно изменит климат и экосистемы, но каким именно образом – предсказать трудно. Уверенно можно сказать одно: это приведет к коллапсу биосферы и массовому вымиранию видов на всех ее уровнях.

Возможно – хоть и маловероятно, – что люди выживут. Если это случится, мир будет выглядеть так:


Осушить океаны, часть II

ВОПРОС: А что, если мы действительно осушили бы Мировой океан и сбросили всю воду на марсоход «Кьюриосити»? Как менялся бы Марс по мере накопления на нем земной воды?

– Иэн

ОТВЕТ: В предыдущем ответе мы открыли портал на дне Марианской впадины и позволили океанам через него вытечь. Мы не особенно интересовались тем, куда именно они вытекают. Я выбрал Марс: марсоход «Кьюриосити» с таким усердием ищет следы воды, что мне захотелось упростить ему задачу.



«Кьюриосити» находится в ударном кратере Гейла, круглом углублении на поверхности Марса. В центре кратера возвышается гора Шарпа (гора Эолида).

На самом деле, на Марсе много воды, но только в виде льда. В жидком состоянии вода не может существовать здесь долго, потому что на Марсе слишком холодно и слишком мало воздуха. Если поставить на поверхность Марса кружку с теплой водой, вода попытается вскипеть, замерзнуть и испариться практически одновременно. Похоже, вода на Марсе готова находиться в любом состоянии, кроме жидкого.

Однако мы сбрасываем на Марс очень много воды и очень быстро (и вся она на несколько градусов теплее нуля), так что она не будет успевать замерзнуть, вскипеть или испариться. Если наш портал достаточно велик, то кратер Гейла скоро превратится в озеро – так же, как это произошло бы на Земле. Мы можем использовать отличные топографические карты Марса, созданные Геологической службой США, чтобы отследить движение воды.

Вот кратер Гейла в начале нашего эксперимента:



Вода продолжает прибывать, озеро заполняется, и «Кьюриосити» скоро будет погребен под сотнями метров воды:



В конце концов гора Шарпа уйдет под воду и на ее месте образуется озеро. Однако до того как вершина горы полностью скроется под водой, вода перельется через северный край кратера и потечет по песку марсианской пустыни.



Есть признаки того, что из-за периодических периодов сильной жары лед в марсианской почве иногда тает и тогда образуется жидкая вода. Когда это случается, струйка воды высыхает раньше, чем успевает куда-либо утечь. Однако в нашем распоряжении не струйка, а целый Мировой океан.



Водоемы на Северном плато:



Постепенно плато будет полностью покрыто водой:



Однако если внимательно посмотреть на карту регионов Марса, расположенных ближе к экватору (там много вулканов), мы увидим, что немалая часть суши все же находится далеко от воды:


[Проекция Меркатора, полюса не показаны.]


Откровенно говоря, я считаю эту карту довольно скучной – на ней мало что происходит. Это просто большой пустой кусок земли с океаном сверху.


Карта на двоечку.


Мы еще и близко не подошли к тому, чтобы перелить на Марс весь Мировой океан, но, хотя к концу нашего прошлого ответа на карте Земли оставалось немало синих пятен, оставшиеся моря были мелкими – большая часть объема океана была уже истрачена.

Марс гораздо меньше Земли, так что из того же объема воды получатся более глубокие моря.

К этому моменту вода заполняет долины Маринер, создавая береговые линии необычных очертаний. Карта уже не такая скучная, но окрестности великих каньонов демонстрируют довольно странные формы.



Вода добирается до марсоходов «Спирит» и «Оппортьюнити» и накрывает их.

В конце концов она прорывается в ударный кратер Эллады, самую низкую точку Марса.

На мой вкус, теперь соответствующая часть карты выглядит весьма неплохо.



Вода обильно растекается по поверхности, и на карте появляется несколько крупных островов (и великое множество мелких).



Вода быстро покрывает большую часть высоких плато, оставив только несколько островов.



И тут, наконец, поток останавливается: океаны Земли осушены.

Давайте приглядимся к основным марсианским островам внимательнее:



Все марсоходы давно погребены под толщей воды.

Вершины Олимпа и нескольких других вулканов останутся над водой. Как ни странно, они и близко не подошли к тому, чтобы стать подводными. Олимп все еще возвышается больше чем на 10 км над новым уровнем моря. На Марсе очень высокие горы.

Эти невероятные острова – результат того, что вода заполнила Лабиринт Ночи, загадочный лабиринт ущелий, происхождение которых до сих пор неизвестно.

Океаны на Марсе не протянут долго. Возможно, благодаря парниковому эффекту на Красной планете на какое-то время наступит потепление, но все же Марс слишком холодный. В конце концов океаны замерзнут, будут занесены пылью и постепенно сместятся в область вечной мерзлоты на полюсах.

Однако это займет немало времени, а до тех пор Марс будет чрезвычайно любопытным местом.

Если вспомнить, что у нас имеется портал, позволяющий перемещаться между планетами, то последствия очевидны:


Уникальные твиты

ВОПРОС: Сколько уникальных твитов можно написать на английском языке? Сколько времени потребуется населению Земли, чтобы прочесть их все вслух?

– Эрик Х., Хопарконг, Нью-Джерси

Далеко на севере, в земле под названием Свеи, стоит скала высотой в сотню миль и шириной в сотню миль. Раз в тысячу лет маленькая птица прилетает к скале, чтобы поточить клюв. Когда эта скала будет сточена, пройдет один день вечности.

Хендрик Виллем Ван Лон

ОТВЕТ: Длина твита составляет 140 символов. В английском алфавите 26 букв – то есть 27 символов, если считать пробел. Используя эти знаки, можно составить 27140 ~ 10200 возможных последовательностей.

Но «Твиттер» не ограничивается этими знаками. Можно поиграть с полным набором Unicode, где имеется более миллиона символов. «Твиттер» довольно сложно считает символы Unicode, но сумма возможных последовательностей может достичь 10800.

Конечно, почти все они будут бессмысленным набором букв и знаков из десятка разных языков. Даже если ограничиться латинским алфавитом, в получившихся последовательностях будут бессмыслицы вроде ptikobj. Но вопрос Эрика был о твитах, которые действительно несут какую-то информацию на английском языке. Сколько их может быть?

Это сложный вопрос. Первый порыв – позволить использовать только английские слова. Затем можно ограничить себя только грамматически правильными предложениями.

Но все равно возникают трудности. Например, Hi, I’m Mxyztplk – грамматически правильное предложение, если вас на самом деле так зовут. (Строго говоря, если подумать, оно грамматически правильное, даже если вы соврали.) Очевидно, не имеет смысла считать каждую фразу, начинающуюся со слов «Привет, я – …», отдельным предложением. Для англоязычного читателя Hi, I’m Mxyztplk практически неотличимо от Hi, I’m Mxzkqklt, и засчитывать оба не стоит. Зато Hi, I’m xPoKeFaNx, очевидно, отличается от первых двух, хотя xPoKeFaNx никак нельзя посчитать английским словом.

Итак, этот способ отбора, похоже, не работает.

К счастью, есть более удачный подход.

Давайте представим себе язык, в котором есть только два верных предложения, и каждый твит должен быть одним из этих предложений. Это:


В пятом стойле есть лошадь

и

В моем доме полно ловушек.


Тогда лента вашего «Твиттера» выглядела бы вот так:



Эти сообщения довольно длинные, но в них не очень много информации – они только показывают, выбрал человек сообщение про ловушки или про лошадь. По сути, это двоичный код – цифра 0 или цифра 1. Хотя здесь много букв, для читателя, владеющего этим языком, каждый твит содержит лишь один бит информации.

Этот пример подводит нас к важной идее о том, что информация фундаментально связана с неуверенностью получателя в содержании сообщения и его неспособностью предсказать это содержание заранее.

Клод Шэннон – который практически в одиночку изобрел современную теорию информации – придумал хитрый метод, чтобы измерять информационную содержательность языка. Он показывал группам людей образцы типичного английского текста, которые были произвольно оборваны, и предлагал угадать, какая буква последует дальше.


Наши города рискуют просто утонуть в информации!


Основываясь на частоте верных догадок – и сложном математическом анализе, – Шэннон определил, что информационная насыщенность типичного письменного английского составляет 1–1,2 бита на букву. Это значит, что хороший алгоритм сжатия должен позволять сжать английский текст в кодировке ASCII, в котором восемь бит на букву, примерно до одной трети изначального объема. И в самом деле, если применить к электронной книге в формате .txt хороший архиватор, примерно это и произойдет.

Если фрагмент текста содержит n бит информации, в определенном смысле это значит, что есть 2? сообщений, которые он может передавать. Конечно, не обошлось без математического жонглирования (в частности, в том, что касается длины сообщения и штуки под названием «расстояние единственности»), но суть в том, что предположительное количество различных осмысленных твитов на английском составляет 2140x1,1 = 2 x 1046, а не 10200 или 10800.

И сколько же времени потребуется миру, чтобы прочесть их все вслух?

Чтение 2 x 1046 твитов займет у человека порядка 1047 секунд.

Неважно, читает ли их один человек или миллиард, – этих твитов так много, что нельзя прочесть сколько-нибудь существенную их часть, даже если читать все то время, что существует Земля.

Давайте лучше вернемся к птичке, которая точит клюв о скалу. Предположим, что раз в тысячу лет птица откалывает от скалы маленький кусочек и, улетая, уносит на себе несколько десятков крупиц камня. (Нормальная птица оставила бы на скале вещества клюва, чем унесла бы камня, но в этой истории нет ничего нормального, так давайте просто продолжим в том же духе.)

Предположим, вы читаете твиты вслух по 16 часов в день, ежедневно. И за вашей спиной каждую тысячу лет прилетает птица и соскребает несколько невидимых крупиц с вершины скалы.

Когда скала будет сточена до основания, пройдет один день вечности.

Скала появляется снова, и цикл возобновляется еще на один день вечности. 365 дней вечности = каждый из них длиной 10?? лет – составляют год вечности.

Сотня лет вечности, за которые птица сточит 36 500 скал, составит столетие вечности.

Но столетия не хватит. Как и тысячелетия.

Чтобы прочесть все твиты, вам потребуется десять тысяч лет вечности.



Этого времени достаточно, чтобы увидеть, как разворачивается вся история человечества с момента изобретения письменности и до сего дня – если каждый день будет занимать столько времени, сколько нужно птичке, чтобы сточить скалу.



Кажется, что 140 знаков – это немного, но нам всегда будет что сказать.

LEGO-мост

ВОПРОС: А что, если построить мост от Лондона до Нью-Йорка из кирпичиков конструктора Lego? Существует ли вообще в мире столько деталей Lego?

– Джерри Петерсон

ОТВЕТ: Начнем с чуть менее амбициозной задачи.

Наводим мосты

В мире совершенно точно произведено достаточно кирпичиков Lego[119], чтобы соединить Нью-Йорк и Лондон. Если измерять средствами LEGO[120], то Нью-Йорк и Лондон разделяют 700 миллионов шипов. Это значит, что, если кирпичики расположить так…



…потребуется 350 миллионов кирпичиков, чтобы соединить два города. Такой мост будет крайне непрочен, и вряд ли он выдержит вес чего-то более серьезного, чем LEGO®-человечек[121], но для начала неплохо.

За много лет было произведено более 400 миллиардов деталей Lego[122]. Но какую часть из них составляют кирпичики, годные для постройки моста, а какую – маленькие шлемы для лего-человечков, вечно теряющиеся где-то в ворсе ковра?

Предположим, что мы строим мост с использованием самой распространенной детали LeGo[123], кирпичика 2 x 4 шипа.



Используя данные, который предоставил мне Дэн Богер, коллекционер Lego[124] и владелец большой базы данных по конструктору (http://thePeeron.com), я получил следующую приблизительную оценку: одна из 50– 100 деталей – это кирпичик 2 x 4. Значит, существует примерно 5–10 миллиардов кирпичиков 2 x 4: этого более чем достаточно для нашего моста шириной в один кирпичик.

Выдержать автомобиль

Конечно, если мы хотим, чтобы наш мост был пригоден для транспорта, нам придется сделать его чуточку пошире.

Возможно, имеет смысл сделать мост понтонным. Атлантический океан довольно глубокий [источник не указан], и мы хотели бы по возможности избежать необходимости строить сваи из Lego в 3 мили высотой.



При соединении кирпичики Lego не образ уют водонепроницаемых швов[125], а пластик, из которого их делают, плотнее воды. Это не проблема: если покрыть кирпичик слоем герметика, то он станет значительно менее плотным, чем вода.



Каждый вытесненный кубический метр воды позволяет мосту нести нагрузку в 400 кг. Типичная легковая машина весит чуть меньше 2000 кг, так что мосту потребуется как минимум 10 кубических метров Lego на каждую машину.

Если сделать мост толщиной в 1 метр и шириной в 5 метров, он должен держаться на плаву без особых проблем (хотя он погрузится в воду довольно глубоко) и будет достаточно прочным, чтобы по нему могли ездить автомобили.

Детали Lego довольно прочные: согласно одному репортажу BBC[126], можно поставить один на другой четверть миллиона кирпичиков 2 x 2, прежде чем самый нижний будет раздавлен.

Первая проблема нашего проекта заключается в том, что в мире недостаточно деталей Lego, чтобы построить мост такого размера. Вторая проблема – это океан.

Сверхмощные силы

В Северной Атлантике часто штормит. Хотя наш мост проходит вдалеке от самых быстрых частей Гольфстрима, он тем не менее подвергнется влиянию сильных ветров и волн.

Насколько крепким будет наш мост? Благодаря исследованиям Тристана Лостро из университета Южного Квинсленда, у нас есть данные о прочности соединений Lego при растяжении. Тристан, как и BBC, пришел к выводу, что кирпичики Lego поразительно прочные.

Оптимальный материал для нашего проекта – длинные и широкие пластины (10 x 6 шипов), уложенные «в перевязку»:



Мост вышел бы очень крепким – его прочность на растяжения была бы сопоставима с прочностью бетона, – но все же недостаточно крепким. Ветер, волны и течения выгнули бы центр моста в сторону, создав огромное натяжение.



Традиционный способ решения такой проблемы – зафиксировать понтонный мост, прикрепив его ко дну, чтобы тот не смог слишком сильно сдвинуться в ту или иную сторону. Если мы, помимо кирпичиков Lego, используем тросы, нам, возможно, удастся прикрепить нашу конструкцию к морскому дну[127].



Но проблемы на этом не заканчиваются. Будь наш мост перекинут через спокойный пруд, по нему могли бы ездить машины, но мост через океан должен быть достаточно высоким, чтобы оставаться над водой, когда о него разбиваются волны. Средняя высота волны в открытом океане может достигать нескольких метров, так что поверхность нашего моста должна возвышаться над водой на высоту как минимум 4 м.

Можно сделать нашу конструкцию более плавучей, добавив воздушные мешки и полости, но кроме того, придется сделать ее шире – иначе мост перевернется. Это значит, что потребуется больше якорей с поплавками, которые не позволят якорям затонуть. Но эти поплавки создадут еще больше натяжения, что еще больше усилит напряжение тросов и потянет наш мост вниз, а значит, потребуется еще больше поплавков…


Морское дно

Если же мы решим построить мост наш мосто на опорах, стоящих на морском дне, у нас возникнет еще несколько проблем. Под таким давлением мы уже не сможем позволить себе оставить внутри кирпичков воздушные полости для плавучести, а значит, конструкции придется выдержать как минимум свой собственный вес. Чтобы противостоять океанским течениям, нам пришлось бы сделать опоры как можно более мощными и толстыми. В конце концов выяснится, что мы, по сути дела, строим поперек океана дамбу.

Одним из побочных эффектов нашего строительства было бы прекращение циркуляции воды в Северной Атлантике.

Климатологи, с которыми я пытался обсудить эту проблему, сказали мне, что это, «вероятно, плохо»[128].

Кроме того, наш мост пересекает Среднеатлантический хребет. Если мерять в Lego, Атлантический океан расширяется со скоростью одного шипа в 112 дней. Нам придется предусмотреть на мосту компенсационные зазоры или периодически выезжать на середину моста и добавлять немного кирпичиков.

Стоимость

Кирпичики Lego делаются из пластика, который на момент сдачи этой книги в печать стоил примерно доллар за килограмм. Даже простейший мост (с учетом стоимости нескольких километров стальных тросов) будет стоить больше пяти миллиардов долларов.

Имейте в виду: общая стоимость рынка недвижимости Лондона составляет 2,1 миллиарда долларов, а стоимость трансатлантических перевозок – примерно 30 долларов за тонну.

Это значит, что за чуть меньшие деньги, чем стоимость нашего моста, можно скупить всю недвижимость в Лондоне, разобрать и переправить по частям в Нью-Йорк. Затем ее можно вновь собрать на каком-нибудь новом острове в Нью-Йоркской бухте и соединить оба города Lego-мостом попроще.



Возможно, у нас даже останется достаточно денег, чтобы купить набор Lego «Тысячелетний сокол» из 1238 деталей.

Самый долгий закат

ВОПРОС: Как долго можно наблюдать самый долгий закат, если использовать только асфальтированные дороги и не превышать скорость?

– Майкл Берг

ОТВЕТ: Прежде чем ответить на этот вопрос, надо определиться, что именно мы называем закатом.




Закат начинается в ту секунду, когда Солнце касается горизонта, и заканчивается, когда оно совершенно исчезает. Если Солнце касается горизонта и вновь поднимается, закат дисквалифицирован. Чтобы закат был засчитан, Солнце должно зайти за некий идеальный горизонт, а не просто за ближайшую гору. Это, например, не закат, хотя выглядит похоже:



Считать это закатом нельзя, потому что, если бы разрешалось использовать случайные препятствия, то вы могли бы устраивать себе закат каждый раз, просто спрятавшись за углом.

Нам также нужно учесть рефракцию. Атмосфера Земли искажает солнечные лучи, так что когда солнечный диск на самом деле находится на уровне горизонта, нам кажется, что солнце все еще стоит над землей на высоте примерно в один диаметр диска. Обычно этот эффект учитывается при расчетах, и я его тоже учел.

На экваторе в марте и сентябре закат длится чуть дольше двух минут. Ближе к полюсам, например в Лондоне, он может занять 200–300 секунд. Короче всего он осенью и весной (когда Солнце над экватором), а длиннее всего – летом и зимой.

Если стоять на Южном полюсе неподвижно в начале марта, солнце весь день остается в небе и описывает полный круг, не касаясь горизонта. Где-то в районе 21 марта оно касается горизонта, и это можно считать единственным закатом в году. Он продолжается 38–40 часов, то есть солнце проходит по горизонту больше чем полный круг.

Но Майкл задал свой вопрос очень хитро. Он спросил, как долго может длиться закат, если использовать только асфальтированные дороги. Существует дорога, ведущая к исследовательской станции на Южном полюсе, но она выложена утрамбованным снегом. Ни у одного полюса нет асфальтированных дорог.

Ближайшая к полюсу дорога, которая годится на роль асфальтированной, – это главная дорога в Лонгйире на норвежском острове Шпицберген. (По взлетной полосе аэропорта Лонгйира можно было бы подъехать чуть ближе к полюсу, но если вы въедете на взлетную полосу на машине, у вас могут возникнуть проблемы.)

Лонгйир находится ближе к Северному полюсу, чем антарктическая станция «Мак-Мердо» – к Южному. К северу от Лонгйира есть еще несколько военных баз, исследовательских станций и рыбацких поселков, но в них нет ничего, напоминающего дороги, – только взлетные полосы, обычно выстланные гравием и снегом.

Если прокатиться по Лонгйиру[129], то самый долгий закат, который вы увидите, будет длиться без нескольких минут час. Но на самом деле неважно, умеете вы водить машину вы или нет: городок слишком мал для того, чтобы ваши перемещения играли какую-то роль.

Лучше уж отправиться на материк, где дороги длиннее. Если выехать откуда-нибудь из тропиков и все время ехать по асфальтированным шоссе, то самая северная точка, до которой можно доехать, – конец европейской трассы Е69 в Норвегии. Северную Скандинавию пересекает множество дорог, так что это, похоже, удачное место для старта. Но какую именно дорогу нам выбрать?

Интуитивно кажется, что нам нужно забраться так далеко на север, как только возможно. Чем ближе мы к полюсу, тем легче успевать за солнцем.

К сожалению, выясняется, что следовать за солнцем – плохая стратегия. Даже на широтах Норвегии солнце движется слишком быстро. В конце Е69, в самой далекой точке, до которой можно добраться, если ехать от экватора по асфальтированным дорогам, все равно придется ехать со скоростью, равной где-то половине скорости звука, чтобы не отставать от солнца (кроме того, Е69 ведет с юга на север, а не с запада на восток, так что в итоге вы все равно въедете в Баренцево море).

К счастью, есть более удачный способ.

Если вы находитесь в Северной Норвегии в тот день, когда солнце лишь касается горизонта и снова поднимается, то терминатор (так называется линия линия светораздела, отделяющая освещенную часть Земли от неосвещенной) перемещается вот так:



Не путать с Терминатором, который перемещается так:


Не могу сказать, от которого из терминаторов я предпочел бы убегать.


Стратегия долгого за ката довольно проста: дождитесь дня, когда терминатор практически дойдет до места, где вы находитесь. Сидите в машине, пока терминатор вас не достигнет, затем поезжайте на север, чтобы слегка его обгонять, – так долго, как сможете (зависит от устройства местной дорожной сети), затем сделайте разворот и поезжайте обратно на юг, достаточно быстро, чтобы проскочить мимо терминатора и скрыться под покровом ночи[130].

Как ни странно, эта стратегия равно успешно работает практически где угодно в пределах Полярного круга, так что этот долгий закат можно увидеть на многих дорогах Финляндии и Норвегии. Я поискал дороги, которые годятся для этих наблюдений, при помощи GPS-карт норвежских шоссе. Оказалось, при большом выборе траекторий и скоростей перемещения самый долгий закат стабильно длится около 95 минут – это прогресс по сравнению с 40 минутами, проведенными в неподвижности на Шпицбергене.

Но если вы застряли на Шпицбергене и хотите, чтобы закат – или рассвет – продлился чуть дольше, всегда можно попробовать крутиться вокруг своей оси против направления движения солнца[131]. Правда, это добавит земным часам лишь неизмеримо малую часть наносекунды. Но в зависимости от того, с кем вместе вы это проделываете…



…оно вполне может того стоить.

Случайный чих

ВОПРОС: А что, если набрать случайный телефонный номер и сказать: «Будьте здоровы»? Какова вероятность, что человек, который снял трубку, действительно только что чихнул?

– Мими

ОТВЕТ: Точные данные найти сложно, но, похоже, примерно один шанс из 40 000.

Прежде чем начать набирать номер, нужно помнить, что с вероятностью 1:1 000 000 000 человек, который сейчас снимет трубку, только что кого-то убил[132]. Возможно, вы не всем хотели бы пожелать здоровья.

Однако если учесть, что чихают люди чаще, чем совершают убийства[133], у вас все же больше шансов угодить на насморк, чем на убийство. Так что если вы детектив, я бы вам такую стратегию расследования не рекомендовал.


Себе на заметку: так и буду говорить, когда кто-нибудь чихнет.


В отличие от статистики убийств, статистика чихания не так часто привлекает внимание ученых. Самую приблизительную цифру привел некий врач, давший в интервью ABC News, оценку примерно в 200 чихов на человека в год.

Одно из немногих научных исследований частоты чихания – это изучение того, как чихают люди с искусственно вызванной аллергической реакцией. Чтобы оценить частоту чихания, мы можем опустить все серьезные медицинские данные, которые пытались собрать исследователи, и просто посмотреть на контрольную группу. Им не давали аллергены – они просто сидели в одиночестве в комнате, 176 сеансов по 20 минут[134].

Участники контрольной группы чихнули четыре раза за эти 58 часов[135], и если считать, что чихаем мы, только когда не спим, это дает нам примерно 400 чихов на человека в год. Поисковая машина «Академия Google» находит за 2012 год 5980 научных статей, где упоминается «чихание». Если половина из них была написана в США и у каждой в среднем указано четыре автора, значит, если позвонить по случайному номеру, существует вероятность 1 к 10 000 000, что вы попадете на кого-то, кто ровно в этот день опубликовал статью о чихании.

С другой стороны, всего примерно 50 человек ежегодно погибают от удара молнии в США. Это значит, что вероятность того, что вы позвоните кому-то из них спустя 35 секунд после того, как он был убит, составляет всего 1 к 10 000 000 000 000.



Наконец, давайте предположим, что в день, когда эта книга вышла, пять человек, прочитавших ее, решили в самом деле провести такой эксперимент. Если они будут звонить каждый день, то есть 1 шанс из 30 000, что в течение суток один из них услышит в трубке короткие гудки, потому что человек по ту сторону трубки тоже звонит случайному незнакомцу, чтобы сказать: «Будьте здоровы!»

И с вероятностью 1 из 10 000 000 000 000 они одновременно позвонят друг другу.



Но к этому моменту законы вероятности, скорее всего, уже взбунтуются, так что обоих собеседников поразит молния.

Странные (и тревожные) вопросы из папки «Входящие» сайта «А что, если?»

ВОПРОС: А что, если меня ударят ножом в спину или грудь? Каковы шансы, что ни один жизненно важный орган не будет задет и я выживу?

– Томас

ВОПРОС: А что, если прыгнуть на мотоцикле с трамплина? Насколько быстро надо ехать, чтобы успеть раскрыть парашют во время прыжка?

– Аноним

ВОПРОС: А что, если бы у каждого человека каждый день был один шанс из ста превратиться в индейку? А у каждой индейки – один шанс из ста превратиться в человека?

– Кеннет

Расширяющаяся планета

ВОПРОС: А что, если бы радиус Земли возрастал на 1 см каждую секунду? Сколько времени потребовалось бы человеку, чтобы заметить, что его вес увеличился (если считать, что в целом состав земной коры остается прежним)?

– Деннис О’Доннелл

ОТВЕТ: В настоящий момент Земля не расширяется.

Ученые давно предполагали, что подобное возможно. Задолго до того, как в 1960-х годах была подтверждена гипотеза континентального дрейфа[136], люди заметили, что очертания континентов таковы, что если эти части суши сблизить, они бы в некоторых местах почти идеально совпали. Выдвигались различные теории, позволявшие объяснить это, включая идею, согласно которой бассейны океанов сперва представляли собой трещины, образовавшиеся на изначально однородной поверхности Земли во время расширения планеты. Эта теория никогда не была широко распространена[137], но все еще периодически обсуждается на YouTube.

Чтобы избежать трещин в земной коре, давайте предположим, что вся Земля, от коры до ядра, расширяется равномерно. А чтобы избежать очередного осушения океанов, предположим, что Мировой океан тоже расширяется[138]. При этом все построенное людьми остается неизменным.

t = 1 секунда

Когда Земля начнет расширяться, вы почувствуете легкий толчок и можете даже на секунду потерять равновесие. Но ненадолго. Поскольку вы двигаетесь с равномерной скоростью 1 см/с, то не почувствуете никакого ускорения. За весь оставшийся день вы ничего особенного не заметите.

t = 1 день

За день Земля расширится на 864 метра.



Чтобы гравитация возросла заметно, потребуется много времени. Если вы весили 70 килограммов, когда расширение началось, вы будете весить 70,01 кг к концу первого дня.

Как насчет дорог и мостов? Они ведь должны будут в конце концов сломаться, да?

Не так быстро, как можно подумать. Однажды я слышал такую загадку:


Представьте, что вы плотно обвязали Землю веревкой.



Теперь представьте, что вам нужно поднять веревку на метр от поверхности Земли.



Насколько нужно удлинить веревку?


Хотя на первый взгляд кажется, что дополнительно потребуются километры веревки, ответ – всего лишь 6,28 м.

Окружность пропорциональна радиусу, так что, если увеличить радиус на одну единицу измерения, окружность увеличивается на 2? единиц.

Если отрезок длиной в 40 000 км увеличить на 6,28 м, это практически не будет заметно. Даже спустя еще один день буквально все сооружения, созданные людьми, легко справятся с появлением дополнительных 5,4 км. Даже бетон способен ежедневно растягиваться и сжиматься в еще большей степени.

После начального толчка одним из первых заметных эффектов будет тот факт, что у вас перестанет работать GPS. Спутники останутся примерно на тех же орбитах, но синхронизация системы GPS за несколько часов совершенно нарушится. Для работы GPS необходима очень высокая точность – это одна из немногих инженерных задач, для решения которой потребовалось применить и общую, и специальную теорию относительности.

Большая часть обычных часов продолжит работать нормально. Однако если у вас есть очень точные часы с маятником, можно заметить кое-что странное – к концу дня они будут спешить на три секунды.

t = 1 месяц

Спустя месяц Земля расширится на 26 км – примерно на 0,4 % радиуса, – и ее масса возрастет на 1,2 %. Но ускорение свободного падения на поверхности возрастет только на 0,4 %, а не на 1,2 %, так как оно пропорциональна радиусу[139].

Вы могли бы заметить разницу на весах, но это не так уж серьезно. В такой степени ускорение свободного падения и сейчас различается между разными городами. Это стоит помнить, если вы приобретаете электронные весы. Если их точность выше, чем два знака после запятой, вам нужно будет откалибровать их при помощи специальных эталонных гирь – сила гравитации на фабрике, где они были произведены, необязательно совпадает с силой гравитации у вас дома.

Хотя увеличения гравитации вы пока не заметите, расширение уже будет очевидным. Спустя месяц вы увидите трещины на длинных бетонных конструкциях и повреждения эстакад и мостов. Большая часть зданий, скорее всего, пока будет в порядке, хотя те из них, что построены на скальной породе, могут повести себя непредсказуемым образом[140].

К этому моменту космонавты на МКС уже начнут тревожиться. Мало того, что поверхность Земли (и верхняя граница атмосферы) будут приближаться к ним, возрастающая гравитация заставит станцию постепенно снижаться. Экипажу придется срочно эвакуироваться, так как всего через несколько месяцев станция сойдет с орбиты и войдет в атмосферу.

t = 1 год

Спустя год ускорение свободного падения вырастет на 5 %. Возможно, вы заметите, что прибавили в весе и уж точно вы видите, что уничтожены все дороги, мосты, линии электропередачи, спутники и подводные кабели. Ваши часы с маятником спешат уже на пять дней.

А как насчет атмосферы?

Если она не расширяется так же, как земля и вода, то давление воздуха начнет падать. Это связано с несколькими факторами. По мере возрастания гравитации воздух становится тяжелее. Но поскольку этот воздух распространен по большему объему, суммарно давление снизится.

С другой стороны, если атмосфера тоже расширяется, давление воздуха будет возрастать. Спустя годы вершина Эвереста уже не будет находиться в «зоне смерти». С другой стороны, поскольку вы станете тяжелее, а гора – выше, взойти на нее станет еще сложнее.

t = 5 лет

Спустя пять лет ускорение свободного падения возрастет уже на 25 %. Если вы весили 70 кг, когда расширение началось, сейчас вы весите уже 88 кг.

Большая часть инфраструктуры разрушена. Причиной этого стало расширение земли, на которой все это было построено, а не возросшая гравитация. Как ни странно, большая часть небоскребов неплохо продержится и под воздействием гораздо более высокой гравитации[141]. Для большей части из них определяющим фактором выживания станет не их собственная масса, а сила ветра.

t = 10 лет

Спустя 10 лет ускорение свободного падения вырастет на 50 %. Если атмосфера не будет расширяться, воздух к этому времени станет таким разреженным, чтобы дышать будет трудно даже на уровне моря. В обратном случае мы протянем еще немного.

t = 40 лет

Спустя 40 лет ускорение свободного падения на на поверхности Земли возрастет втрое[142]. К этому моменту даже самые физически сильные люди будут ходить с большим трудом. Дышать будет сложно. Деревья упадут. Злаки склонятся под собственным весом. По склонам практически всех гор сойдут гигантские оползни из-за стремления грунта к изменения угла естественного откоса.

Геологическая активность тоже возрастет. Большая часть земного тепла возникает в результате радиоактивного распада минералов в коре и мантии[143]. Земли стало больше – значит, больше стало тепла. Поскольку объем возрастает быстрее, чем площадь поверхности, объем тепла на квадратный метр возрастет.

Этого все равно недостаточно, чтобы значительно разогреть поверхность планеты – температура поверхности Земли в основном формируется под воздействием атмосферы и Солнца, – но станет больше извержений вулканов и землетрясений, а тектоническое движение плит ускорится. Аналогичная картина имела место на Земле миллиарды лет назад, когда радиоактивных материалов было больше и мантия была горячее.

Более активное движение плит может пойти на пользу нашему выживанию. Тектоника плит играет ключевую роль в стабилизации климата, а планеты меньшего размера, чем Земля (например, Марс), не обладают достаточным количеством внутреннего тепла, чтобы поддерживать долгосрочную геологическую активность, которая возрастает по мере увеличения размера планеты. Вот почему ученые считают, что экзопланеты большего размера, чем Земля («суперземли»), могут оказаться более подходящими для жизни, чем те, что равны ей по размеру.

t = 100 лет

Спустя 100 лет мы будем испытывать ускорение свободного падения примерно в 6g. Мы не только не сможем перемещаться в поисках пищи – наши сердца бьются недостаточно сильно, чтобы при такой гравитации доставить кровь к мозгу. Только маленькие насекомые (и морские животные) будут способны перемещаться. Возможно, люди могли бы выжить в специально построенных куполах с контролируемым давлением, где они могли бы двигаться, поскольку большая часть их тел была бы погружена в воду.



Дышать при этом будет сложно. Тяжело сделать вдох, когда на тебя давит толща воды, – именно поэтому дышать через трубки можно, только если легкие находятся совсем близко к поверхности.

За пределами куполов со сниженным давлением воздух будет непригодным для дыхания по другой причине. Где-то на уровне давления в шесть атмосфер самый обычный воздух становится ядовитым. Даже если бы нас не убили все остальные проблемы, спустя сто лет мы умерли бы от отравления кислородом. Но даже если не брать в расчет токсичность, дышать воздухом под таким давлением трудно, потому что он тяжелый.

Черная дыра?

Когда же Земля наконец станет черной дырой?

На этот вопрос трудно ответить, потому что наш сценарий исходит из предположения, что радиус постоянно возрастает, но плотность сохраняется, тогда как в черной дыре плотность также возрастает.

Динамика гигантских каменных планет редко становится объектом изучения, поскольку нет никаких очевидных предпосылок, при которых такие планеты могли бы сформироваться, – все столь крупные тела будут обладать достаточной гравитацией, чтобы притянуть достаточно водорода и гелия во время формирования планеты и стать газовым гигантом.

В какой-то момент наша растущая Земля достигнет точки, в которой дальнейшее прибавление массы будет заставлять ее сжиматься, а не расширяться. После этого произойдет коллапс, и она обратится в испаряющегося белого карлика или нейтронную звезду, и если ее масса продолжит возрастать, в итоге она станет черной дырой. Но пока до этого далеко…

t = 300 лет

Жаль, что человечество не просуществует так долго, потому что к этому моменту произойдет кое-что замечательное.

Земля будет продолжать расти, а Луна, как и все наши искусственные спутники, – постепенно приближаться к ней по спирали. Спустя несколько веков Луна окажется достаточно близко к разбухшей Земле, чтобы притяжение между Землей и Луной оказалось сильнее, чем лунная гравитация, сохраняющая наш спутник как единое тело.

Когда Луна перейдет эту границу, которую называют пределом Роша, она постепенно распадется на части[144], и у Земли на короткое время появятся кольца…


Невесомая стрела

ВОПРОС: А что, если выстрелить из лука в какой-то среде с нулевой гравитацией, но с такой же атмосферой, как на Земле? Сколько времени потребуется, чтобы сопротивление воздуха полностью остановило стрелу и она повисла бы в воздухе?

– Марк Эстано

ОТВЕТ: Ну, с кем из нас такого не случалось. Пробираешься, бывало, на нижний ярус огромной космической станции, чтобы подстрелить кого-нибудь из лука…



По сравнению с обычной задачкой по физике, тут условия нестандартные. Обычно мы учитываем гравитацию и игнорируем сопротивление воздуха, а не наоборот[145].

Как можно ожидать, сопротивление воздуха замедлит стрелу, и в итоге она остановится… но только улетев очень, очень далеко. К счастью, в течение большей части полета она ни для кого не будет представлять опасности.

Давайте рассмотрим подробнее, что произойдет.

Предположим, вы выпустили стрелу из лука со скоростью 85 м/с. Это примерно в два раза выше скорости мяча, брошенного бейсболистом высшей лиги, и чуть меньше, чем 100 м/с – такова скорость стрелы, выпущенной из профессионального композитного лука.

Стрела очень быстро замедлится. Сопротивление воздуха пропорционально квадрату скорости, то есть когда стрела летит быстро, она испытывает впечатляющее сопротивление.

Через десять секунд полета стрела пролетит 400 м, и ее скорость упадет с 85 до 25 м/с. 25 м/с – это скорость, с которой средний человек может бросить стрелу рукой.



На этой скорости стрела значительно менее опасна.

Мы знаем из охотничьих рассказов, что даже небольшое изменение скорости стрелы влияет на ее убойную силу – на то, какое животное она способна убить. Стрела весом 25 г, летящая со скоростью 100 м/с, подойдет для охоты на лося или черного медведя. На скорости 70 м/с она будет слишком медленной, чтобы убить хотя бы олененка – или, в нашем случае, космического олененка.



После того как скорость стрелы опустилась ниже этого значения, сама стрела уже не очень опасна… но до полной остановки еще далеко.

Спустя пять минут стрела пролетит где-то милю и замедлится до скорости пешехода. На этой скорости она будет испытывать очень незначительное сопротивление и просто поползет в воздухе вперед, постепенно замедляясь.

К этому моменту она уже пролетит больше, чем любая стрела на Земле. Луки высокого класса позволяют выстрелить на дистанцию в пару сотен метров, но мировой рекорд дальности для стрельбы из лука – чуть больше километра. Этот рекорд был установлен в 1987 году американским лучником по имени Дон Браун, который стрелял тонкими металлическими стержнями из угрожающего вида машины, которая лишь отдаленно напоминала обычный лук.



Минуты превращаются в часы, и стрела все больше и больше замедляется. Поток воздуха вокруг нее тоже становится иным.

У воздуха невысокая вязкость, иными словами, он не липкий. Это значит, что летящие предметы испытывают сопротивление, потому что им приходится выталкивать воздух со своего пути, а не из-за слишком прочных связей между молекулами воздуха. Это больше похоже на то, как если бы вы водили рукой в ванне, полной воды, чем в ванне, полной меда.



Спустя несколько часов стрела двигается уже так медленно, что это движение практически нельзя увидеть. Если предположить, что воздух почти неподвижен, к данному моменту он начнет напоминать скорее мед, нежели воду, и стрела очень медленно остановится.

На какой именно дистанции это произойдет, напрямую зависит от особенностей самой стрелы. Незначительные вариации ее формы могут серьезно повлиять на воздушные потоки вокруг стрелы на медленных скоростях. Но в любом случае, она должна пролететь как минимум несколько километров, возможно достигнув отметки в 5–10 км.

Проблема вот в чем. Сейчас существует только одно место, где имеется атмосфера, подобная земной, и одновременно нулевая гравитация. Это Международная космическая станция. А самый длинный модуль МКС, Кибо, имеет длину всего 10 м.

Так что проведи вы этот эксперимент в действительности, стрела пролетела бы не более 10 м. После чего либо остановилась бы… либо всерьез испортила чей-то день.


Земля без Солнца

ВОПРОС: А что, если Солнце внезапно погаснет? Что будет с Землей?

– Многие, многие читатели

ОТВЕТ: Это, вероятно, самый популярный вопрос в жанре «А что, если». Отчасти я до сих пор не отвечал на него потому, что такие ответы уже существуют. Если сделать в Google запрос: «А что, если солнце погаснет», тут же найдется много отличных статей, подробно разбирающих эту ситуацию.

Однако этот вопрос задают все чаще и чаще, так что я решил тоже ответить на него.

Если бы Солнце погасло…

Рис. 1. Солнце гаснет ?


Не будем спрашивать, как именно это произошло. Просто предположим, что мы заставили Солнце быстро миновать несколько стадий его развития, и оно превратилось в безжизненную холодную сферу.

Каковы будут последствия для нас здесь, на Земле?

Давайте посмотрим…

Понижается риск вспышек на Солнце

В 1859 году на Солнце произошла сильная вспышка и на Землю обрушилась геомагнитная буря. Магнитные бури индуцируют в проводах вихревые токи. К несчастью, к 1859 году люди уже успели опутать Землю телеграфными проводами. Буря вызвала токи, оборвала коммуникации и в нескольких случаях привела к пожарам.

С тех пор мы опутали Землю значительно бо?льшим количеством проводов. Если бы аналогичная буря разразилась сегодня, то по оценкам Министерства внутренней безопасности США, одним только Соединенным Штатам был бы нанесен ущерб в несколько миллиардов долларов – больше, чем от всех ураганов, когда-либо случавшихся в стране, вместе взятых. Если Солнце погаснет, эта угроза будет предотвращена.


Улучшенная спутниковая связь

Когда спутник связи проходит мимо Солнца, оно может заглушить его радиосигнал, вызывая сбой трансляции. Отключение Солнца решит эту проблему.


Улучшенная астрономия

В отсутствие солнечного света все обсерватории, расположенные на Земле, смогут работать круглосуточно[146]. Прохладный воздух снизит количество шумов в атмосфере, а значит, сократится нагрузка на системы адаптивной оптики, которая с ними борется. Это позволит получать более четкие снимки.


Стабильная пыль

С исчезновением солнечного света исчезнет эффект Пойнтинга – Робертсона, а значит, мы наконец-то сможем окружить Солнце кольцом из пыли, которое будет стабильно держаться на солнечной орбите. Не знаю, нужно ли это кому-нибудь, но никогда нельзя сказать наверняка – вдруг пригодится.


Более дешевая инфраструктура

Департамент транспорта США оценивает стоимость ремонта и обслуживания всех мостов США в 20 миллиардов долларов в год. Бо?льшая часть мостов США перекинута через водоемы – и когда в отсутствие Солнца водоемы замерзнут, мы могли бы сэкономить на ремонте мостов, просто положив асфальт поверх льда.


Меньше торговых издержек

Часовые пояса усложняют международную торговлю – непросто работать с партнером, чей рабочий день не совпадает с вашим. Если Солнце погаснет, необходимость в часовых поясах отпадет, и мы все могли бы перейти на время по Гринвичу. В результате возможен рост глобальной экономики.


Безопасность для детей

Согласно данным департамента здоровья штата Северная Дакота, младенцам меньше шести месяцев от роду нельзя находиться на солнце. В отсутствие солнечного света наши дети будут в большей безопасности.


Безопасность для пилотов

Многие люди чихают, оказавшись на ярком солнечном свету. Природа этого рефлекса неизвестна, но он может представлять опасность для военных пилотов во время боя. Если бы Солнце больше не светило, этой опасности удалось бы избежать.


Безопасный борщевик

Борщевик – удивительно неприятное растение. Его листья содержат соединения, которые называются фурокумарины и которые человеческая кожа впитывает без последствий… поначалу. Однако спустя некоторое время, когда кожа окажется на солнечном свету (даже спустя несколько дней или недель), фурокумарины могут вызвать тяжелый химический ожог – так называемый фитофотодерматит. Выключенное Солнце спасло бы нас от угрозы борщевика.



Итак, если бы Солнце погасло, во многих сферах нашей жизни произошли бы положительные изменения.

А есть ли какие-то негативные стороны у этого сценария?

Только одна: мы все замерзнем и умрем.


Правки в распечатанной «Википедии»

ВОПРОС: А что, если распечатать всю «Википедию» (скажем, английскую)? Сколько принтеров понадобится, чтобы успевать вносить в нее все изменения, появляющиеся в сетевой версии?

– Марейн Кёнингс

ОТВЕТ: Вот столько.


Если девушка пригласит вас к себе домой, и вы увидите в гостиной ряды принтеров, что вы подумаете?


Как ни странно, принтеров не особенно много! Но прежде чем вы соберетесь создать бумажную «Википедию», обновляющуюся в реальном времени, давайте посмотрим, что именно эти принтеры будут делать… и во сколько это обойдется.

Распечатать «Википедию»

Людям уже приходила в голову мысль распечатать «Википедию». Однажды студент по имени Роб Мэттьюз распечатал все избранные статьи (featured articles) «Википедии», и в итоге у него получилась стопка бумаги больше метра высотой. И это лишь маленький избранный отрывок – вся энциклопедия целиком была бы гораздо больше.

Пользователь Tompw создал инструмент, который рассчитывает текущий объем английской «Википедии» в печатных книгах.

Это многотомное издание заняло бы немало книжных полок, и успевать за правками было бы непросто.

Как успеть?

В настоящее время в английскую «Википедию» вносится 125 000–150 000 правок в день, то есть 90–100 в минуту.

Можно было бы попробовать рассчитать число слов в средней правке, но вероятность успеха тут минимальна. К счастью, этого и не нужно. Мы можем просто исходить из того, что каждая правка вынуждает нас распечатать соответствующую страницу заново. Многие правки на самом деле потребуют замены нескольких страниц, зато другие исправления отменяют предыдущие, что позволит нам просто вернуть на место некоторые из ранее распечатанных страниц[147]. Так что принцип «одна страница на одну правку» кажется вполне подходящим.

Типичная статья «Википедии» представляет собой сочетание иллюстраций, таблиц и текста, а значит, хороший струйный принтер сможет печатать 15 страниц в минуту. Таким образом, необходимо шесть постоянно работающих принтеров, чтобы успевать за правками.

Бумаги потребуется немало. Используя книгу Роба Мэттьюза как точку отсчета, я прикинул на салфетке, каков объем современной английской «Википедии». Получилось, что потребуется около 300 м? бумаги, чтобы распечатать весь материал – причем исключительно в текстовом формате.

Для сравнения: в ходе правки 300 м? придется печатать ежемесячно.

500 000 долларов в месяц

Шесть принтеров – это не так уж много, но они будут работать постоянно, а это недешево. Электричества они потребляют всего на несколько долларов в день, бумага стоит примерно 1 цент за лист, значит, на бумагу будет уходить где-то 1000 долларов в день. Придется нанять людей, чтобы круглосуточно присматривать за принтерами, но это обойдется дешевле бумаги.

Даже сами принтеры окажутся не такими уж дорогими, несмотря на то, что они будут изнашиваться угрожающе быстро.

Но вот чернильные картриджи станут настоящей катастрофой.

Чернила

Исследование, проведенное Quality Logic, показало, что для типичного струйного принтера стоимость чернил варьируется от 5 центов за страницу при черно-белой печати до 30 центов за страницу при печати фотографий. Это значит, что у вас будут уходить тысячи или десятки тысяч долларов в день на картриджи.



Лазерный принтер гораздо экономичнее, иначе спустя всего месяц или два этот проект уже будет стоить вам полмиллиона долларов.

Но это еще не самое страшное.

18 января 2012 года страницы «Википедии» стали черными в знак протеста против законов, ограничивающих свободу Интернета. Если когда-нибудь «Википедия» решит это повторить… что ж, придется раздобыть ящик маркеров и собственноручно закрасить все страницы.

В общем, я бы пока обходился электронной версией.


«Фейсбук» мертвецов

ВОПРОС: Когда в «Фейсбуке» станет больше страниц умерших людей, чем живых, и произойдет ли это вообще?

– Эмили Данхэм

«Наденьте наушники!» – «Не могу. Уши отвалились».


ОТВЕТ: В 2060-х или 2130-х годах.

В «Фейсбуке» покойников немного[148]. Все дело в том, что «Фейсбук» и его пользователи пока еще молоды. Средний возраст пользователя «Фейсбука» немного вырос за последние годы, но все же этой социальной сетью гораздо чаще пользуется молодежь.

Прошлое

Если исходить из роста аудитории «Фейсбука» и возрастной динамики его пользователей[149], где-то 10–20 миллионов пользователей уже успели завести аккаунт в «Фейсбуке», а потом умереть. На данный момент эти люди равномерно распределяются по всему возрастному спектру. Смертность среди молодежи ниже, чем среди людей в возрасте 60–70 лет, но молодые составляют немалую часть мертвецов на «Фейсбуке» просто потому, что среди пользователей их очень много.


Косплей пожилого Кори Доктороу: на нем надето то, что, с точки зрения людей будущего, он носил в прошлом.


Будущее

Около 290 000 американских пользователей «Фейсбука», вероятно, умерли в 2013 году. Для всего остального мира эта цифра в 2013 году составляет несколько миллионов[150]. Спустя всего семь лет количество умерших возрастет вдвое, а спустя еще семь – снова удвоится.

Даже если завтра «Фейсбук» прекратит регистрацию новых пользователей, в течение многих десятилетий количество смертей в год будет расти, пока будут стареть поколения, учившееся в университете в 2000–2020 годах.

Решающий фактор для определения того момента, когда мертвых станет больше, чем живых, – это способность Фейсбука привлекать новых живых пользователей (по возможности молодых) достаточно быстро, чтобы обогнать это цунами смерти.

«Фейсбук» в 2100-м

И это подводит нас к вопросу о будущем «Фейсбука».

Мы недостаточно давно наблюдаем социальные сети, чтобы уверенно предсказать, сколько просуществует «Фейсбук». Многие интернет-проекты начинались как вполне успешные, но затем утратили популярность, так что логично предположить, что «Фейсбук» постигнет та же участь[151].

В сценарии, согласно которому рыночная стоимость «Фейсбука» в ближайшее десятилетие падает и уже не возвращается на прежний уровень, линии графика пересекутся – то есть мертвых пользователей станет больше, чем живых, – около 2065 года.



Но это не единственный сценарий. Может быть, с «Фейсбуком» случится то же, что и с TCP-протоколом: он станет частью инфраструктуры, на которой будут основаны другие системы, и продолжит существовать по инерции.

Если «Фейсбук» останется с нами на века, то линии могут пересечься значительно позже, в середине 2100-х годов.

Но это кажется маловероятным. Ничто не длится вечно, и для компьютерных технологий быстрые перемены всегда были нормой. Мир усеян останками сайтов и технологий, которые десять лет назад казались незыблемыми.

Возможно, истина где-то посередине[152]. Нам просто придется подождать и узнать самим.

Судьба наших аккаунтов

«Фейсбук» вполне мог бы хранить все наши страницы с информацией вечно. Живые пользователи всегда будут генерировать больше информации, чем мертвые[153], и именно профили живых пользователей должны быть удобны для просмотра. Даже если аккаунты мертвых или неактивных пользователей составят большинство, они вряд ли когда-нибудь составят бо?льшую часть бюджета этой социальной сети.

Важнее будут наши собственные решения. Чего мы хотим от этих страниц? Если не потребовать, чтобы «Фейсбук» удалил страницу, он, предположительно, будет хранить всю информацию вечно. Если даже этого не сделает «Фейсбук», то сделают другие организации, занимающиеся сбором информации.

Сейчас ближайшие родственники покойного могут превратить его аккаунт в мемориальную страницу. Но есть много вопросов, связанных с паролями и доступом к личной информации, на которые пока не существует общепринятых ответов. Должны ли такие аккаунты быть общедоступными? Что должно оставаться в привате? Имеют ли близкие родственники право доступа к сообщениям? Можно ли оставлять комментарии на мемориальных страницах? Как бороться с троллями и хакерами? Как вообще взаимодействовать с аккаунтом умершего человека? Можно ли его, например, включить этого человека в список друзей?

Ответы на все эти вопросы мы сейчас ищем методом проб и ошибок. Смерть всегда была серьезным, сложным и эмоционально напряженным вопросом, и каждое общество отвечает на этот вопрос по-своему.

Основные кирпичики, из которых складывается человеческая жизнь, неизменны. Мы всегда искали пищу, учились, росли, влюблялись, боролись и умирали. В зависимости от страны, культуры и технологий мы вырабатываем разные поведенческие модели, связанные с этими видами деятельности.

Как и каждое поколение до нас, мы учимся играть в те же игры на нашем собственном поле. Пробуя и ошибаясь, мы вырабатываем новые социальные нормы для свиданий, споров, учебы и взросления в Интернете. Рано или поздно мы научимся и оплакивать мертвых.


Закат над Британской империей

ВОПРОС: Закатилось ли наконец солнце над Британской империей, и если да, то когда именно?

– Курт Амундсон

ОТВЕТ: Пока что еще не закатилось. Но только благодаря нескольким десяткам людей, живущим на территории площадью меньше «Диснейленда».

Самая большая империя в мире

Когда-то Британская империя охватывала весь земной шар. В результате появилось выражение «Империя, над которой никогда не заходит солнце», поскольку всегда где-то на ее территории был день.

Трудно сказать, когда начался этот бесконечный день. Процесс присоединения к собственному государству колонии (то есть территории, на которой уже живет другой народ) довольно сложен. По сути дела, британцы строили империю, просто плавая по морям и втыкая свои флаги на случайно встреченных берегах. Из-за этого сложно решить, когда та или иная часть империи была «официально» причислена к ней.


А это что за темное пятно?

Это Франция. Однажды мы и туда доберемся.


День, когда солнце перестало заходить над Британской империей, вероятно, наступил где-то в конце XVIII или начале XIX века, когда к ней были присоединены первые австралийские территории.

Британская империя распалась в середине XX века, но, как ни странно, солнце над ней по-прежнему не заходит.

14 территорий

У Великобритании есть 14 заморских (то есть находящимися вне Британских островов) территорий – это осколки Британской империи.



Многие колонии Британии, став независимыми государствами, присоединились к Содружеству наций. В некоторых из этих стран, например в Канаде и Австралии, королева Елизавета II официально считается сувереном. Однако это вполне независимые государства (просто у них по случайности одна и та же королева), и они не являются частью какой-либо империи[154].

Солнце никогда не заходит над всеми четырнадцатью британскими заморскими территориями разом (или даже тринадцатью, если не считать территорию в Антарктиде). Однако если Великобритания потеряет хотя бы одно из этих маленьких заморских владений, над ней впервые за два столетия зайдет солнце.

Около полуночи по Гринвичу солнце заходит над Каймановыми островами; оно взойдет над Заморской территорией Великобритании в Индийском океане только после часа ночи. На этот час крошечные острова Питкэрн на юге Тихого океана остаются единственной британской территорией, освещенной солнцем.

На островах Питкэрн живут несколько десятков человек – это потомки мятежников с корабля «Баунти». Острова стали печально известны в 2004 году, когда шесть человек, включая губернатора, – треть взрослого населения, – были осуждена за педофилию.

Но как бы ужасна ни была репутация этих островов, они все еще остаются частью Британской империи, и если британцы от них не откажутся, двухсотлетний британский день будет продолжаться.

Будет ли это продолжаться вечно?

Что ж, это не исключено.

В апреле 2432 года, впервые с тех пор, как на острове высадились мятежники «Баунти», случится полное солнечное затмение.

К счастью для империи, затмение произойдет в тот момент, когда солнце будет стоять над Каймановыми островами в Карибском море. Там полного затмения не будет, более того, солнце все еще будет светить и в Лондоне.

На самом деле ни одно полное затмение в ближайшую тысячу лет не произойдет над островами Питкэрн в момент, подходящий для того, чтобы прервать победную череду дней. Если Великобритания сохранит свою нынешнюю территорию и свои заморские территории, она может растянуть «британский день» на очень, очень долгое время.

Но не навсегда. Однажды – спустя множество тысячелетий – над островами Питкэрн случится затмение, и солнце наконец зайдет над Британской империей.


Помешивая чай

ВОПРОС: Я помешивал чай, и мне в голову пришла мысль: «Разве я не добавляю чашке кинетической энергии?» Мы помешиваем горячий чай, чтобы остудить его, но что, если мешать гораздо быстрее? Можно ли вскипятить чашку воды, просто помешивая ее?

– Уилл Эванс

ОТВЕТ: Нет.

Исходная идея логична. Температура – это просто кинетическая энергия. Когда вы помешиваете чай, вы повышаете его кинетическую энергию, которая должна быть на что-то потрачена. Поскольку чай не делает ничего увлекательного: не поднимается в воздух и не начинает излучать свет, энергия должна обращаться в тепло.


Я что – неправильно завариваю чай?


Вы не замечаете этого тепла по той причине, что добавляете его очень мало. Требуется очень много энергии, чтобы сильно нагреть воду, поскольку ее теплоемкость выше, чем у всех остальных привычных нам веществ[155].

Если вы хотите за пару минут довести воду комнатной температуры практически до кипения, вам потребуется большая мощность[156]:



Наша формула показывает, что, если мы разогреваем чашку воды за две минуты, нам нужен источник питания мощностью в 700 Вт. Обычная микроволновка потребляет 700–1100 Вт, и ей требуется где-то две минуты, чтобы нагреть кружку воды и заварить чай. Здорово, когда все получается!


Феликс Баумгартнер, британская версия


Нагревание чашки воды в микроволновке за две минуты при мощности в 700 Вт означает, что воде досталось очень много энергии. Когда вода падает с вершины Ниагарского водопада, она приобретает кинетическую энергию, которая уже у подножия водопада обращается в тепло. Но даже после падения с такой высоты вода нагревается только на долю градуса[157]. Чтобы вскипятить чашку воды, ее нужно уронить из точки, находящейся выше границы атмосферы.

Что общего у помешивания и работы микроволновки?

Судя по цифрам, которые я взял из инженерных отчетов по разработке промышленных миксеров, можно сказать, что энергичное помешивание чая нагревает его примерно на уровне одной стотысячной ватта. Это число, которым можно пренебречь.

На самом деле физический эффект перемешивания довольно сложен. Большая часть тепла покидает чашку благодаря конвекции воздуха, которая происходит над поверхностью жидкости, так что чай остывает сверху вниз. Помешивание поднимает горячую воду со дна чашки на поверхность, что дополнительно ускоряет этот процесс. Но происходят и другие явление – перемешивание повлияет и на воздух, который нагреет стенки чашки. Без точных измерений сложно сказать, что именно произойдет.

К счастью, у нас есть Интернет. Пользователь сети Stack Exchange под ником drhodes измерил скорость остывания чашки, сравнив ситуации, когда чай помешивают, когда его не помешивают и когда ложку периодически окунают в чашку и тут же вынимают из нее. Также Drhodes любезно выложил графики в хорошем разрешении плюс необработанные данные – о многих статьях в научных журналах такого не скажешь.

Вывод: не имеет особого значения, помешиваете ли вы чай, просто окунаете в него ложку или вообще ничего не делаете – чай остывает примерно с одинаковой скоростью (хотя в ситуации, когда в него окунали ложку, он остыл чуточку быстрее).

И тут мы возвращаемся к изначальному вопросу: можно ли вскипятить чай, если очень интенсивно его помешать?

Нет.

Первая проблема – это мощность. Необходимая мощность, 700 Вт, равна примерно одной лошадиной силе, так что если хотите вскипятить чай за две минуты, вам потребуется как минимум одна лошадь, которая его будет помешивать.

Можно обойтись и меньшей мощностью, если вы готовы разогревать чай очень долго, но если это время растянуть слишком сильно, чай будет остывать с той же скоростью, с которой вы его нагреваете.



Даже если бы вы смогли помешивать его с достаточной скоростью – 10 000 помешиваний в секунду, – в дело вступят законы гидродинамики. На таких скоростях движения жидкости начнется кавитация – вдоль пути ложки будет образовываться вакуум, и помешивание станет неэффективным.

И если вы помешиваете чай столь интенсивно, что происходит кавитация, площадь его поверхности начнет стремительно расширяться, после чего он за несколько секунд остынет до комнатной температуры.

Как бы сильно вы ни помешивали чай, теплее он не станет.


Все молнии мира

ВОПРОС: А что, если бы все молнии, которые случаются во всем мире за день, ударили в одно и то же место? Что бы случилось с этим местом?

– Тревор Джонс

ОТВЕТ: Говорят, молния не ударяет в одно и то же место дважды.

Так вот, это неправда. С точки зрения эволюции удивительно, что эта поговорка продолжает восприниматься на веру: логично предположить, что все, кто в это верили, постепенно были отфильтрованы из человеческой популяции.


Так работает эволюция, да?


Мои читатели часто интересуются, можно ли получать электричество из молний. На первый взгляд, это логично: в конце концов, молнии – это и есть электричество[158], и у разряда молнии действительно немалая мощность. Проблема в том, что не так-то просто заставить молнию ударить именно туда, куда вам нужно[159].

Типичный удар молнии имеет мощность, достаточную для того, чтобы в течение пары дней обеспечивать жилой дом электричеством. Это значит, что даже небоскреб Эмпайр-стейт-билдинг, в который молнии бьют где-то 100 раз в год, не сможет обеспечивать себя энергией исключительно за счет небесного электричества.



Даже в тех районах Земли, где молния наблюдается особенно часто, например во Флориде или Восточном Конго, энергия, которую земля получает от солнечного света, в миллион раз превышает энергию, получаемую от молний. Вырабатывать электричество из молний – это все равно, что построить ветряк, работающий на торнадо: это будет совершенно непрактично[160].

Молния Тревора

Но в сценарии Тревора все молнии мира бьют в одно и то же место. Это делает молниевую энергетику куда более перспективной!

Предположим, что определение «в одно и то же место» значит, что молнии бьют параллельно одна другой.

Лидер молнии, несущий в себе заряд, имеет диаметр в 1–2 см. Наш пучок содержит примерно миллион отдельных молний, так что общий диаметр пучка составит где-то 6 м.

Авторы научно-популярной литературы любят сравнивать чуть ли не все физические явления с мощностью бомбы, сброшенной на Хиросиму[161], так что давайте тоже пойдем по этому пути.

Мощность молнии составит примерно две бомбы. С практической точки зрения этой энергии достаточно, чтобы несколько миллионов лет питать плазменный телевизор и игровую приставку. Или, иначе говоря, чтобы удовлетворить энергетические запросы США… на пять минут.

Сам разряд будет иметь диаметр не больше центрального круга на баскетбольной площадке, но оставит воронку размером с целую площадку.



Внутри пучка молний воздух превратится в заряженную плазму. Свет и жар от разряда немедленно подожгут почву на несколько миль вокруг. Ударная волна повалит деревья и снесет здания. Так что можно сказать, что сравнение с Хиросимой вполне оправдано.

Можем ли мы себя от всего этого защитить?

Громоотводы

До сих пор ведутся споры по поводу механизма, благодаря которому работает громоотвод. Кое-кто считает, что это приспособление буквально «отгоняет» молнии, излучая заряд из земли в воздух, снижая таким образом перепад напряжения между облаком и землей и вероятность удара молнии. На данный момент Национальная ассоциация противопожарной защиты (NFPA) не поддерживает эту идею. Не знаю, что бы сказали в NFPA по поводу гигантской молнии Тревора, но громоотвод от нее все равно не спасет. Теоретически такой разряд мог бы пройти по медному штырю толщиной в метр и не расплавить его. К сожалению, когда разряд достигнет нижнего конца проводника, выяснится, что у почвы не такая хорошая проводимость, и взрыв расплавленного грунта в любом случае уничтожит ваш дом[162].


Молнии Кататумбо

Собрать молнии со всего мира в одном месте, очевидно, невозможно. Но как насчет всех молний какой-нибудь одной местности?

На Земле нет места, где молнии били бы постоянно, но в Венесуэле имеется область, в которой они бьют почти постоянно. Вблизи юго-западного берега озера Маракайбо наблюдается удивительный феномен: постоянные ночные грозы. В дву зонах, над озером и над его западным берегом, где грозы формируются практически каждую ночь. Во время этих гроз молнии могут вспыхивать раз в две секунды, и потому озеро Маракайбо – грозовая столица мира.



Если бы вам удалось направить все разряды одной грозовой ночи в Кататумбо по единственному громоотводу и использовать их, чтобы зарядить огромный конденсатор, в нем набралось бы достаточно мощности, чтобы игровая консоль и плазменный телевизор, подключенные к нему, работали примерно сотню лет.

Конечно, случись такое, известную поговорку пришлось бы переделать еще сильнее.


Самый одинокий человек

ВОПРОС: Насколько далеко от всех остальных живых людей оказывался когда-либо человек? Было ли ему одиноко?

– Брайан Дж. Маккартер

ОТВЕТ: Сложно сказать наверняка!

Наиболее вероятными подозреваемыми будут шесть пилотов командных модулей кораблей «Аполлон», которые оставались на орбите Луны во время лунных экспедиций: Майкл Коллинз, Дик Гордон, Стью Руса, Эл Уорден, Кен Мэттингли и Рон Эванс.

Каждый из этих астронавтов оставался в одиночестве в командном модуле, пока двое его товарищей высаживались на Луну. В самой высокой точке орбиты они находились примерно в 3585 километрах от своих товарищей.



А если взглянуть с другой точки зрения, то это как раз человечеству в тот день удалось оказаться максимально далеко от этих надоедливых астронавтов.

Казалось бы, лунные астронавты должны быть вне конкуренции, но все не так просто. Есть еще несколько кандидатов, которые недалеко от них ушли!

Полинезийцы

Не так-то просто оказаться в 3585 километрах от человеческого поселения[163]. Полинезийцы – первые люди, колонизировавшие острова Тихого океана. Кто-то из них, одинокий моряк, лодка которого затерялась в море, – например, во время шторма, – мог уплыть очень далеко от своих товарищей, но мы никогда не узнаем наверняка, случалось ли что-то подобное.

Когда острова Океании были колонизованы, на Земли стало куда сложнее найти место, где можно оказаться в изоляции – чтобы на 3585 километров в любую сторону не было ни одного человека. Теперь, когда даже в Антарктиде постоянно проживают люди, это почти наверняка невозможно.

Исследователи Антарктиды

Во время покорения Южного полюса некоторым путешественника почти удалось побить (будущий) рекорд астронавтов. Возможно, одному из них это действительно удалось. Очень близок к этому мог быть Роберт Фалькон Скотт – британский полярник трагической судьбы. Его экспедиция достигла Южного полюса в 1911 году – лишь для того, чтобы узнать, что норвежский исследователь Руаль Амундсен побывал здесь всего несколько месяцев назад. Разочарованный Скотт и его товарищи отправились обратно к побережью, но все они погибли на шельфовом леднике Росса.

Последний оставшийся в живых участник экспедиции мог бы на короткое время оказаться одним из самых одиноких людей на Земле[164]. Однако вокруг него, кто бы он ни был, на расстоянии 3585 км находилось некоторое количество людей, в частности другие исследователи Антарктиды, а также маори, жившие на Ракиуре (остров Стюарт) в Новой Зеландии.

Хватает и других кандидатов. Французский моряк XVIII века Пьер-Франсуа Перон утверждал, что его бросили на острове Амстердам на юге Индийского океана. Если это правда, он был близок к тому, чтобы побить (будущий) рекорд астронавтов, но, во-первых, остров Амстердам находится недостаточно далеко от Маврикия, юго-западного побережья Австралии и Мадагаскара, а во-вторых, Перон был не один: вместе с ним на острове находились еще четверо моряков.

Возможно, еще какой-то моряк, потерпевший кораблекрушение и дрейфовавший в шлюпке по южным морям, мог установить рекорд самого «удаленного от всех» человека в мире – мы этого никогда не узнаем. И пока не появится достоверных исторических свидетельств, шестеро астронавтов шести кораблей «Аполлон» – достаточно убедительные кандидаты.

И это подводит нас ко второй части вопроса Брайана – было ли им одиноко?

Одиночество

После возвращения на Землю пилот командного модуля «Аполлон-11» Майк Коллинз сказал, что совсем не чувствовал себя одиноким. Он так и написал в своей книге «Несущий огонь: Путешествия астронавта»:

Я вовсе не чувствую себя одиноким и покинутым. Я ощущаю, что я – часть того, что происходит на лунной поверхности. Но я не спорю: да, я сейчас один. Я сам по себе, и это ощущение лишь усиливается, потому что радиосвязь с Землей обрывается в ту секунду, когда я оказываюсь позади Луны. Сейчас я один, абсолютно один, абсолютно изолирован от всех известных форм жизни. Я – и все остальные. Если бы кто-то решил подсчитать, то с той стороны Луны оказалось бы три миллиарда плюс два человека, а с этой – один человек плюс неизвестно что еще…

Элу Уордену, пилоту командного модуля «Аполлон-15», одиночество даже понравилось:

Быть одному – это одно, быть одиноким – это другое. Я был один, но не одинок. Изначально я был пилотом истребителя в ВВС, затем летчиком-испытателем, тоже по большей части на военных самолетах, и я привык быть один. Я искренне этим наслаждался. Я больше не должен был говорить с Дэйвом и Джимом… По ту сторону Луны не надо было говорить даже с Хьюстоном, и это стало лучшим моментом за весь полет.


Интроверты поймут – самый одинокий человек в истории просто радовался нескольким минутам тишины и покоя.

Странные (и тревожные) вопросы из папки «Входящие» сайта «А что, если?»

ВОПРОС: А что, если бы все жители Великобритании собрались на одном из берегов своего острова и стали грести? Смогли бы они сдвинуть остров хоть немного?

– Эллен Юбэнкс

ВОПРОС: Возможны ли огненные торнадо?

– Сет Уишмэн

ДА. Огненные торнадо – это реальная штука, которая случается на самом деле. Не вижу смысла что-то добавлять.

Капля дождя

ВОПРОС: А что, если бы во время сильного дождя вся вода пролилась в виде одной крупной капли?

– Майкл Макнил

ОТВЕТ: В Канзасе – середина лета. Жарко и душно. Два старичка сидят на веранде в креслах-качалках.

На юго-западе горизонта появляются угрожающего вида облака. Они приближаются и разрастаются, принимают форму, напоминающую наковальню. Слышно, как звенит на веранде музыка ветра. Небо темнеет.


Влажность

В воздухе есть вода. Если бы вам удалось каким-то образом огородить от окружающего пространства столб воздуха от земли до верхней границы атмосферы, а затем охладить его, то влага, которая в нем содержится, выпала бы в виде дождя. Эта вода собралась бы в низу столба, и толщина слоя воды составила бы от нуля до пары десятков сантиметров. Это называется количеством осадков.



Обычно это количество составляет 1–2 см.

Метеорологические спутники анализируют количество этого водяного пара в каждой точке земного шара и на основе этого создаются невероятно красивые карты.

Представим, что наша буря протянулась во все стороны на 100 км и количество осажденной воды довольно велико – 6 см. Это значит, что объем воды в нашем шторме примерно равен:

(3,14 x 100? км/ч) x 6 см = 0,47 км?.

Эта вода весила бы 470 миллионов тонн (и это, по случайному совпадению, чуть меньше примерного суммарного веса всего вида Homo sapiens). В обычных условиях, часть такой воды выпала бы каплями в виде осадков, количество которых составило бы максимум 6 см.

В нашей буре вся вода собирается в одну гигантскую каплю – водяную сферу, имеющую около полукилометра в диаметре. Предположим, что что эта сфера формируется в паре километров над землей, как и обычный дождь.



Капля начинает падать.

Первые пять-шесть секунд ничего не видно. Затем основание облака начинает прогибаться вниз. На секунду нам кажется, что это формируется торнадо. Затем образовавшийся выступ стремительно увеличивается, и на десятой секунде из облака показывается низ капли.



Сейчас капля падает со скоростью 90 м/с. Яростный встречный ветер превращает ее поверхность в водяную пыль. Передний край капли обращается в пену, потому что воздух врывается в жидкость. Если бы капля падала достаточно долго, сила ветра в конце концов превратила бы единую каплю во множество дождевых капель.

Но до того как это произойдет, примерно спустя 20 секунд после возникновения капли, ее нижний край ударится о землю. Сейчас вода движется со скоростью больше 200 м/с. Прямо под точкой столкновения воздух не успевает убраться с ее пути, и сжатие нагревает его так быстро, что трава могла бы вспыхнуть, будь у нее достаточно времени.

К счастью для травы, жар продержится всего несколько миллисекунд, потому что его зальет холодная вода. К несчастью для травы, холодная вода движется со скоростью больше половины скорости звука.



Если бы все это время вы парили в центре этой сферы, то до этого момента вы не ощутили бы ничего необычного. В центре капли было бы довольно темно, но если бы вам хватило времени (и объема легких), чтобы проплыть несколько метров к краю капли, можно было бы разглядеть мутный дневной свет.

При приближении капли к земле нарастающее сопротивление воздуха привело бы к повышению давления, от которого у вас лопнут барабанные перепонки. Однако это уже не имеет значения: спустя несколько секунд при ударе капли о землю вас просто раздавит насмерть – ударная волна на время создаст давление большее, чем на дне Марианской впадины.

Вода врезается в землю, однако грунт не поддается. Давление заставляет каплю разлететься, образуя направленные во все стороны сверхзвуковые потоки, которые уничтожают все на своем пути.



Стена воды расширяется километр за километром и сносит на своем пути деревья и дома, сдирает верхний слой почвы. Дом, веранда и сидящие на ней старички мгновенно уничтожены. Все в радиусе нескольких километров полностью снесено, остается только лужа грязи поверх скальной породы. Вода продолжает движение, разрушая все на расстоянии 20–30 км. Лишь зоны, защищенные горными грядами, остаются в безопасности, и поток устремляется по естественным долинам и руслам.

Все находящееся за пределами этой территории пребывает в относительной безопасности, хотя через несколько часов в сотнях километров вниз по течению начнутся наводнения.

По миру распространяются новости о необъяснимой катастрофе. Людей охватывают шок и недоумение, и в течение некоторого времени каждое облако в небе будет вызывать массовую панику. Но пройдут годы, и катастрофа больше не повторится.

Попытки метеорологов выяснить, что случилось, так и не увенчаются успехом. В конце концов ученые сдадутся, а оставшийся неизвестным феномен назовут «последней каплей», поскольку, по словам одного исследователя, «это была лишь одна капля – зато какая!»

Экзамен наугад

ВОПРОС: А что, если бы все школьники, сдающие экзамен SAT, выбирали ответы наугад? Сколько стопроцентных результатов можно было бы ожидать?

– Роб Балдер ОТВЕТ: Ни одного.

SAT – стандартизированный тест, который сдают старшеклассники в Америке. Оценка выставляется таким образом, что в некоторых ситуациях ответ можно угадать. Но что, если бы вы попытались угадать все?

Экзамен SAT состоит не только из тестовых вопросов, но для простоты мы сфокусируемся именно на них. Предположим, что ваше сочинение написано отлично и в задачах на вычисления ошибок нет.

В SAT 2014 года было 44 тестовых вопроса по математике (количественный раздел), 67 – на анализ текста (качественный раздел) и 67 вопросов в письменном разделе.

На каждый вопрос предлагается пять вариантов ответа, так что при выборе наугад шанс ответить правильно составляет 20 %.

Вероятность угадать все 158 ответов:



Эту вероятность можно рассчитать по следующей формуле:


Это один шанс из 270 октодециллионов.


Если 4 миллиона 17-летних школьников сдают SAT и угадывают ответы случайным образом, можно быть практически уверенным, что стопроцентного балла не получит никто, ни в одном из разделов.

Насколько уверенным? Все очень просто: если все они каждый день будут проходить тест на компьютере миллион раз и если они будут продолжать в том же духе в течение пяти миллиардов лет, пока Солнце не превратится в красного гиганта и Земля не сгорит, то вероятность того, что кто-нибудь из них получит стопроцентный балл хотя бы в одном только разделе математики, составит около 0,0001 %.

Насколько это мало? Каждый год молния попадает примерно в 500 жителей США (я основываюсь на статистике, согласно которой в год происходит 45 смертей от молнии, а доказанная смертность при этой травме составляет 9 –10 %). Иными словами, для случайно выбранного американца вероятность получить удар молнии составляет примерно 1 к 700 000[165].

Это значит, что вероятность получить стопроцентный балл в SAT наугад меньше, чем вероятность того, что во всех ныне живущих экс-президентов США и актеров сериала «Светлячок» попадет молния… в один и тот же день!



Всем, кто в этом году сдает экзамен, – удачи!

Но ее одной будет недостаточно.

Нейтронная пуля

ВОПРОС: А что, если выстрелить в направлении центра Земли пулей, имеющей плотность нейтронной звезды? Уничтожит ли она Землю?

– Шарлотта Эйнсворт

ОТВЕТ: Пуля с плотностью нейтронной звезды будет весить примерно столько же, сколько Эмпайр-стейт-билдинг.

Неважно, из чего вы стреляете – эта пуля пройдет сквозь почву и пробьет земную кору, словно скальные породы сделаны из бумаги.

Рассмотрим две проблемы.

• Что станет с Землей после попадания пули?

• Если бы пуля осталась на поверхности, что происходило бы вокруг нее? И можно ли до нее будет дотронуться?


Сначала немного теории.

Что такое нейтронные звезды?

Нейтронная звезда – это то, во что превратилась гигантская звезда после коллапса под действием собственной гравитации.

Звезды существуют в состоянии равновесия. Их мощная гравитация всегда пытается подтолкнуть их к коллапсу, но это сжатие вызывает к жизни другие силы, которые удерживают звезду от коллапса.

В случае Солнца коллапсу противостоит жар ядерного синтеза[166]. Когда топливо для синтеза у звезды кончается, начинается сжатие (сложный процесс, во время которого происходит несколько взрывов), пока коллапс не остановится за счет закона квантовой механики, не позволяющего двум частицам вещества одновременно занимать одно и то же место[167].

Если звезда достаточно тяжелая, она преодолевает это квантовое давление и коллапс продолжается (при этом случается еще один, более сильный взрыв), чтобы стать нейтронной звездой. Если звезда еще более тяжелая, она станет черной дырой[168].

Нейтронные звезды – почти самые плотные объекты в мироздании (не считая плотности некоторых черных дыр). Их собственная огромная гравитация превращает их в подобие компактного «квантово-механического супа», отдаленно похожего на ядро атома, но размером с гору.

Наша пуля сделана из нейтронной звезды?

Нет. Шарлотта предложила пулю, которая такая же плотная, как нейтронная звезда, но не сделана из нейтронной звезды. Это хорошо, потому что из вещества такой звезды пулю не сделаешь. Если вещество нейтронной звезды вынести за пределы сопутствующей ему обычно колоссальной гравитации, оно тут же расширится, превратившись в невероятно раскаленное нормальное вещество, выделив при этом больше энергии, чем любое ядерное оружие.

Вероятно, именно поэтому Шарлотта предложила нам сделать пулю из волшебным образом стабильного вещества, у которого та же плотность, что у нейтронной звезды.

Что станет с Землей?

Можно представить себе, что мы выпускаем нашу пулю из пистолета[169], но, может быть, интереснее просто ее уронить? В любом случае, пуля ускорится, пробьет земную кору и устремится к центру Земли.

Землю это не уничтожит, но эффект будет весьма странный.

Когда пуля будет в нескольких метрах от поверхности, сила ее гравитации притянет огромное количество почвы, частицы которой будут хаотически колебаться вокруг падающей пули, разлетаясь во все стороны. После падения пули вы почувствуете, как вздрогнула земля, а на месте падения останется смятый, неровный кратер без входного отверстия.

Пуля пройдет прямо сквозь земную кору. Вибрация на поверхности быстро прекратится. Но глубоко внутри пуля в процессе падения будет сдавливать и испарять вещество мантии. Ударная волна отбросит вещество с пути пули, оставляя за ней след из раскаленной плазмы. Подземная комета! Этого в истории Вселенной еще не было.



В конце концов пуля застрянет в никелево-железном ядре Земли[170]. Энергия, которую получит при этом планета, будет гигантской в масштабах человека, но Земля ее практически не заметит. Притяжение пули повлияет только на вещество Земли в радиусе нескольких метров – хотя пуля достаточно тяжела, чтобы пробить кору, ее гравитации самой по себе недостаточно, чтобы разрушить прочные скальные породы.

Пуля будет навсегда погребена в Земле. И когда рано или поздно Землю поглотит стареющее разбухающее Солнце, пуля упокоится в его центре.

Солнце недостаточно плотное, чтобы самому стать нейтронной звездой. После того как наше светило поглотит Землю, оно пройдет еще через несколько стадий расширения и сжатия и в итоге станет маленьким белым карликом с пулей, все еще хранящейся в его центре. Однажды, далеко в будущем, когда Вселенная будет в тысячи раз старше, чем сейчас, этот белый карлик окончательно остынет и погаснет.

Таков ответ на вопрос о том, что бы случилось, если бы мы выпустили пулю в направлении центра Земли. Но что, если бы мы просто держали ее у поверхности?

Поставим пулю на прочный постамент

Сначала нам потребуется установить пулю на бесконечно прочный волшебный постамент, а постамент придется установить на столь же прочную платформу – достаточно большую, чтобы выдержать подобный вес. В противном случае вся конструкция уйдет в землю.



Если площадь постамента будет примерно равна площади городского квартала на Манхэттене, то он, скорее всего, сможет продержать пулю над землей хотя бы несколько дней, возможно – дольше. В конце концов, Эмпайр-стейт-билдинг – который весит столько же, сколько наша пуля, – стоит на аналогичной платформе, и хотя ему уже больше, чем несколько дней [источник не указан], под землю он еще не ушел [источник не указан].

Пуля не превратит атмосферу в вакуум. Она наверняка сдавит окружающий воздух и слегка его подогреет, но, как ни странно, не настолько, чтобы это стало заметным.

Можно ли до нее дотронуться?

Давайте представим, что вы это сделали.

Гравитация у этой штуки очень сильная, но не чрезмерная. Представьте, что стоите в десяти метрах от нее. На этом расстоянии вы почувствуете очень слабое притяжение в направлении постамента. Ваш мозг, не приспособленный к нестандартной гравитации, решит, что вы стоите на склоне.


Не надевайте ролики!


Этот мнимый склон становится все круче по мере того, как вы подходите к постаменту, словно земля под ногами постепенно опрокидывается.



Когда вы подойдете на расстояние нескольких метров, очень сложно будет удержаться на ногах и не покатиться к постаменту. Однако если за что-нибудь ухватиться – за какую-нибудь ручку или за столб, – можно подойти довольно близко.


Физики из Лос-Аламоса сказали бы, что мы «дергаем дракона за хвост».


Но я хочу его потрогать!

Если решите подойти достаточно близко, чтобы дотронуться до пули, вам потребуется за что-то очень крепко держаться. Честно говоря, лучше всего использовать ремни безопасности или хотя бы шейный корсет, потому что на расстоянии вытянутой руки от пули ваша голова будет весить примерно столько же, сколько маленький ребенок, и ваша кровь перестанет понимать, куда ей течь. Однако если вы – военный пилот, привыкший к перегрузкам, ваш организм с этим справится.



Кровь под таким углом будет приливать к голове, но дышать вы пока еще можете.

Когда вы вытянете руку, притяжение станет гораздо сильнее. 20 см от пули – это точка невозврата. После того как кончики ваших пальцев окажутся за этой чертой, рука станет слишком тяжелой, чтобы вы могли отдернуть ее (если вы умеете подтягиваться на одной руке, вам удастся подобраться еще чуть ближе).

На расстоянии нескольких сантиметров сила, действующая на ваши пальцы, станет огромной, их потянет вперед – с вами или без вас, – и кончики пальцев действительно коснутся пули (вероятно, в этот момент они будут вырваны из суставов, как и плечо).

Когда кончик пальца коснется пули, давление в пальцах станет слишком сильным и кожа порвется под напором крови.

Ривер Тэм из сериала «Светлячок» говорит, что человека можно полностью обескровить за 8,6 секунды при наличии «правильных условий».

Коснувшись пули, вы создали правильные условия.

Ваше тело удерживают ремни безопасности, а рука все еще прикреплена к туловищу – человеческое тело удивительно прочное, – но кровь вытекает из кончика вашего пальца гораздо быстрее, чем при обычном порезе, и возможно, необходимое время окажется даже меньше, чем 8,6 секунды.

Затем начинаются странности.

Кровь окружает пулю, формируя светящуюся темно-красную сферу, поверхность которой жужжит и вибрирует, и по ней бегут мелкие волны, слишком быстрые, чтобы их разглядеть подробно.


Но подождите!

Есть еще кое-что важное.

Кровь добавляет вам плавучести.

Сфера вокруг пули растет, а сила, воздействующая на вашу руку, становится слабее… потому что кончики пальцев, погруженные в кровь, плавают в ней. Кровь плотнее, чем плоть, а половина веса вашей руки в последние минуты приходилась на две последние фаланги ваших пальцев. Когда они погружены в кровь на несколько сантиметров, их вес значительно уменьшается.

Если дождаться момента, когда кровавая сфера станет глубиной в 20 см, и если ваше плечо до сих пор не повреждено, можно было бы даже вытащить руку.

Проблема в том, что эта модель требует в пять раз больше крови, чем всего содержится в человеческом теле.

Похоже, это конец.

Перемотаем на начало.

Как коснуться нейтронной пули: соль, вода и водка

Вы можете коснуться пули и выжить… но ее надо окружить водой.


ОБЯЗАТЕЛЬНО попытайтесь сделать это дома и пришлите мне видео.


А если хотите продемонстрировать, какой вы умный, можно опустить шланг в воду и заставить гравитацию пули всасывать воду самостоятельно.

Чтобы коснуться пули, наливайте воду на постамент, пока глубина со стороны пули не составит 1–2 метра. Вода примет приблизительно такую форму.


Если эти лодки утонут, не надо их спасать.


Теперь окуните в воду голову и руку.

Благодаря воде вы можете водить рукой вокруг пули безо всяких проблем! Пуля тянет вас к себе, но воду она притягивает с той же силой. Вода (как и плоть) практически не сжимается, даже при таком давлении, так что никакие жизненно важные функции не пострадают[171].

Однако возможно, что вам не совсем удастся коснуться пули. Когда ваши пальцы окажутся в нескольких миллиметрах от нее, сила гравитации приведет к тому, что огромную роль начнет играть плавучесть. Если рука чуть менее плотная, чем вода, она не сможет пройти этот последний миллиметр. Если она чуть плотнее, ее затянет вниз.

Вот для чего нам тут понадобятся соль и водка. Если вы почувствуете, что пуля притягивает кончики ваших пальцев, когда вы к ней тянетесь, это значит, что плавучесть ваших пальцев недостаточна. Добавьте в воду соли, чтобы вода стала плотнее. Если вы почувствуете, что пальцы соскальзывают с невидимого барьера вокруг пули, сделайте воду менее плотной, подлив в нее водку.

Если вам удастся добиться правильного баланса, можно коснуться пули и выжить.

Может быть[172].

Альтернативный план

Звучит слишком рискованно? Ничего страшного. Весь этот план: пуля, вода, соль и водка – можно рассматривать просто как рецепт самого сложного напитка в истории – коктейля «Нейтронная звезда».

Так что хватайте соломинку и пейте.



Но помните: если кто-то бросит вишенку в вашу «Нейтронную звезду» и вишенка уйдет на дно, не пытайтесь ее выудить. Ее больше нет.

Странные (и тревожные) вопросы из папки «Входящие» сайта «А что, если?»

ВОПРОС: А что, если я проглочу клеща, зараженного болезнью Лайма? Сможет ли кислота в моем желудке убить клеща?

– Кристофер Фогель

ВОПРОС: Если предположить, что резонансная частота пассажирского самолета постоянна, сколько кошек и на какой частоте должны замяукать, чтобы он упал?

– Бритни

15 баллов по шкале Рихтера

ВОПРОС: А что, если в Америке (скажем, в Нью-Йорке) случится землетрясение силой 15 баллов по шкале Рихтера? А 20 баллов? А 25?

– Алек Фарид

ОТВЕТ: Шкала Рихтера, которую сейчас заменила шкала магнитуды землетрясений[173], измеряет энергию, которая выделяется при землетрясении. Это шкала не ограничена, но поскольку обычно мы слышим о землетрясениях силой от 3 до 9 баллов, многие люди, скорее всего, считают, что она ранжирована от 1 до 10.



На самом деле десятка – не предел шкалы, но вполне могла бы им быть. Землетрясение в 9 баллов уже влияет на вращение Земли в степени, которую можно замерить. Два землетрясения мощностью 9 или более баллов, произошедших в этом столетии, изменили продолжительность суток (пусть и на мельчайшую долю секунды).

Землетрясение в 15 баллов привело бы к выделению примерно 10?? Дж энергии, что примерно равно гравитационной энергии, которая не позволяет Земле развалиться на части. Можно сказать, что «Звезда смерти» вызвала на Альдераане землетрясение в 15 баллов.



Теоретически на Земле могло бы случиться и более сильное землетрясение, но на практике это означало бы только, что расширяющееся облако обломков будет более горячим.

Солнце, гравитационная энергия которого выше, могло бы пережить землетрясение мощностью 20 баллов (хотя это почти точно спровоцировало бы вспышку сверхновой). Такова мощность самых сильных землетрясений в пределах известной нам Вселенной (они происходят в сверхтяжелых нейтронных звездах). Такой выброс энергии примерно соответствовал бы энергии одновременного взрыва собранных вместе водородных бомб, равных по объему Земле.

Мы много говорим о грандиозных и опасных событий, но как насчет «хорошего конца» шкалы? Может ли случиться землетрясение мощностью в 0 баллов?



Да! На самом деле, шкала Рихтера опускается «в хорошую сторону» даже ниже нуля. Давайте рассмотрим варианты землетрясений с низкой мощностью и представим себе, что случилось бы с вашим домом.


Мощность = 0

Команда «Даллас Ковбойз» с разбегу врезается в стену гаража вашего соседа.



Мощность = -1

Один футболист врезается в дерево в вашем саду.



Мощность = -2

Кошка падает с комода



Мощность = -3

Кошка роняет телефон с тумбочки



Мощность = -4

Монетка падает со спины собаки



Мощность = -5

Нажатие клавиши на клавиатуре IBM Model M.



Мощность = -6

Нажатие клавиши на современной клавиатуре.



Мощность = -7

Перышко опускается на землю.



Мощность = -8

Песчинка падает в кучку песка в нижней части маленьких песочных часов.



…и давайте сразу перескочим на


Мощность = -15

Пылинка приземляется на стол.



Приятно иногда для разнообразия не уничтожить мир.


Слова благодарности

Книгу, которую вы держите в руках, мне помогали писать многие. Спасибо моему редактору Кортни Янг за то, что она с самого начала читала xkcd и была с этой книгой до конца. Спасибо разным потрясающим сотрудникам издательского дома Houghton Mifflin Harcourt, благодаря которым книга увидела свет. Спасибо Сету Фишмэну и Гернертам за терпение и неутомимость.

Спасибо Кристине Глисон за то, что она сделала из этой книги именно книгу, – хотя для этого ей приходилось в три часа утра расшифровывать мои каракули про астероиды. Спасибо специалистам, которые помогали мне отвечать на вопросы, в частности Ройвену Лазарусу и Эллен Макманис (радиация), Элис Каанта (генетика), Дереку Лоу (химические вещества), Николь Гульюччи (телескопы), Иэну Маккею (вирусы) и Саре Гиллеспи (пули). Спасибо davean, благодаря которому все это увидело свет, но который ненавидит внимание и, возможно, будет недоволен даже тем, что я его упомянул.

Спасибо ребятам из IRC за комментарии и правку, спасибо Финну, Эллен, Аде и Рики за то, что они прочитали все присланные вопросы и отбраковали те, которые были про Гоку. Спасибо и самому Гоку – персонажу аниме, обладающему, похоже, безграничной силой: он спровоцировал сотни вопросов, пришедших на сайт «А что, если?» – хотя я все равно отказался смотреть аниме-сериал Dragon Ball Z, чтобы ответить на них.

Спасибо моей семье за то, что она научила меня отвечать на абсурдные вопросы, многие годы терпеливо отвечая на мои. Спасибо отцу за то, что он научил меня измерениям, и матери за то, что научила меня понимать модели. И спасибо моей жене, которая научила меня быть сильным, смелым и разбираться в птицах.

Источники и ссылки

Всемирный ураган

Merlis, Timothy M., and Tapio Schneider. Atmospheric dynamics of Earth-like tidally locked aquaplanets // Journal of Advances in Modeling Earth Systems 2 (December 2010). DOI:10.3894/ JAMES.2010.2.13.

What Happens Underwater During a Hurricane?

http://www.rsmas.miami.edu/blog/2012/10/22/what-happens-under-water-during-a-hurricane

Купание в ядерном бассейне

Behavior of spent nuclear fuel in water pool storage.

http://www.osti.gov/energycitations/servlets/purl/7284014-xaMii9/7284014.pdf

Unp lanned Exposure During Diving in the Spent Fuel Pool // SciTech Connect

http://www.isoe-network.net/index.php/publications-mainmenu-88/isoe-news/doc_download/1756-ritter2011ppt.html

Лазерна я указка

Mapping the World’s Population by Latitude, Longitude // GOOD

http://www.good.is/posts/mapping-the-world-s-population-by-lati-tude-longitude

http://www.wickedlasers.com/arctic

Периодическая стена элементов

Таблица на с. 9 (страница в печатном варианте 15, pdf-файле 15)

http://www.epa.gov/opptintr/aegl/pubs/arsenictrioxide_p01_tsddelete.pdf

Прыг-скок!

What if everyone jumped? // Dot Physics

http://scienceblogs.com/dotphysics/2010/08/26/what-if-everyone-jumped/

If everyone in China jumped off chairs at once, would the earth be thrown out of its orbit? // Straight Dope

http://www.straightdope.com/columns/read/142/if-all-chinese-jumped-at-once-would-cataclysm-result

Моль кротов

How many habitable planets are there in the galaxy? // Disсover http://blogs.discovermagazine.com/badastronomy/2010/10/29/how-many-habitable-planets-are-there-in-the-galaxy

Фен в ящике

Determination of Skin Burn Temperature Limits for Insulative Coatings Used for Personnel Protection

http://www.mascoat.com/assets/files/Insulative_Coating_Evaluation_NACE.pdf

The Nuclear Potato Cannon Part 2

http://nfttu.blogspot.com/2006/01/nuclear-potato-cannon-part-2.html

Последний свет человечества

Wind Turbine Lubrication and Maintenance: Protecting Investments in Renewable Energy

http://www.renewableenergyworld.com/rea/news/article/2013/05/wind-turbine-lubrication-and-maintenance-protecting-investments-in-renewable-energy

McComas, D. J., J. P. Carrico, B. Hautamaki, M. Intelisano, R. Lebois, M. Loucks, L. Policastri, M. Reno, J. Scherrer, N. A. Schwadron, M. Tapley, and R. Tyler. A new class of long – term stable lunar resonance orbits: Space weather applications and the Interstellar Boundary Explorer// Space Weather, 9, S11002, doi: 10.1029/2011SW000704, 2011.

Swift, G. M., et al. In-flight annealing of displacement damage in GaAs LEDs: A Galileo story // IEEE Transactions on Nuclear Science, Vol. 50, Issue 6 (2003).

Geothermal Binary Plant Operation and Maintenance Systems with Svartsengi Power Plant as a Case Study

http://www.os.is/gogn/unu-gtp-report/UNU-GTP-2002-15.pdf

Пулеметный ракетный ранец

Lecture L14-Variable Mass Systems: The Rocket Equation

http://ocw.mit.edu/courses/aeronautics-and-astronautics/16-07-dy-namics-fall-2009/lecture-notes/MIT16_07F09_Lec14.pdf

[2.4] Attack Flogger in Service

http://www.airvectors.net/avmig23_2.html#m4

Равномерно вверх

Otis: About Elevators

http://www.otisworldwide.com/pdf/AboutElevators.pdf

National Weather Service: Wind Chill Temperature Index

http://www.nws.noaa.gov/om/windchill/images/wind-chill-brochure.pdf

Prediction of Survival Time in Cold Air: см. таблицы на с. 24

http://cradpdf.drdc-rddc.gc.ca/PDFS/zba6/p144967.pdf

Linda D. Pendleton. When Humans Fly High: What Pilots Should Know About High-Altitude Physiology, Hypoxia, and Rapid Decompression.

Раздел коротких ответов

Currency in Circulation: Volume

http://www.federalreserve.gov/paymentsystems/coin_currcircvolume.htm

Subject: C5c, Why don’t we try to destroy tropical cyclones by nuking them? // NOAA

http://www.aoml.noaa.gov/hrd/tcfaq/C5c.html

Stagnation Temperature // NASA

http://www.grc.nasa.gov/WWW/BGH/stagtmp.html

Молния

Lightning Captured @ 7,207 Fps

http://www.youtube.com/watch?v=BxQt8ivUGWQ

Lightning: Expert Q&A // NOVA

http://www.pbs.org/wgbh/nova/earth/dwyer-lightning.html

Computation of the diameter of a lightning return stroke // JGR

http://onlinelibrary.wiley.com/doi/10.1029/JB073i006p01889/abstract

Мозг и компьютер

Moore’s Law at 40

http://www.ece.ucsb.edu/~strukov/ece15bSpring2011/others/Moore-sLawat40.pdf

Маленькая планета

Чтобы освежить в своей памяти «Маленького принца», прокрутите к последнему разделу чудесной страницы Мэллори Ортберга

http://the-toast.net/2013/08/02/texts-from-peter-pan-et-al/

Rugescu, Radu D., and Daniele Mortari. Ultra Long Orbital Tethers Behave Highly Non-Keplerian and Unstable // WSEAS Transactions on Mathematics, Vol. 7, No. 3, March 2008. P. 87–94.

http://www.academia.edu/3453325/Ultra_Long_Orbital_Tethers_Be-have_Highly_Non-Keplerian_and_Unstable

Падающий стейк

Falling Faster than the Speed of Sound

http://blog.wolfram.com/2012/10/24/falling-faster-than-the-speed-of-sound

Stagnation Temperature: Real Gas Effects

http://www.grc.nasa.gov/WWW/BGH/stagtmp.html

Calculation of Reentry-Vehicle Temperature History

http://www.dtic.mil/dtic/tr/fulltext/u2/a231552.pdf

Back in the Saddle

http://www.ejectionsite.com/insaddle/insaddle.htm

How to Cook Pittsburgh-Style Steaks

http://www.livestrong.com/article/436635-how-to-cook-pittsburgh-style-steaks

Хоккейная шайба

KHL’s Alexander Ryazantsev sets new ‘world record’ for hardest shot at 114 mph

http://sports.yahoo.com/blogs/nhl-puck-daddy/khl-alexander-ryazant-sev-sets-world-record-hardest-shot-174131642.html

Superconduc ting Magnets for Maglifter Launch Assist Sleds

http://www.psfc.mit.edu/~radovinsky/papers/32.pdf

Two-Stage Light Gas Guns

http://www.nasa.gov/centers/wstf/laboratories/hypervelocity/gasguns.html

Hockey Vide o: Goalies, Hits, Goals, and Fights

http://www.youtube.com/watch?v=fWj6-Cf9QA

Простуда

P. Stride. The St. Kilda boat cough under the microscope // The Journal of the Royal College of Physicians of Edinburgh, 2008; 38. P. 272–279.

L. Kaiser, J. D. Aubert, et al. Chronic Rhinoviral Infection in Lung Transplant Recipients // American Journal of Respiratory and Critical Care Medicine, Vol. 174. 2006. P. 1392–1399. 10.1164/ rccm.200604-489OC

Oliver, B. G. G., S. Lim, P. Wark, V. Laza-Stanca, N. King, J. L. Black, J. K. Burgess, M. Roth, and S. L. Johnston. Rhinovirus Exposure Impairs Immune Responses To Bacterial Products In Human Alveolar Macrophages // Thorax 63, no. 6 (2008). P. 519–525.

Наполовину пустой стакан

Shatter beer bottles: Bare-handed bottle smash

http://www.youtube.com/watch?v=77gWkl0ZUC8

Инопланетные астрономы

Douglas Adams. The Hitchhiker’s Guide to the Galaxy. (Книга неоднократно издавалась на русском языке: Адамс Д. Автостопом по Галактике. – Прим. ред.)

A Failure of Serendipity: The Square Kilometre Array will struggle to eavesdrop on Human-like ETI

http://arxiv.org/PS_cache/arxiv/pdf/1007/1007.0850v1.pdf

Eavesdropping on Radio Broadcasts from Galactic Civilizations with Upcoming Observatories for Redshifted 21cm Radiation

http://arxiv.org/pdf/astro-ph/0610377v2.pdf

The Earth as a Distant Planet a Rosetta Stone for the Search of Earth-Like Worlds

http://www.worldcat.org/title/earth-as-a-distant-planet-a-rosetta-stone-for-the-search-of-earth-like-worlds/oclc/643269627

SETI on the SKA

http://www.astrobio.net/exclusive/4847/seti-on-the-ska

Gemini Planet Imager

http://planetimager.org/

Больше никакой ДНК

Enjalbert, Fran?oise, Sylvie Rapior, Janine Nouguier-Soul?, Sophie Guillon, No?l Amouroux, and Claudine Cabot. Treatmen t of Amatoxin Poisoning: 20-Year Retrospective Analysis // Clinical Toxicology 40, no. 6 (2002). P. 715–757.

Richard Eshelman. I nearly died after eating wild mushrooms // The Guardian (2010).

http://www.theguardian.com/lifeandstyle/2010/nov/13/nearly-died-eating-wild-mushrooms

Amatoxin: A review

http://www.omicsgroup.org/journals/2165-7548/2165-7548-2-110.php?aid=5258

Всепланетный самолет

The Martian Chronicles

http://www.x-plane.com/adventures/mars.html

Aerial Regional-Scale Environmental Survey of Mars

http://marsairplane.larc.nasa.gov/

Panoramic Views and Landscape Mosaics of Titan Stitched from Huygens Raw Images

http://www.beugungsbild.de/huygens/huygens.html

New images from Titan

http://www.esa.int/Our_Activities/Space_Science/Cassini-Huygens/New_images_from_Titan

Да пребудет с вами Сила!

Saturday Morning Breakfast Cereal

http://www.smbc-comics.com/index.php?db=comics&id=2305#comic ‘Beethoven Virus’–Musical Tesla Coils // Youtube

http://www.youtube.com/watch?v=uNJjnz-GdlE

Beast. The 15Kw 7’ tall DR (DRSSTC 5)

http://www.goodchildengineering.com/tesla-coils/drsstc-5-10kw-mon-ster

Падение с помощью гелия

De Haven, H. Mechanical analysis of survival in falls from heights of fifty to one hundred and fifty feet // Injury Prevention, 6(1). P. 62–68.

http://injuryprevention.bmj.com/content/6/1/62.3.long

Armchair Airman Says Flight Fulfilled His Lifelong Dream // New York Times, July 4, 1982.

http://www.nytimes.com/1982/07/04/us/armchair-airman-says-flight-fulfilled-his-lifelong-dream.html?pagewanted=all

Jason Martinez. Falling Faster than the Speed of Sound // Wolfram Blog, October 24, 2012.

http://blog.wolfram.com/2012/10/24/falling-faster-than-the-speed-of-sound

Все на выход!

Project Orion: The Tr ue Story of the Atomic Spaceship

http://www.amazon.com/Project-Orion-Story-Atomic-Spaceship/dp/0805059857

Самооплодотворение

Sperm Cells Created From Human Bone Marrow

http://www.sciencedaily.com/releases/2007/04/070412211409.htm

Nayernia, Karim, Tom Strachan, Majlinda Lako, Jae Ho Lee, Xin Zhang, Alison Murdoch, John Parrington, Miodrag Stojkovic, David Elliott, Wolfgang Engel, Manyu Li, Mary Herbert, and Lyle Armstrong. RETRACTION-In Vitro Derivation Of Human Sperm From Embryonic Stem Cells // Stem Cells and Development (2009): 0908w75909069.

Подробнее об этом см.: F. M. Lancaster. Genetic and Quantitative Aspects of Genealogy

http://www.genetic-genealogy.co.uk/Toc115570144.html

Бросок вверх

A Prehis tory of Throwing Things

http://ecodevoevo.blogspot.com/2009/10/prehistory-of-throwing-things.html

Chapter 9. Stone tool s and the evolution of hominin and human cognition

http://www.academia.edu/235788/Chapter_9._Stone_tools_and_the_evolution_of_hominin_and_human_cognition

The unitary hypothesi s: A common neural circuitry for novel manipulations, language, plan-ahead, and throwing?

http://www.williamcalvin.com/1990s/1993Unitary.htm

Evolution of the human hand: The role of throwing and clubbing

http://www.ncbi.nlm.nih.gov/pmc/articles/PMC1571064

Errors in the control of joint rotations associated with inaccuracies in overarm throws

http://jn.physiology.org/content/75/3/1013.abstract

Speed of Nerve Impulses

http://hypertextbook.com/facts/2002/DavidParizh.shtml

Farthest Distance to Throw a Golf Ball

http://recordsetter.com/world-record/world-record-for-throwing-golf-ball/7349#contentsection

Смертоносные нейтрино

Karam, P. Andrew. Gamma and Neutrino Radiation Dose from Gamma Ray Bursts and Nearby Supernovae // Health Physics 82, no. 4 (2002). P. 491–99.

Лежачий полицейский

Speed bump-induced spinal column injury

http://akademikpersonel.duzce.edu.tr/hayatikandis/sci/hayatikandis12.01.2012_08.54.59sci.pdf

Speed hump spine fractures: Injury mechanism and case series

http://www.ncbi.nlm.nih.gov/pubmed/21150664

The 2nd American Conference on Human Vibration

http://www.cdc.gov/niosh/mining/UserFiles/works/pdfs/2009-145.pdf

Speed bump in Dubai + flying Gallardo

http://www.youtube.com/watch?v=Vg79_mM2CNY

Parker, Barry R. Aerodynamic Design // The Isaac Newton School of Driving: Physics and your car. Baltimore, MD: Johns Hopkins University Press, 2003. P. 155.

The Myth of the 200-mp h “Lift-Off Speed”

http://www.buildingspeed.org/blog/2012/06/the-myth-of-the-200-mph-lift-off-speed/

Mercedes CLR-GTR Le Mans Flip

http://www.youtube.com/watch?v=rQbgSe9S54I

National Highway Transportation NHTSA. Summary of State Speed Laws. 2007.

Пропускная способность FedEx

FedEx still faster than the Internet

http://royal.pingdom.com/2007/04/11/fedex-still-faster-than-the-internet

Cisco Visual Networking Index: Forecast and Methodology, 2012– 2017

http://www.cisco.com/en/US/solutions/collateral/ns341/ns525/ns537/ns705/ns827/white_paper_c11-481360_ns827_Networking_Solutions_White_Paper.html

Intel® Solid-State Driv e 520 Series

http://download.intel.com/newsroom/kits/ssd/pdfs/intel_ssd_520_product_spec_325968.pdf

Trinity test press releases (May 1945)

http://blog.nuclearsecrecy.com/2011/11/10/weekly-document-01

NEC and Corning achieve petabit optical transmission

http://optics.org/news/4/1/29

Свободное падение

Super Mario Bros. – Speedrun level 1–1 [370]

http://www.youtube.com/watch?v=DGQGvAwqpbE

Sprint ring cycle

http://www1.sprintpcs.com/support/HelpCenter.jsp?FOLDER%3C%3Efolder_id=1531979#4

Glide data

http://www.dropzone.com/cgi-bin/forum/gforum.cgi?post=577711#577711

Jump. Fly. Land // Air & Space

http://www.airspacemag.com/flight-today/Jump-Fly-Land.html

Prof. Dr. Herrligkoffer. The East Pillar of Nanga Parbat // The Alpine Journal (1984).

The Guestroom – Dr. Glenn Singleman and Heather Swan

http://www.abc.net.au/local/audio/2010/08/24/2991588.htm

Highest BASE jump: Valery Rozov breaks Guinness world record

http://www.worldrecordacademy.com/sports/highest_BASE_jump_Valery_Rozov_breaks_Guinness_world_record_213415.html

Dean Potter. Above It All

http://www.tonywingsuits.com/deanpotter.html

Спарта

Если верить парню из Интернета, Энди Любенски: Andy Lubien-ski. The Longbow

http://www.pomian.demon.co.uk/longbow.htm

Осушить океаны

Если экстраполировать максимальное давление, которое выдерживает обшивка корпуса ледокола

http://www.iacs.org.uk/document/public/Publications/Unified_requirements/PDF/UR_I_pdf410.pdf

An experimental study of critical submergence to avoid free-surface vortices at vertical intakes

http://www.leg.state.mn.us/docs/pre2003/other/840235.pdf

Осушить океаны, часть II

D. L. Santiago et al. Mar s climate and outflow events

http://spacescience.arc.nasa.gov

D. L. Santiago et al. Cloud formation and water transport on Mars after major outflow events // 43rd Planetary Science Conference (2012).

Maggie Fox. Mars May Not Have Been Warm o r Wet

http://rense.com/general32/marsmaynothave.htm

Уникальные твиты

Hendrik Willem Van Loon. The Story of Mankind

http://books.google.com/books?id=RskHAAAAIAAJ&pg=PA1#v=onepage&q&f=false Counting Characters

https://dev.twitter.com/docs/counting-characters

A Mathematical Theory of Communication

http://cm.bell-labs.com/cm/ms/what/shannonday/shannon1948.pdf

LEGO-мост

How tall can a Lego tower get?

http://www.bbc.co.uk/news/magazine-20578627

Investigation Into the Strength of Lego Technic Beams and Pin Connections

http://eprints.usq.edu.au/20528/1/Lostroh_LegoTesting_2012.pdf

Total value of property in London soars to ?1.35 trn

http://www.standard.co.uk/business/business-news/total-value-of-property-in-london-soars-to-135trn-8779991.html

Случайный чих

Cari Nieren berg. The Perils of Sneezing, ABC News. Dec. 22, 2008

http://abcnews.go.com/Health/ColdandFluNews/story?id=6479792&page=1

Bischoff Werner E., Miche lle L. Wallis, Brian K. Tucker, Beth A. Re-boussin, Michael A. Pfaller, Frederick G. Hayden, and Robert J. Sherertz. ‘Gesundheit!’ Sneezing, Common Colds, Allergies, and Staphylococcus aureus Dispersion // J Infect Dis. (2006), 194 (8). P. 1 119–1126. doi:10.1086/507908.

Annual Rates of Lightning Fatalities by Country

http://www.vaisala.com/Vaisala%20Documents/Scientific%20papers/Annual_rates_of_lightning_fatalities_by_country.pdf

Расширяющаяся Земля

Wu, X., X. Collilieux, Z. Altamimi, B. L. A. Vermeersen, R. S. Gross, and I. Fukumori. Accuracy of the International Terrestrial Reference Frame origin and Earth expansion, Geophys. Res. Lett., 38,

L13304. 2011. doi:10.1029/2011GL047450: In conclusion, no statistically significant present expansion rate is detected by our study within the current measurement uncertainty of 0.2 mm yr-1.

http://repository.tudelft.nl/view/ir/uuid%3A72ed93c0-d13e-427c8c5f-f013b737750e/

Lawrence Grybosky. Thermal Expansion and Contraction

http://www.engr.psu.edu/ce/courses/ce584/concrete/library/cracking/thermalexpansioncontraction/thermalexpcontr.htm


Sasselov, Dimitar D. The life of super-Earths: How the hunt for alien worlds and artificial cells will revolutionize life on our planet. New York: Basic Books, 2012.

Franz, R. M. and P. C. Schutte. Barometric hazards within the context of deep-level mining // The Journal of The South African Institute of Mining and Metallurgy.

Plummer, H. C. Note on the motion about an attracting centre of slowly increasing mass // Monthly Notices of the Royal Astronomical Society, Vol. 66. P. 83

http://adsabs.harvard.edu/full/1906MNRAS..66…83P

Невесомая стрела

Hunting Arrow Selection Guide: Chapter 5

http://www.huntersfriend.com/carbon_arrows/hunting_arrows_selection_guide_chapter_5.htm

USA Archery Records, 2009

http://www.usaarcheryrecords.org/FlightPages/2009/2009%20World%20Regular%20Flight%20Records.pdf

Air flow around the point of an arrow

http://pip.sagepub.com/content/227/1/64.full.pdf

STS-124: KIBO, NASA

http://www.nasa.gov/pdf/228145main_sts124_presskit2.pdf

Земля без Солнца

The extreme magnetic storm of 1–2 September 1859

http://trs-new.jpl.nasa.gov/dspace/bitstream/2014/8787/1/02-1310.pdf

Tsurutani T. B. et al. Geomagnetic Storms // Journal of Geophysical Research, vol. 108, № A7, 1268, 2003. doi:10.1029/2002JA009504

http://www.oecd.org/governance/risk/46891645.pdf

Roger A. Pielke Jr. et al. Normalized Hurricane Damage in the United States: 1900–2005

http://sciencepolicy.colorado.edu/admin/publication_files/resource-2476-2008.02.pdf

Impacts of Federal-Aid Highway Investments Modeled by NBIAS

http://www.fhwa.dot.gov/policy/2010cpr/chap7.htm#9

Time zones matter: The impact of distance and time zones on services trade

http://eeecon.uibk.ac.at/wopec2/repec/inn/wpaper/2012-14.pdf

Baby Fact Sheet

http://www.ndhealth.gov/familyhealth/mch/babyfacts/Sunburn.pdf

The photic sneeze reflex as a risk factor to combat pilots

http://www.ncbi.nlm.nih.gov/pubmed/8108024

Burned by wild parsnip

http://dnr.wi.gov/wnrmag/html/stories/1999/jun99/parsnip.htm

Правки в распечатанной «Википедии»

Wikipedia as a Printed Book // BrandNew

http://www.brandnew.uk.com/wikipedia-as-a-printed-book/

ToolServer: Edit rate

http://toolserver.org/~emijrp/wmcharts/wmchart0001.php

Закат над Британской империей

Eddie Izzard. Do you have a flag?

http://www.youtube.com/watch?v=uEx5G-GOS1k

This Sceptred Isle: Empire. A 90 part history of the British Empire

http://www.bbc.co.uk/radio4/history/empire/map

A Guide to the British Overseas Territories

http://www.telegraph.co.uk/news/wikileaks-files/london-wikileaks/8305236/A-GUIDE-TO-THE-BRITISH-OVER-SEAS-TERRITORIES.html

William Prochanu and Laura Parker. Trouble in Paradise // Vanity Fair. January 2008.

http://www.vanityfair.com/culture/features/2008/01/pitcairn200801

Long History of Child Abuse Haunts Island ‘Paradise’

http://www.npr.org/templates/story/story.php?storyId=103569364

JavaScript Solar Eclipse Explorer

http://eclipse.gsfc.nasa.gov/JSEX/JSEX-index.html

Помешивая чай

Brawn Mixer, Inc., Principles of Fluid Mixing. 2003

http://www.craneengineering.net/products/mixers/documents/craneEngineeringPrinciplesOfFluidMixing.pdf

Cooling a cup of coffee with help of a spoon

http://physics.stackexchange.com/questions/5265/cooling-a-cup-of-coffee-with-help-of-a-spoon/5510#5510

Все молнии мира

Introduction to Lightning Safety // National Weather Service, Wilmington, Ohio

http://www.erh.noaa.gov/iln/lightning/2012/lightningsafetyweek.php

B?rgesser Rodrigo E., Maria G. Nicora, and Eldo E. ?vila. Characterization of the lightning activity of Rel?mpago del Catatumbo // Journal of Atmospheric and Solar-Terrestrial Physics. 2011

http://wwlln.net/publications/avila.Catatumbo2012.pdf

Самый одинокий человек

BBC Future взяло интервью у Эла Уолдена (April 2, 2013)

http://www.bbc.com/future/story/20130401-the-loneliest-human-being/1

Капля дождя

SSMI/SSMIS/TMI-derive d Total Precipitable Water-North Atlantic

http://tropic.ssec.wisc.edu/real-time/mimic-tpw/natl/main.html

Structure of Florida Thunderstorm s Using High-Altitude Aircraft Radiometer and Radar

Экзамен наугад

Cooper, Mary Ann, MD. Disability, Not Death Is the Main Problem with Lightning Injury

http://www.uic.edu/labs/lightninginjury/Disability.pdf

2008 Lightning Fatalities // National Oceanic and Atmospheric Administration (NOAA)

http://www.nws.noaa.gov/om/hazstats/light08.pdf

Нейтронная пуля

Influence of Small Arms Bullet Construction on Terminal Ballistics

http://hsrlab.gatech.edu/AUTODYN/papers/paper162.pdf

McCall, Benjamin. Q & A: Neutron Star Densities // University of Illinois

http://van.physics.illinois.edu/qa/listing.php?id=16748


Сноски

1

В смысле, не погибнет сразу (здесь и далее примечания автора, если не оговорено иное. – Прим. ред.).

(обратно)

2

См. мой комикс http://what-if.xkcd.com/26, в котором объясняется, почему это происходит.

(обратно)

3

После того, как я впервые опубликовал все эти расчеты, физик из Массачусетского технологического института по имени Ханс Риндеркнехт рассказал мне, что он смоделировал этот сценарий на компьютерах своей лаборатории. Выяснилось, что в начале полета мяча большая часть молекул воздуха двигалась слишком быстро, чтобы это привело к синтезу: они просто пролетали бы сквозь мяч, нагревая его более медленно и равномерно, чем описано у меня.

(обратно)

4

А жаль, получился бы отличный энергетический напиток.

(обратно)

5

Также известные как делавары или ленни-ленапе.

(обратно)

6

Также известных как горные львы, они же кугуары, они же горные кошки.

(обратно)

7

Хотя вы вряд ли увидите зрелище, которое представилось европейским поселенцам: миллиарды птиц, закрывающих небо. В своей книге «1491» историк Чарльз Манн предполагает, что эти гигантские стаи могли возникнуть именно в результате колонизации и связанного с ней хаоса в экосистеме, потревоженной появлением оспы, мятлика и медоносных пчел.

(обратно)

8

Остров в устье р. Гудзон, самый южный округ города Нью-Йорк. – Прим. ред.

(обратно)

9

Город на Гудзоне к северу от Нью-Йорка. – Прим. ред.

(обратно)

10

Разве что рекламных щитов было поменьше.

(обратно)

11

Был. Но уже скоро мы с этим покончим.

(обратно)

12

Если что – это случайное совпадение.

(обратно)

13

Если вы знаете, напишите мне, пожалуйста.

(обратно)

14

См. http://xkcd.com/314.

(обратно)

15

К тому моменту, как вы это читаете, могли добавить и восьмой ряд. А если вы читаете это в 2038 году, то в таблице уже наверняка 10 рядов, но любое ее обсуждение запрещено роботами, захватившими нашу планету!

(обратно)

16

При условии, что все они находятся в двухатомной форме, т. е. О2, N2. Если кубик состоит из отдельных атомов, они немедленно объединятся, разогревшись при этом до нескольких тысяч градусов.

(обратно)

17

Дерек ведет прекрасный химический блог In the Pipeline.

(обратно)

18

Именно из-за этих его свойств белый фосфор используют в печально известных зажигательных бомбах и снарядах.

(обратно)

19

Поищите на Youtube по словам gallium infiltration, чтобы увидеть, насколько это странное зрелище.

(обратно)

20

В этом случае мы еще могли бы пожать плечами и выкинуть его из головы.

(обратно)

21

В оригинале игра слов, mole of moles. – Прим. пер.

(обратно)

22

Один моль – это приблизительное число атомов в грамме водорода. Случайным образом это также вполне достойная оценка количества песчинок на Земле.

(обратно)

23

См., например: ru.wikipedia.org/wiki/Звездонос.

(обратно)

24

Совпадение с названием вымышленного вещества из романа Курта Воннегута «Колыбель для кошки» – случайность.

(обратно)

25

Но совсем не обязательно верно для зарядных устройств, подключенных к другому прибору. Если зарядка соединена со смартфоном или ноутбуком, то электроэнергия может утекать из розетки через зарядку в устройство.

(обратно)

26

Если мы с вами когда-нибудь окажемся запертыми в горящем здании, пожалуйста, игнорируйте мои идеи о том, как нам оттуда выбраться.

(обратно)

27

Первый в мире взрыв атомной бомбы, 16 июля 1945 года, штат Нью-Мексико. – Прим. ред.

(обратно)

28

Падение на Юпитер было необходимо, чтобы безопасно уничтожить зонд и не допустить заражения близлежащих небесных тел – например, спутника Юпитера Европы, на которой есть вода, земными бактериями.

(обратно)

29

В СССР строили маяки, работавшие на радиоактивном распаде, но они уже не функционируют.

(обратно)

30

Судя по количеству боеприпасов, которые разбросаны у них по всему дому, этот штат уже превратился в постапокалиптическую зону боевых действий в духе фильма «Безумный Макс».

(обратно)

31

В идеале кого-то с меньшим запасом боеприпасов.

(обратно)

32

Это не поможет вам выжить, но все же…

(обратно)

33

Для этих расчетов я использую стандартный температурный профиль атмосферы, который, конечно же, может варьироваться в зависимости от места.

(обратно)

34

Давление в кабине обычно находится на уровне 70–80 % давления на уровне моря, если верить барометру в моем телефоне.

(обратно)

35

Кроме Кельвина.

(обратно)

36

И откровенно говоря, этот сценарий создает больше вопросов, чем ответов.

(обратно)

37

Вот почему крутые парни в кино гасят спички пальцами, но никогда не делают этого с факелами – хотя спички и факелы имеют приблизительно одинаковую температуру.

(обратно)

38

Когда лодка окажется на солнечном свету, поверхность ее корпуса нагреется, но все равно терять тепло будет быстрее, чем получать.

(обратно)

39

Why don’t we try to destroy tropical cyclones by nuking them?’ by Chris Landsea (Крис Лэндси: «Почему мы не пытаемся уничтожать тропические циклоны ядерными бомбами»).

(обратно)

40

3 x 1024 – это верхняя граница оценки, есть и более умеренные оценки, в которых количество нулей на два меньше. – Прим. науч. ред.

(обратно)

41

Или настоящий, раз уж на то пошло.

(обратно)

42

Несмотря на название («обратный разряд молнии»), электричество все еще идет сверху вниз. Однако разряд выглядит так, словно направлен вверх. Похожий эффект можно наблюдать, когда светофор переключается на зеленый: машины в первых рядах трогаться с места, затем машины, стоящие за ними, так что кажется, что движение распространяется от первых рядов назад.

(обратно)

43

Я бы все же не стал использовать настоящий шар – это может быть небезопасно.

(обратно)

44

Кроме сорта «ред делишес», название которого («красный вкусный») возмутительно обманчиво.

(обратно)

45

Когда я был маленьким, у нас дома было довольно много ваз.

(обратно)

46

По крайней мере, пока что.

(обратно)

47

Moravec Hans. Robot: Mere Machine to Transcendent Mind. Oxford University Press, 2000.

(обратно)

48

Хотя даже этого может оказаться недостаточно, чтобы полностью охватить все, что происходит в мозгу. Биология – хитрая штука.

(обратно)

49

Используя 82 944 процессора, каждый с 750 млн транзисторов, Kcomputer понадобилось 40 минут, чтобы симулировать одну секунду деятельности виртуального мозга, обладавшего всего 1 % связей от числа имеющихся в настоящем человеческом мозгу.

(обратно)

50

Если вы читаете это уже после 2036 года, привет из далекого прошлого! Надеюсь, в будущем дела обстоят лучше. P. S. Пожалуйста, придумайте, как забрать нас к себе.

(обратно)

51

Это касается тех случаев, когда размеры спутника сопоставимы с расстоянием до центра притягивающего тела. В случае с планетой Маленького принца такая орбита будет находиться в нескольких метрах над поверхности астероида. – Прим. науч. ред.

(обратно)

52

Hach? Alain. The Physics of Hockey. Johns Hopkins University Press, 2002. – Прим. ред.

(обратно)

53

В том смысле, что стейк не расколется и не разлетится на кусочки, но он не обязательно будет пригоден в пищу.

(обратно)

54

Я знаю, о чем вы сейчас хотите спросить. Ответ: нет, он не проведет достаточно времени в радиационном поясе Земли, чтобы радиация могла его полностью стерилизовать.

(обратно)

55

На самом деле, любая инфекция дыхательных путей может быть причиной так называемой простуды.

(обратно)

56

Вообще причиной симптомов является именно реакция иммунитета, а не деятельность вирусов.

(обратно)

57

Это должно быть верно по чисто математическим причинам. Если бы этого не происходило, вирусы бы вымерли, а если бы заражалось больше одного человека, то все всегда были бы простужены.

(обратно)

58

Около 450 млн чел.

(обратно)

59

Около 650 млн чел.

(обратно)

60

Жители Сент-Килда верно определили, что причиной эпидемий становились корабли. Однако медики того времени отвергали эти предположения, считая, что островитяне болеют из-за того, что долго стоят на холодном ветру, встречая корабль, и слишком много пьют, празднуя его прибытие.

(обратно)

61

Разве что у нас закончится еда во время карантина, и мы все умрем от голода. В этом случае наши риновирусы погибнут вместе с нами.

(обратно)

62

Карл Циммер. «Планета вирусов». М, «Феникс», 2012. – Прим. ред.

(обратно)

63

Даже вакуум не стопроцентно пуст, но это уже вопрос терминологии.

(обратно)

64

Ну, если очень хочется…

(обратно)

65

Пока на Земле существуют радиостанции и эфирные телепередачи, часть излучения будет уходить в космос. Другое дело, что для воспроизведения сигнала инопланетяне должны будут знать еще и систему его кодирования. – Прим. науч. ред.

(обратно)

66

Я родился лишь в конце этого периода, но, насколько я слышал, обстановка была напряженной.

(обратно)

67

Мы, земляне, столкнулись с этим в 1977 году. Природа так называемого «сигнала „Ого-го“» (его еще называют «сигнал Wow!») длиной в 72 секунды до сих пор неизвестна.

(обратно)

68

Не могу утверждать этого наверняка, но думаю, мы бы узнали, случись с кем-нибудь подобное.

(обратно)

69

Более известное название рода Amanita – мухоморы. В этом роду есть несколько видов, которые называют «ангелами-убийцами», и они наряду с бледной поганкой, которая тоже (сюрприз! сюрприз!) относится к роду Amanita, отвечают за большую часть смертей от отравления грибами.

(обратно)

70

Откуда я это знаю: мой приятель пробрался в вашу комнату с микроскопом и все проверил, пока вы спали.

(обратно)

71

Иммуностимуляторы наподобие пэгфилграстима (препарат неуластим) позволяют безопасно пережить частые сеансы химиотерапии. Они стимулируют выработку лейкоцитов, по сути убеждая тело, что оно сильно инфицировано кишечной палочкой и с ней нужно бороться.

(обратно)

72

Но есть некоторая разница: если капнуть суперклеем на хлопковую пряжу, она загорится.

(обратно)

73

Очень сильные дозы радиации убивают людей быстро, но не за счет повреждения ДНК. Вместо этого они физически растворяют гематоэнцефалический барьер, и наступает быстрая смерть от кровоизлияния в мозг.

(обратно)

74

Это «Сессна-172», вероятно, самый распространенный самолет в мире.

(обратно)

75

Не говоря уже о том, что топливо для таких двигателей сделано из древних растений.

(обратно)

76

Он даже пишет о самолетах ТОЛЬКО КАПСЛОКОМ.

(обратно)

77

Полет над Венерой… как-то мне не очень удается сделать эту идею привлекательной, не так ли?

(обратно)

78

Девиз экспедиции: «Туда, где еще голубее!»

(обратно)

79

Немаловажно, что из Канады, в отличие США, летает много коммерческих рейсов на Кубу.

(обратно)

80

Ситуация изменилась в 2013 году, когда компания Frontier Airlines запустила рейс Ньюкасл – Форт-Майерс (Флорида). Эти данные не были учтены в моих расчетах, и возможно, за счет этого рейса Делавэр опустится на более низкое место в рейтинге.

(обратно)

81

Помимо штрафа в 1500 долларов, полет Уолтерса был отмечен свидетельством о рекорде высоты для кластерных воздушных шаров, а также Премией Дарвина. – Прим. науч. ред.

(обратно)

82

Вероятно, в одних только Соединенных Штатах живет около миллиона тонн домашних собак.

(обратно)

83

В 2011 году ученые впервые создали полноценные сперматозоиды из стволовых клеток. Правда, не человеческие, а мышиные, но тем не менее при помощи этих сперматозоидов родились здоровые мышата. – Прим. науч. ред.

(обратно)

84

Потому что единица – это мультипликативное тождество.

(обратно)

85

Некоторые формы спинной мышечной атрофии связаны с дефектом в двух генах, так что на практике статистика несколько сложнее.

(обратно)

86

Игра вроде «Змеи и лестницы». – Прим. пер.

(обратно)

87

Ну, это одна из причин.

(обратно)

88

Амбистома Трембле – гибридный вид саламандр, который размножается самооплодотворением. Этот вид состоит исключительно из самок, и у него, как ни странно, три набора хромосом вместо двух. Чтобы размножиться, они выполняют ритуал ухаживания с самцом родственного вида, а затем откладывают яйца. Самцу это ничего не дает – его просто используют для того, чтобы стимулировать откладывание яиц.

(обратно)

89

На самом деле они так защищаются от хищников.

(обратно)

90

Источник информации: я когда-то играл в юношеской бейсбольной лиге.

(обратно)

91

Тот же источник.

(обратно)

92

И однажды оставшийся там навсегда.

(обратно)

93

И немало другой полезной информации.

(обратно)

94

Если вы ребенок, это случается еще реже, потому что в вас меньше атомов. Статистически первое столкновение с нейтрино происходит в возрасте около десяти лет.

(обратно)

95

Примерно тот же смысл, как в русском выражении «стать последней каплей». – Прим. ред.

(обратно)

96

Что все равно составляет меньше одного процента всех муравьев в мире.

(обратно)

97

Если хотите помучить учеников, начавших изучать алгебру, предложите им взять производную от ln([(x))]e по x». Кажется, что в ответе должно быть что-то вроде «1», но это не так.

(обратно)

98

3,262 световых года, или чуть меньше, чем расстояние от нас до альфы Центавра.

(обратно)

99

Таблица доз радиации – http://xkcd.com/radiation.

(обратно)

100

Можете выйти на улицу с линейкой и проверить – машин везде полно.

(обратно)

101

На высоких скоростях можно потерять управление и без искусственных неровностей. После аварии Джоуи Хоникатта на скорости 350 км/ч от его Camaro остался только выжженный остов.

(обратно)

102

Например: а что случилось с остальными людьми? Они в порядке?

(обратно)

103

Может, изобрести калькулятор видимости – было бы забавно! Теперь я знаю, чем мне заняться вечером в субботу.

(обратно)

104

Поэтому обычно мы стараемся не принимать в расчет такие вещи.

(обратно)

105

Конечно, возникнут кое-какие сложности. В болотах Луизианы, в карибских мангровых зарослях и в норвежских фьордах вы разовьете несколько меньшую скорость, чем на ровном песчаном пляже.

(обратно)

106

Особенно если речь об околоземной орбите, на которой находится МКС и куда могут летать космические корабли.

(обратно)

107

X-15 дважды достиг высоты в 100 км, и оба раза им управлял Джо Уокер.

(обратно)

108

Главное, помните, что лететь нужно вверх, а не вниз, иначе получится нехорошо.

(обратно)

109

Если вы находитесь в верхней части околоземной орбиты – чуть поменьше.

(обратно)

110

Это возрастание по экспоненте – центральная проблема ракетостроения. Топливо, требующееся для того, чтобы ускорить вас на 1 км/ч, делает вас тяжелее примерно в 1,4 раза. Чтобы выбраться на орбиту, нужно разогнаться до 8 км/ч, значит, вам потребуется много топлива: 1,4 x 1,4 x 1,4 x 1,4 x 1,4 x 1,4 x 1,4 x 1,4 = 15, то есть приблизительно в 15 раз больше, чем изначальная масса вашего корабля.

При ипользовании ракетного двигателя для торможения снижение скорости на 1 км/с потребует увеличения вашей массы в те же 1,4 раза. Если же вы хотите замедлиться до нуля – и медленно и осторожно войти в атмосферу, – топливо опять же увеличит вашу массу в 15 раз.

(обратно)

111

Существуют различные приложения и сайты, позволяющие вычислить положение станции и других спутников.

(обратно)

112

Использование какой-либо ритмичной песни для измерения времени – это техника, которая применяется, например, при обучении сердечно-легочной реанимации. Там для этого используется песню Stayin’ Alive («Остаться в живых») группы Bee Gees».

(обратно)

113

Источник: AAAAAAAAAAAAAAAAAAAAAAAAAAA!!!

(обратно)

114

Персонаж культового научно-фантастического сериала «Светлячок», девочка со сверхспособностями, «машина для убийства». – Прим. пер.

(обратно)

115

Отсылка к выражению smartest thing since sliced bread («самая умная вещь после нарезанного хлеба»). – Прим. перев.

(обратно)

116

Примерная оценка такова: человек на квадратный метр – это не слишком плотная толпа, четыре человека на квадратный метр – это танцпол.

(обратно)

117

Challenger Deep – самая глубокая точка Мирового океана, ее глубина составляет 10994±40 м. – Прим. ред.

(обратно)

118

Только не забывайте раз в несколько дней очищать фильтры от китов.

(обратно)

119

Знатоки настаивают, что правильно писать прописными: LEGO.

(обратно)

120

Строго говоря, LEGO Group® требует, чтобы писали LEGO®.

(обратно)

121

С другой стороны, писатели не обязаны каждый раз ставить торговый знак. Руководство по стилистике английской «Википедии» считает, что нужно писать Lego.

(обратно)

122

С другой стороны, стиль самой Википедии тоже частенько критикуют. Обсуждения этой темы растянулись на много экранов жарких споров. Не менее горячие дискуссии возникают по поводу использования курсива.

(обратно)

123

Нет уж, вот так точно не пишет.

(обратно)

124

ОК, дальше пишем так.

(обратно)

125

Источник информации: однажды я построил лодочку из Lego, опустил ее в воду, и она утонула ?

(обратно)

126

Видимо, в этот день более важных новостей не было.

(обратно)

127

В некоторые наборы Lego входят маленькие нейлоновые тросики (на тот случай, если мы хотим ограничиться исключительно Lego).

(обратно)

128

Потом они сказали: «Минутку, что-что вы собираетесь построить?», а потом: «А как вы вообще сюда попали, сэр?»

(обратно)

129

Не забудьте сфотографироваться на фоне знака «Осторожно, белые медведи!»

(обратно)

130

Эта стратегия подходит для спасения от любого из терминаторов.

(обратно)

131

См. комикс «Угловой момент», http://xkcd.com/162/.

(обратно)

132

Средний показатель для США (и один из самых высоких для развитых стран) – четыре убийства на 100 000 жителей.

(обратно)

133

Источник информации: вы до сих пор живы.

(обратно)

134

Для наглядности: столько времени занимает песня Hey Jude, прослушанная 490 раз подряд.

(обратно)

135

За все 58 часов исследования максимальным результатом оказались четыре чиха. Я предпочел бы 490 раз послушать Hey Jude.

(обратно)

136

Последним аргументом, подтвердившим теорию континентального дрейфа, было открытие явления спрединга (растяжения) морского дна. Взаимное подтверждение теорий инверсии магнитного поля Земли и спрединга морского дна – один из любимых моих примеров того, как вообще работает научное открытие.

(обратно)

137

Очень быстро выяснилось, что она никуда не годится.

(обратно)

138

Как выясняется, океан расширяется, потому что становится теплее. Это (на данный момент) основной механизм повышения уровня воды при глобальном потеплении.

(обратно)

139

Масса пропорциональна радиусу в кубе, а гравитация пропорциональна массе, умноженной на обратный квадрат радиуса, так что радиус? / радиус? = радиус.

(обратно)

140

Именно это вам и хочется услышать о небоскребе, в котором вы находитесь.

(обратно)

141

Хотя я на всякий случай не стал бы пользоваться лифтами.

(обратно)

142

За десятилетия сила гравитации станет нарастать быстрее, чем можно было ожидать, поскольку вещество, из которого состоит Земля, будет уплотняться под собственным весом. Давление внутри планеты приблизительно пропорционально квадрату площади ее поверхности, так что ядро Земли сильно сожмется.

(обратно)

143

Хотя некоторые радиоактивные элементы, такие как уран, довольно тяжелые, они выталкиваются из нижних слоев Земли, поскольку их атомы не встраиваются в кристаллическую решетку горных пород на такой глубине.

(обратно)

144

Прости, пожалуйста, Луна!

(обратно)

145

Кроме того, стрельба в астронавтов из лука обычно не входит в программу по физике – ну, по крайней мере, в программу бакалавриата.

(обратно)

146

Конечно, кроме солнечных – им-то уж точно крышка!

(обратно)

147

Подходящую систему хранения вообразить непросто. Я изо всех сил сдерживаю себя, чтобы не погрузиться в ее разработку.

(обратно)

148

Во всяком случае, так было, когда я это писал, то есть до этой проклятой революции роботов.

(обратно)

149

Можно выяснить количество пользователей каждой возрастной группы, используя инструмент для создания рекламы в «Фейсбуке». Однако надо учитывать, что те или иные причины заставляют некоторых людей скрывать свой настоящий возраст.

(обратно)

150

В некоторых расчетах я использовал данные о возрасте пользователей из США и затем экстраполировал эти данные на всю пользовательскую базу «Фейсбука» (легче найти статистику по США, чем сразу по всем странам мира, где пользуются «Фейсбуком»). США – не усредненная модель мира, но динамика, при которой успех «Фейсбука» определяется количеством молодых пользователей, а рост числа пользователей продолжается некоторое время, после чего выходит на плато, скорее всего, будет верной везде. Если мы предположим, что развивающийся мир, где население моложе и растет быстрее, будет охвачен «Фейсбуком», это сдвинет даты на несколько лет, но общую картину изменит не так сильно, как можно было бы ожидать.

(обратно)

151

Во всех этих сценариях я предполагаю, что информация никогда не удаляется. До сих пор это было именно так: если вы создали страницу в «Фейсбуке», эта информация, скорее всего, по-прежнему существует, и большая часть людей, перестав пользоваться каким-либо сервисом, не тратят времени на удаление профиля. Если люди станут действовать иначе или если «Фейсбук» массово удалит свои архивы, баланс может быстро и непредсказуемо измениться.

(обратно)

152

Конечно, если не случится внезапного резкого роста смертности пользователей «Фейсбука» (и, вероятно, всего человечества в целом). В таком случае линии могут пересечься и завтра.

(обратно)

153

Во всяком случае, я на это надеюсь.

(обратно)

154

Во всяком случае, так считают канадцы и австралийцы.

(обратно)

155

У водорода и гелия массовая теплоемкость выше, но они представляют собой газы. Единственным более или менее привычным веществом с большей массовой теплоемкостью будет аммиак. Однако все три проигрывают воде, если речь об объемной теплоемкости.

(обратно)

156

Чтобы довести почти кипящую воду до кипения, потребуется всплеск энергии вдобавок к той, что нужна для нагрева до точки кипения. Это называется удельной теплотой парообразования.

(обратно)

157

Высота Ниагарского водопада x (ускорение свободного падения ? удельная теплоемкость воды) = 0,12 °С

(обратно)

158

Источник информации: доклад, который я сделал в третьем классе начальной школы, одетый в костюм Бенджамина Франклина.

(обратно)

159

И она ведь никогда не бьет дважды в одно и то же место, правильно?

(обратно)

160

Да-да, я просчитал эффективность использования торнадо для ветряных двигателей, и это еще менее практично, чем молния. Если выбрать любую точку в самом центре так называемой Аллеи торнадо, выяснится, что торнадо проходит через нее всего лишь раз в 4000 лет. Даже если вам удалось бы собрать всю мощность, набранную торнадо, все равно получилось бы меньше одного ватта.

(обратно)

161

Мощность Ниагарского водопада имеет мощность, сравнимую с хиросимской бомбой, взрывающейся каждые восемь часов! Атомная бомба, сброшенная на Нагасаки, была в 1,3 раза мощнее бомбы, сброшенной на Хиросиму! Для сравнения: легкий ветерок, дующий над прерией, тоже обладает кинетической энергией бомбы, сброшенной на Нагасаки.

(обратно)

162

Впрочем, ваш дом к этому времени уже все равно будет охвачен огнем из-за жара от плазмы, разлитой в воздухе.

(обратно)

163

Из-за кривизны земной поверхности для этого пришлось бы пройти 3619 км.

(обратно)

164

Экспедиция Амундсена к тому моменту уже покинула континент.

(обратно)

165

См. http://xkcd.com/795/.

(обратно)

166

Точнее давление вещества, разогретого этим жаром, и в меньшей степени давление излучения. – Прим. науч. ред.

(обратно)

167

Так называемый принцип Паули не позволяет двум электронам находиться в одном квантовом состоянии. Этот принцип – одна из основных причин того, что ваш ноутбук не проваливается сквозь ваши колени.

(обратно)

168

Возможно, существует еще одна категория объектов – они тяжелее нейтронных звезд, но недостаточно тяжелые, чтобы стать черными дырами. Их называют «странными звездами».

(обратно)

169

О, волшебный неразрушаемый пистолет, который можно держать в руке, и при этом вашу руку не оторвет! Не беспокойтесь, до этого мы еще доберемся.

(обратно)

170

Перед этим она совершит некоторое количество колебаний вокруг ядра: сила Кориолиса, вызванная вращением Земли, будет отклонять траекторию пули от прямой, так что при первом колебании она пройдет в стороне от центра Земли. – Прим. науч. ред.

(обратно)

171

Когда будете вытаскивать руку, следите за симптомами кессонной болезни, которая может возникнуть из-за пузырьков азота в крови, наполняющей сосуды вашей руки.

(обратно)

172

Столь сильное притяжение может привести к изменению солености воды вокруг пули: у самой пули соберется наиболее соленый раствор, который будет отталкивать вашу руки за счет архимедовой силы. На большем расстоянии от пули раствор будет менее соленым, и рука, наоборот, будет притягиваться к пуле. – Прим. науч. ред.

(обратно)

173

Иногда от способа измерения отказываются, потому что меры его ужасно неудобные – например градусы по Ранкину (градусы по Фаренгейту, которые отсчитывают от абсолютного нуля) или kcfs – кубические килофуты в секунду (мне приходилось читать техническую документацию, в которой они использовались). А иногда складывается ощущение, что ученым просто нравится поправлять других ученых.

(обратно)

Оглавление

  • Политех
  • Отказ от ответственности
  • Введение
  • Всемирный ураган
  • Релятивистский бейсбольный мяч
  • Купание в ядерном бассейне
  • Странные (и тревожные) вопросы из папки «Входящие» сайта «А что, если?»
  • Машина времени по-нью-йоркски
  • Где моя вторая половинка?
  • Лазерная указка
  • Периодическая стена элементов
  • Прыг-скок!
  • Моль кротов
  • Фен в ящике
  • Странные (и тревожные) вопросы из папки «Входящие» сайта «А что, если?»
  • Последний свет человечества
  • Пулеметный ракетный ранец
  • Равномерно вверх
  • Странные (и тревожные) вопросы из папки «Входящие» сайта «А что, если?»
  • Субмарина на орбите
  • Раздел коротких ответов
  • Все, что вы хотели знать про молнию…
  • Странные (и тревожные) вопросы из почтового ящика «А что, если?»
  • Мозг и компьютер
  • Планета Маленького принца
  • Стейк из поднебесья
  • Забить вратаря в ворота
  • Истребление простуды
  • Наполовину пустой стакан
  • Странные (и тревожные) вопросы из папки «Входящие» сайта «А что, если?»
  • Инопланетные астрономы
  • Больше никакой ДНК
  • Всепланетный самолет
  • Странные (и тревожные) вопросы из папки «Входящие» сайта «А что, если?»
  • Да пребудет с вами Сила!
  • Транзитные штаты
  • Падение с помощью гелия
  • Все на выход!
  • Странные (и тревожные) вопросы из папки «Входящие» сайта «А что, если?»
  • Самооплодотворение
  • Бросок вверх
  • Смертоносные нейтрино
  • Странные (и тревожные) вопросы из папки «Входящие» сайта «А что, если?»
  • Лежачий полицейский
  • Потерявшиеся бессмертные
  • Орбитальная скорость
  • Пропускная способность FedEx
  • Свободное падение
  • Странные (и тревожные) вопросы из папки «Входящие» сайта «А что, если?»
  • Спартанские стрелы
  • Осушить океаны
  • Осушить океаны, часть II
  • Уникальные твиты
  • LEGO-мост
  • Самый долгий закат
  • Случайный чих
  • Странные (и тревожные) вопросы из папки «Входящие» сайта «А что, если?»
  • Расширяющаяся планета
  • Невесомая стрела
  • Земля без Солнца
  • Правки в распечатанной «Википедии»
  • «Фейсбук» мертвецов
  • Закат над Британской империей
  • Помешивая чай
  • Все молнии мира
  • Самый одинокий человек
  • Странные (и тревожные) вопросы из папки «Входящие» сайта «А что, если?»
  • Капля дождя
  • Экзамен наугад
  • Нейтронная пуля
  • Странные (и тревожные) вопросы из папки «Входящие» сайта «А что, если?»
  • 15 баллов по шкале Рихтера
  • Слова благодарности
  • Источники и ссылки

  • Наш сайт является помещением библиотеки. На основании Федерального закона Российской федерации "Об авторском и смежных правах" (в ред. Федеральных законов от 19.07.1995 N 110-ФЗ, от 20.07.2004 N 72-ФЗ) копирование, сохранение на жестком диске или иной способ сохранения произведений размещенных на данной библиотеке категорически запрешен. Все материалы представлены исключительно в ознакомительных целях.

    Copyright © читать книги бесплатно